You are on page 1of 258

International Business

Sixteenth Edition, Global Edition

Chapter 5
Trade and Factor
Mobility Theory

Copyright © 2018 Pearson Education Limited. All Rights Reserved.


Factor Mobility and Trade Theory Questions
Objective 5-1
• Trade theory helps managers and government
policymakers focus on these questions:
• What products should we import and export?
• • How much should we trade?
• • With whom should we trade?
(notes)
Learning Objective 1: Understand why policymakers rely on international trade and factor mobility theories.

• Factor mobility is the movement of capital, technology, and people.

• Understanding trade theory helps managers to focus on these questions:


• What products should we import and export?
• How much should we trade?
• With whom should we trade?

Copyright © 2018 Pearson Education Limited. All Rights Reserved.


Interventionist Theories
Objective 5-2
• 1. MERCANTILISM?
Mercantilism holds that a country’s wealth is measured by its holdings of “treasure,”
which usually means its gold.

Notes
Learning Objective 2: Illustrate the historical and current rationale for interventionist and free trade theories.

• Mercantilism holds that a country’s wealth is measured by its holdings of “treasure,” which usually means its gold. This theory,
which formed the foundation of economic thought from about 1500 to 1800,2 holds that countries should export more than they
import (run a trade surplus) and, if successful, receive gold from countries that run deficits.

• Government policies to enact mercantilism: To run a trade surplus, governments restricted imports and subsidized noncompetitive
production.

• Balance of Trade: Some mercantilist terminology has endured. For example, a favorable balance of trade (also called a trade
surplus) still indicates that a country is exporting more than it imports. An unfavorable balance of trade (also known as a trade
deficit) indicates the opposite. These terms are misnomers because the word favorable implies “benefit,” and the word
unfavorable suggests “impairment.” In fact, running a trade surplus is not necessarily beneficial, nor is running a trade deficit
necessarily detrimental.

• Neomercantilism is the running of a favorable balance of trade to achieve some social or objective. For example, a country may
reduce unemployment by encouraging its companies to produce in excess of the home demand and send the surplus abroad. Or
it may attempt to maintain political influence in an area by sending more merchandise there than it receives, such as a
government granting merchandise aid or loans to a foreign government.

Copyright © 2018 Pearson Education Limited. All Rights Reserved.


• 1.1Governmental policies
• To run a trade surplus, governments restricted imports
and subsidized noncompetitive production. Countries
with colonies imported commodities from them that
they would otherwise have to purchase from
elsewhere.
• They monopolized colonial trade in order to force the
colonies to export less highly valued raw materials to
them and import more highly valued manufactured
products from them.

Copyright © 2018 Pearson Education Limited. All Rights Reserved.


• 1.2. The Concept of Balance of Trade
Some mercantilist terminology has endured. For
example, a favorable balance of trade (also called a
trade surplus) still indicates that a country is
exporting more than it imports. An unfavorable
balance of trade (also known as a trade deficit)
indicates the opposite

Copyright © 2018 Pearson Education Limited. All Rights Reserved.


2. NEOMERCANTILISM

• is the running of a favorable balance of trade to


achieve some social or political objective.
• For example, a country may reduce unemployment
by encouraging its companies to produce in excess of
the home demand and send the surplus abroad. Or it
may attempt to maintain political influence in an area
by sending more merchandise there than it receives,
such as a government granting merchandise aid or
loans to a foreign government.

Copyright © 2018 Pearson Education Limited. All Rights Reserved.


Free Trade Theories
Objective 5-2
• Why Trade at all?

• No nation has all the natural resources, geographic conditions, and technology necessary to produce
everything we consume today. In addition, two free trade theories further help answer this question: absolute
Advantage AND Comparative Advantage

(Notes )Learning Objective 2: Illustrate the historical and current rationale for interventionist and free trade theories.
• Why do countries need to trade at all? To begin with, no nation has all the natural resources, geographic conditions, and technology necessary to produce everything we
consume today.

• In 1776, Adam Smith declared that a country’s well-being is its citizens’ access to goods and services rather than the mercantilists’ concept of its ownership of gold. His
theory of absolute advantage holds that different countries produce different things more efficiently than others and that consumers should not have to buy domestically
produced goods when they can buy them more cheaply from abroad.

• A country’s natural advantage in production comes from climatic conditions, access to certain natural resources, or availability of certain labor forces. Most of today’s
world trade is in manufactured goods that compete through an acquired advantage, usually in either product or process technology.

• Comparative advantage says that global efficiency gains may still result from trade if a country specializes in what it can produce most efficiently—regardless of other
countries’ absolute advantage.

• Assumptions made about absolute and comparative advantage


• Fully employed resources—assumes use of full resources
• Economic efficiency--assumes the goal is maximum income
• Division of Gains—may forgo trading if to prevent others from gaining an advantage
• Transport costs—If it costs more to transport than the savings, the theories don’t work
• Insufficient Demand—Even with insufficient demand, there is still an advantage
• Statics and Dynamics—Looking at one point in time
• Services—The theories deal with products rather than services

Copyright © 2018 Pearson Education Limited. All Rights Reserved.


• 1. Theory of Absolute Advantage
Adam Smith (1776) declared that a country’s well-being
is its citizens’ access to goods and services rather than
the mercantilists’ concept of its ownership of gold.
This theory holds that different countries produce
different things more efficiently than others and that
consumers should not have to buy domestically
produced goods when they can buy them more cheaply
from abroad.

Copyright © 2018 Pearson Education Limited. All Rights Reserved.


• 1.1. Natural Advantage
A country’s natural advantage in production comes
from climatic conditions, access to certain natural
resources, or availability of certain labor forces.
• 1.2. Acquired Advantage Most of today’s world
trade is in manufactured goods that compete
through an acquired advantage, usually in either
product or process technology.

Copyright © 2018 Pearson Education Limited. All Rights Reserved.


Illustration of Absolute Advantage
Objective 5-2
Figure 5.2 Production Possibilities under Conditions of Absolute Advantage

Notes: Learning Objective 2: Illustrate


the historical and current
rationale for interventionist and free trade theories.
Comparison of wheat and tea production to illustrate absolute advantage.

Copyright © 2018 Pearson Education Limited. All Rights Reserved.


• 2. Theory of comparative advantage
• In 1817, David Ricardo examined the question,
“What happens when one country can produce all
products at an absolute advantage?”

• His resulting theory of comparative advantage


says that global efficiency gains may still result
from trade if a country specializes in what it can
produce most efficiently—regardless of other
countries’ absolute advantage.

Copyright © 2018 Pearson Education Limited. All Rights Reserved.


• Comparative advantage by analogy
• a country gains if it concentrates its resources on
the commodities it can produce most efficiently. It
then trades some of those for commodities
produced abroad. The following discussion
clarifies this theory.
• Production Possibility

Copyright © 2018 Pearson Education Limited. All Rights Reserved.


Illustration of Comparative Advantage
Objective 5-2
Figure 5.3 Production Possibilities under Conditions of Comparative Advantage

Notes:Learning Objective 2: Illustrate


the historical and current
rationale for interventionist and free trade theories.
Comparison of wheat and tea production to illustrate comparative advantage.

Copyright © 2018 Pearson Education Limited. All Rights Reserved.


Map of State and Relative Country Economies
Objective 5-2
Map 5.2 U.S. States’ Economies Compared to National Economies

Notes:Learning Objective 2: Illustrate


the historical and current
rationale for interventionist and
free trade theories.
This map shows the comparison between
US state economies and similar sized
country economies.

Copyright © 2018 Pearson Education Limited. All Rights Reserved.


Theories to Explain Trade Patterns
• How much does a country trade?
Objective 5-3
– Theory of country size :The theory of country size holds that countries with larger land
masses usually depend less on trade than smaller ones.
– Size of the economy: While land area helps explain the relative dependence on trade,
countries’ economic size helps explain absolute differences in the amount of trade
Notes:Learning Objective 3: Describe theories that explain national trade patterns.

• How much does a country trade?

• The theory of country size holds that countries with larger land masses usually depend less on trade than smaller
ones. They are apt to have more varied climates and an assortment of natural resources that make them more self-
sufficient.

• Size of the economy: While land area helps explain the relative dependence on trade, countries’ economic size
helps explain absolute differences in the amount of trade. The world’s largest five economies in 2014 were also the
top five exporting countries.
– Larger economies are the biggest traders because they produce and consume more.

What types of products should a country trade?


• Factor proportions theory: According to the factor proportions theory, countries have their best trade
advantage when depending on their relatively abundant production factors.

• With whom do countries trade?


• The country-similarity theory says that companies create new products in response to market conditions in
their home market. They then turn to markets they see as most similar to what they are accustomed,
especially those markets where consumers have comparable levels of per capita income.
Copyright © 2018 Pearson Education Limited. All Rights Reserved.
❖What types of products should a country trade?
• Factor Proportions Theory: According to the
factor proportions theory, countries have their
best trade advantage when depending on their
relatively abundant production factors
*
❖With whom do countries trade?
• Country-Similarity theory: says that companies
create new products in response to market
conditions in their home market.
Copyright © 2018 Pearson Education Limited. All Rights Reserved.
• They then turn to markets they see as most
similar to what they are accustomed, especially
those markets where consumers have
comparable levels of per capita income

Copyright © 2018 Pearson Education Limited. All Rights Reserved.


PLC Theory
*
• What is the Product Lifecycle Theory (PLC)?

• According to the PLC theory of trade, the production location for many products moves from one
country to another depending on the stage in the product’s life cycle
Objective 5-4

Notes: Learning Objective 4: Explain why a country’s export capabilities are dynamic.
• The international product life cycle (PLC) theory of trade states that the production location of certain manufactured products shifts as they
go through their life cycle. The cycle consists of four stages: introduction, growth, maturity, and decline

• The introduction stage is marked by


– • innovation in response to observed need,
– • exporting by the innovative country, and
– • evolving product characteristics.

• Growth is characterized by
• increases in exports by the innovating country,
• more competition,
• increased capital intensity, and
• some foreign production.

• Maturity is characterized by
– • a decline in exports from the innovating country,
– • more product standardization,
– • more capital intensity,
– • increased competitiveness of price, and
– • production start-ups in emerging economies.

• Decline is characterized by
– • a concentration of production in developing countries,
Copyright © 2018 Pearson Education Limited. All Rights Reserved.
– • an innovating country becoming a net importer.
• Introduction
• Growth
• Maturity
*
• Decline

Copyright © 2018 Pearson Education Limited. All Rights Reserved.


Limitations of PLC Theory Objective 5-4
• Exceptions to PLC Theory

• According to the diamond of national competitive advantage theory, companies’


development and maintenance of internationally competitive products depends on favorable
demand conditions, factor conditions, related and supporting industries, firm strategy,
structure, and rivalry.

Notes: Learning Objective 4: Explain why a country’s export capabilities are dynamic.

• Some types of products abound for which production locations usually do not shift. Such exceptions
include the following:
• Products with high transport costs (non-tradable goods) that may have to be produced close to
the market, thus never becoming significant exports.
• Products that, because of very rapid innovation, have extremely short life cycles, making it
impossible to reduce costs by moving production from one country to another. Some fashion
items fit this category.
• Luxury products for which cost is of little concern to the consumer. In fact, production in a
developing country may cause consumers to perceive the product as less luxurious.
• Products for which a company can use a differentiation strategy, perhaps through advertising,
to maintain consumer demand without competing on the basis of price.
• Products that require specialized technical personnel to locate near production so as to
continually move the products into their next generation of models. This seems to explain the
long-term U.S. dominance of medical equipment production and German dominance in rotary
printing presses.

Copyright © 2018 Pearson Education Limited. All Rights Reserved.


Diamond of National Competitive Advantage Theory
Objective 5-4 Michael porter
Figure 5.4 The Diamond of National Competitive Advantage
*
*All 4 have to be favorable

Source: Based on Michael E. Porter, “The Competitive Advantage of Nations,” Harvard Business Review, 68:2 (March–April 1990).
Why study this theory?
1-Analyze the external invironment
2-Gives managers an idea of the level of the competition (assesses competition)
3-Assesses the current and potential business
Copyright © 2018 Pearson Education Limited. All Rights Reserved.
Notes slide 21
Learning Objective 4: Explain why a country’s export capabilities are dynamic.

• According to the diamond of national competitive advantage theory, companies’


development and maintenance of internationally competitive products depends on
favorable
• demand conditions,
• factor conditions,
• related and supporting industries, and
• firm strategy, structure, and rivalry.
• Limitations of this theory:
• Observations of foreign or foreign-plus-domestic demand conditions have spurred
much of the recent Asian export growth. In fact, such Japanese companies as
Uniden and Fujitech target their sales almost entirely to foreign markets.
• Companies and countries do not depend entirely on domestic factor conditions. For
example, capital and managers are now internationally mobile, and companies may
depend on foreign locations for portions of their production.
• If related and supporting industries are not available locally, materials and
components are now more easily brought in from abroad because of transportation
advancements and relaxed import restrictions. In fact, many MNEs now assemble
products with parts supplied from a variety of countries.
• Companies react not only to domestic rivals but also to foreign-based rivals at home
and abroad. Thus the prior domestic absence of any of the four conditions from the
diamond may not inhibit companies.

Copyright © 2018 Pearson Education Limited. All Rights Reserved.


• Limitations of the Diamond of National Advantage
Theory: The existence of the four favorable national
conditions does not guarantee that a flourishing
industry will develop.
• Entrepreneurs may face favorable conditions for many
different lines of business. In fact, comparative
advantage theory holds that resource limitations may
cause a country’s firms to avoid competing in some
industries despite having an absolute advantage.

Copyright © 2018 Pearson Education Limited. All Rights Reserved.


Why Production Factors Move Objective 5-5
• Capital

• People

• Effects of factor movements


– Brain drain
Notes: Learning Objective 5: Summarize the reasons for and major effects of international factor movements.
• Capital, especially short-term capital, is the most internationally mobile production factor. Companies and
private individuals primarily transfer capital because of differences in expected return (accounting for risk) that
is caused by their outlooks of economic and political conditions.

• People are less mobile than capital. Some, of course, travel to other countries as tourists, students, and
retirees; however, this does not affect factor endowments because these travelers do not work in the
destination countries. Unlike funds that can be cheaply transferred by wire, people usually must incur high
transportation costs to work abroad.

• A controversial issue is the effect of outward migration on countries. On the one hand, countries lose
potentially productive resources when educated people leave—a situation known as a brain drain. On the
other hand, many of these people are now sending remittances back. For example, remittances account for 29
percent of Nepal’s GDP. There is also evidence that the outward movement and remittances of people leads to
an increase in start-up companies and capital in their home countries. countries receiving productive human
resources also incur costs by providing social services and acculturating people to a new language and
society.

Copyright © 2018 Pearson Education Limited. All Rights Reserved.


• What is the Factor Mobility Theory?
• This theory focuses on why production factors
move, the effects of that movement on
transforming factor endowments, and the impact
of international factor mobility (especially people)
on world trade.

Copyright © 2018 Pearson Education Limited. All Rights Reserved.


The Relationship Between Trade and Factor Mobility
Objective 5-6
• Substitution: There are pressures for the most abundant factors to move to
areas of scarcity.
Complementarity: Factor mobility through foreign
investment often stimulates trade because of the need for components, the
parent company’s ability to sell complementary products, the need for
equipment for subsidiaries.
Notes: Learning Objective 6: Assess the relationship between foreign trade and international factor mobility.

• Substitution: When the factor proportions vary widely among countries, pressures exist for the most abundant factors to
move to countries with greater scarcity, where they can command a better return. If permitted, many in the labor pool
where workers are unemployed or poorly paid, go to countries that have full employment and higher wages. They
receive higher wages not only because of the greater scarcity, but also because more capital-rich countries have
invested in machinery and infrastructure that make the imported laborers more productive than in their home countries.

• Complementarity:
• Factor mobility through foreign investment often stimulates trade because of
• the need for components,
• the parent company’s ability to sell complementary products, and
• the need for equipment for subsidiaries.
• Immigration enhances trade by creating ethnic enclaves of networks that link immigrants with their native
countries. The enclaves serve as niche markets for imports from their native countries (e.g., early U.S. soy sauce
imports sold mainly to Asian-Americans).

Copyright © 2018 Pearson Education Limited. All Rights Reserved.


International Business
Sixteenth Edition, Global Edition

Chapter 6
Trade Protectionism

Copyright © 2018 Pearson Education Limited. All Rights Reserved.


What is Protectionism? Objective 6-1
• What is protectionism?
• Governmental actions to influence international trade are known as
protectionism.
Trade protection: policy that protects domestic industries from unfair competition
from foreign ones < explanation
• Why do governments intervene in trade?
• Despite free-trade benefits, governments intervene in trade to attain economic,
social, or political objectives
Notes: Learning Objective 1: Recognize the conflicting outcomes of trade protectionism.

• Governmental actions to influence international trade are known as protectionism.

• Despite free-trade benefits, governments intervene in trade to attain economic, social, or political
objectives

• Proposals on trade regulations often spark fierce debate among people who believe they will be
affected—the so-called stakeholders. Of course, those most directly affected are most apt to speak
out, such as workers, owners, suppliers, and local politicians whose livelihoods depend on the
actions taken.
Copyright © 2018 Pearson Education Limited. All Rights Reserved.
What is Protectionism? Objective 6-1
• Stakeholders and protectionism:
• Proposals on trade regulations often spark fierce debate
among people who believe they will be affected—the so-
called stakeholders. Of course, those most directly
affected are most apt to speak out, such as workers,
owners, suppliers, and local politicians whose livelihoods
depend on the actions taken.
Notes:Learning Objective 1: Recognize the conflicting outcomes of trade protectionism.

• Governmental actions to influence international trade are known as protectionism.

• Despite free-trade benefits, governments intervene in trade to attain economic, social,


or political objectives

• Proposals on trade regulations often spark fierce debate among people who believe
they will be affected—the so-called stakeholders. Of course, those most directly
affected are most apt to speak out, such as workers, owners, suppliers, and local
politicians whose livelihoods depend on the actions taken.
Copyright © 2018 Pearson Education Limited. All Rights Reserved.
Institutional Factors Affecting the Flow of Trade
Objective 6-1
Figure 6.1 Institutional Factors Affecting the Flow of Goods and Service

Notes:Learning Objective 1: Recognize


the conflicting outcomes of trade
protectionism.

This figure illustrates the variety of factors


that can affect trade restrictions and trade
enhancements.
Copyright © 2018 Pearson Education Limited. All Rights Reserved.
Economic Rationales for Trade Restrictions
Objective 6-2
• To fight unemployment

• To protect infant industries

• To develop an industrial base

• Economic relationships with other countries


Notes:Learning Objective 2: Assess governments’ economic rationales and outcome uncertainties with international trade
intervention.
• Import restrictions to create domestic employment
• may lead to retaliation by other countries,
• affect large and small economies differently,
• reduce import handling jobs,
• may decrease jobs in another industry, or
• may decrease export jobs because of lower incomes abroad.
• The infant-industry argument says that production becomes more competitive over time because of
• increased economies of scale, and
• greater worker efficiency.
• To develop an industrial base: Since the industrial revolution, countries increasing their industrial bases grew their employment
and economies more rapidly. This observation led to protectionist arguments to spur local industrialization. These arguments have
been based on the following assumptions:
• Surplus workers can increase manufacturing output more easily than agricultural output.
• Import restrictions lead to foreign investment inflows, which provide jobs in manufacturing.
• Prices and sales of agricultural products and raw materials fluctuate widely, which is a detriment to economies that depend
heavily on them, especially if the dependence is on just one or a few commodities.
• Markets for industrial products grow faster than markets for both agricultural and raw material commodities.
• Economic Relationships with other countries: Nations monitor their absolute economic situations and compare their performance to
other countries. Among their many practices to improve their relative positions, four stand out: making balance-of-trade
adjustments, gaining comparable access to foreign markets, using restrictions as a bargaining tool, and controlling prices.
Copyright © 2018 Pearson Education Limited. All Rights Reserved.
Economic Rationales for Trade Restrictions
• Import restrictions to create domestic employment: Objective 6-2
• may lead to retaliation by other countries,
• affect large and small economies differently,
• reduce import handling jobs,
• may decrease jobs in another industry, or
• may decrease export jobs because of lower incomes abroad.

• The infant-industry argument says that production becomes more competitive over time because of:
• increased economies of scale, and
• greater worker efficiency.
Notes:Learning Objective 2: Assess governments’ economic rationales and outcome uncertainties with international trade intervention.
• Import restrictions to create domestic employment
• may lead to retaliation by other countries,
• affect large and small economies differently,
• reduce import handling jobs,
• may decrease jobs in another industry, or
• may decrease export jobs because of lower incomes abroad.
• The infant-industry argument says that production becomes more competitive over time because of
• increased economies of scale, and
• greater worker efficiency.
• To develop an industrial base: Since the industrial revolution, countries increasing their industrial bases grew their employment and economies more rapidly.
This observation led to protectionist arguments to spur local industrialization. These arguments have been based on the following assumptions:
• Surplus workers can increase manufacturing output more easily than agricultural output.
• Import restrictions lead to foreign investment inflows, which provide jobs in manufacturing.
• Prices and sales of agricultural products and raw materials fluctuate widely, which is a detriment to economies that depend heavily on them,
especially if the dependence is on just one or a few commodities.
• Markets for industrial products grow faster than markets for both agricultural and raw material commodities.
• Economic Relationships with other countries: Nations monitor their absolute economic situations and compare their performance to other countries. Among
their many practices to improve their relative positions, four stand out: making balance-of-trade adjustments, gaining comparable access to foreign markets,
using restrictions as a bargaining tool, and controlling prices.

Copyright © 2018 Pearson Education Limited. All Rights Reserved.


Economic Rationales for Trade Restrictions
Objective 6-2
• To develop an industrial base: Since the industrial revolution,
countries increasing their industrial bases grew their
employment and economies more rapidly. This observation led
to protectionist arguments to spur local industrialization.
Notes:Learning Objective 2: Assess governments’ economic rationales and outcome uncertainties with international trade
intervention.
• Import restrictions to create domestic employment
• may lead to retaliation by other countries,
• affect large and small economies differently,
• reduce import handling jobs,
• may decrease jobs in another industry, or
• may decrease export jobs because of lower incomes abroad.
• The infant-industry argument says that production becomes more competitive over time because of
• increased economies of scale, and
• greater worker efficiency.
• To develop an industrial base: Since the industrial revolution, countries increasing their industrial bases grew their employment
and economies more rapidly. This observation led to protectionist arguments to spur local industrialization. These arguments
have been based on the following assumptions:
• Surplus workers can increase manufacturing output more easily than agricultural output.
• Import restrictions lead to foreign investment inflows, which provide jobs in manufacturing.
• Prices and sales of agricultural products and raw materials fluctuate widely, which is a detriment to economies that
depend heavily on them, especially if the dependence is on just one or a few commodities.
• Markets for industrial products grow faster than markets for both agricultural and raw material commodities.
• Economic Relationships with other countries: Nations monitor their absolute economic situations and compare their
performance to other countries. Among their many practices to improve their relative positions, four stand out: making balance-
of-trade adjustments, gaining comparable access to foreign markets, using restrictions as a bargaining tool, and controlling
prices.
Copyright © 2018 Pearson Education Limited. All Rights Reserved.
Economic Rationales for Trade Restrictions
Objective 6-2
• Economic Relationships with other countries: Nations monitor their absolute
economic situations and compare their performance to other countries.
Among their many practices to improve their relative positions, four stand
out: making balance-of-trade adjustments, gaining comparable access to
foreign markets, using restrictions as a bargaining tool, and controlling
prices.
Notes:Learning Objective 2: Assess governments’ economic rationales and outcome uncertainties with international trade intervention.
• Import restrictions to create domestic employment
• may lead to retaliation by other countries,
• affect large and small economies differently,
• reduce import handling jobs,
• may decrease jobs in another industry, or
• may decrease export jobs because of lower incomes abroad.
• The infant-industry argument says that production becomes more competitive over time because of
• increased economies of scale, and
• greater worker efficiency.
• To develop an industrial base: Since the industrial revolution, countries increasing their industrial bases grew their employment and
economies more rapidly. This observation led to protectionist arguments to spur local industrialization. These arguments have been based
on the following assumptions:
• Surplus workers can increase manufacturing output more easily than agricultural output.
• Import restrictions lead to foreign investment inflows, which provide jobs in manufacturing.
• Prices and sales of agricultural products and raw materials fluctuate widely, which is a detriment to economies that depend heavily
on them, especially if the dependence is on just one or a few commodities.
• Markets for industrial products grow faster than markets for both agricultural and raw material commodities.
• Economic Relationships with other countries: Nations monitor their absolute economic situations and compare their performance to other
countries. Among their many practices to improve their relative positions, four stand out: making balance-of-trade adjustments, gaining
comparable access to foreign markets, using restrictions as a bargaining tool, and controlling prices.

Copyright © 2018 Pearson Education Limited. All Rights Reserved.


Why Government Intervenes in Trade
Objective 6-2
Table 6.1 Why Governments Intervene in Trade

Notes:Learning Objective 2: Assess


governments’ economic rationales and
outcome uncertainties with international trade intervention

This table illustrates the economic and noneconomic reasons governments


intervene in trade.
Copyright © 2018 Pearson Education Limited. All Rights Reserved.
Noneconomic Rationale for Trade Restrictions
Objective 6-3
• Maintain essential industries
• Promoting acceptable practices abroad
• Maintain or extend spheres of influence
• Preserve national culture
Notes:Learning Objective 3: Assess governments’ noneconomic rationales and outcome uncertainties with
international trade intervention.
• Maintaining essential industries (especially defense): not dependent on foreign supplies during hostile
political periods.
• Promoting acceptable practices abroad: Governments limit exports, even to friendly countries, of strategic
goods that might fall into the hands of potential enemies. They also limit exports and imports to compel a
foreign country to change some objectionable policy or capability. The rationale is to weaken the foreign
country’s economy by decreasing its foreign sales and by limiting its access to needed products, thus coercing
it to amend its practices on some issue such as human rights, environmental protection, military activities, and
production of harmful products.
• Maintaining or extending spheres of influence: Governments use trade to support their spheres of
influence—giving aid and credits to, and encouraging imports from, countries that join a political alliance or
vote a preferred way within international bodies.
• Preserving national culture: To help sustain a collective identity that sets their citizens apart from other
nationalities, governments prohibit exports of art and historical items deemed to be part of their national
heritage. In addition, they limit imports that may either conflict with or replace their dominant values.
Copyright © 2018 Pearson Education Limited. All Rights Reserved.
Instruments of Trade Control: Tariffs
Objective 6-4
• Tariff (Duty)
• Why are Tariffs levied?
• Types of Tariffs
Notes:Learning Objective 4: Describe the major instruments of trade control.
• Tariff barriers directly affect prices, and nontariff barriers may directly affect
either price or quantity. A tariff (also called a duty) is a tax levied on a good
shipped internationally. That is, governments charge a tariff on a good when
it crosses an official boundary— whether it be that of a nation or a group of
nations that have agreed to impose a common tariff on goods crossing the
boundary of their bloc.
• Tariffs may be levied:
• on goods entering, leaving, or passing through a country,
• for protection or revenue, or
• on a per-unit basis, a value basis, or both.
• A tariff assessed on a per-unit basis is a specific duty, on a percentage of
the item’s value an ad valorem duty, and on both a compound duty
Copyright © 2018 Pearson Education Limited. All Rights Reserved.
Instruments of Trade Control: Tariffs
Objective 6-4
• Tariff barriers directly affect prices, and nontariff barriers may directly
affect either price or quantity. A tariff (also called a duty) is a tax
levied on a good shipped internationally. That is, governments charge
a tariff on a good when it crosses an official boundary— whether it be
that of a nation or a group of nations that have agreed to impose a
common tariff on goods crossing the boundary of their bloc.
• Tariffs may be levied:
• on goods entering, leaving, or passing through a country,
• for protection or revenue, or
• on a per-unit basis, a value basis, or both.
• A tariff assessed on a per-unit basis is a specific duty, on a percentage of the item’s
value an ad valorem duty, and on both a compound duty

Copyright © 2018 Pearson Education Limited. All Rights Reserved.


Instruments of Trade Control: Non-tariff (1 of 3)
Objective 6-4
• Subsidies–Definition:
• Subsidies offer direct assistance to companies to
boost their competitiveness. Although this definition is
straightforward, disagreement on what constitutes a
subsidy causes trade frictions. In essence
Governmental subsidies may help companies be
competitive.
NotesLearning Objective 4: Describe the major instruments of trade control.
• Subsidies offer direct assistance to companies to boost their competitiveness. Although this definition
is straightforward, disagreement on what constitutes a subsidy causes trade frictions. In essence
Governmental subsidies may help companies be competitive.
• But there is little agreement on what a subsidy is.
• Agricultural subsidies are difficult to dismantle. Especially to overcome market imperfections
because they are at least controversial
• The one area in which everyone agrees that subsidies exist is agriculture especially in developed
countries. The official reason is that food supplies are too critical to be left to chance. Although
subsidies lead to surplus production, they are argued to be preferable to the risk of food shortages.

Copyright © 2018 Pearson Education Limited. All Rights Reserved.


Instruments of Trade Control: Non-tariff (1 of 3)
Objective 6-4

– Agriculture Subsidies:
Agricultural subsidies are difficult to dismantle.
Especially to overcome market imperfections
because they are at least controversial

Copyright © 2018 Pearson Education Limited. All Rights Reserved.


Instruments of Trade Control: Non-tariff (2 of 3)
Objective 6-4
• Aids and Loans

• Quotas

• Embargo

• Buy Local Legislation

• Standards and Labels

• Specific Permission Requirements

• Administrative Delays
Notes:Learning Objective 4: Describe the major instruments of trade control.
• Aid and Loans: When governments require foreign aid and loan recipients to spend the funds in the donor country, a situation known as
tied aid or tied loans, some otherwise noncompetitive output can compete abroad. For instance, tied aid helps win large contracts for
infrastructure, such as telecommunications, railways, and electric power projects.
• A quota limits the quantity of a product that can be imported or exported in a given time frame, typically per year. Import quotas normally
raise prices because they (1) limit supplies and (2) provide little incentive to use price competition to increase sales. A specific type of
quota that prohibits all trade is an embargo. As with quotas, a country or group of countries may place embargoes on either imports or
exports, on particular products regardless of origin or destination, on specific products with certain countries, or on all products with given
countries.
• Buy local legislation sets rules whereby governments give preference to domestic production in their purchases.
• Standards and Labels: Countries can devise classification, labeling, and testing standards to allow the sale of domestic products while
obstructing foreign-made ones.
• Specific Permission Requirements: Countries may require that importers or exporters secure governmental permission (an import or
export license) before transacting trade.
• Administrative delays: Closely akin to specific permission requirements are administrative customs delays that may be caused by
intention or inefficiency.
Copyright © 2018 Pearson Education Limited. All Rights Reserved.
Instruments of Trade Control: Non-tariff (3 of 3)
• Service Industries Objective 6-4
– Four factors to consider
▪ Essentiality
▪ Not-for-Profit Services
▪ Standards
▪ Immigration
Notes:Learning Objective 4: Describe the major instruments of trade control.
Service is the fastest-growing sector in international trade. In deciding whether to restrict service trade, countries
typically consider four factors: essentiality, not-for-profit preference, standards, and immigration.
• Essentiality Governments sometimes prohibit private companies, foreign or domestic, from operating in some sectors
because they feel the services are essential and provide social stability. In other cases, they set price controls or
subsidize government-owned service organizations that create disincentives for foreign private participation. Some
essential services
in which foreign firms might be excluded are media, communications, banking, utilities, and domestic transport.
• Not-for-Profit Services Mail, education, and hospital health services are often not-for profit sectors in which few
foreign firms compete. When a government privatizes these industries, it customarily prefers local ownership and
control.
• Standards Some services require face-to-face interaction between professionals and clients, and governments limit
entry into many of them to ensure practice by qualified personnel. The licensing requirements include such
professionals as accountants, actuaries, architects, electricians, engineers, gemologists, hairstylists, lawyers, medical
personnel, real estate brokers, and teachers.
• Immigration Satisfying the standards of a particular country is no guarantee that a foreigner can then work there. In
addition, governmental regulations often require an organization— domestic or foreign—to search extensively for
qualified personnel locally before it can even apply for work permits for personnel it would like to bring in from abroad.
Copyright © 2018 Pearson Education Limited. All Rights Reserved.
How Companies Deal with Governmental Influences
Objective 6-5
• Threats from import competition
– Options for companies:
• Move operations to another country.
• Concentrate on market niches that attract less international
competition.
• Adopt internal innovations, such as greater efficiency or
superior products.
• Try to get governmental protection.
Notes:Learning Objective 5: Classify how companies deal with governmental trade influences.
• When companies are threatened by import competition, they have several options, four of
which stand out.
• Move operations to another country.
• Concentrate on market niches that attract less international competition.
• Adopt internal innovations, such as greater efficiency or superior products.
• Try to get governmental protection.
Activity Minute:
Of these, which do you think is the best option for companies? Does it depend on the industry?
Copyright © 2018 Pearson Education Limited. All Rights Reserved.
International Business
Sixteenth Edition, Global Edition

Chapter 7
Economic Integration
and Cooperation

Copyright © 2018 Pearson Education Limited. All Rights Reserved.


What is Economic Integration?
Objective 7-1

• Economic Integration Definition: is a term used


to describe the political and monetary agreements
among nations and world regions in which
preference is given to member countries. There
are three major ways to approach such
agreements
Notes:Learning Objective 1: Define the three major types of international economic
integration

• Economic integration
• Global

Copyright © 2018 Pearson Education Limited. All Rights Reserved.


• Major ways to approach agreements
• Global integration :Countries from all over the
world decide to cooperate through the World
Trade Organization (WTO)
• Bilateral integration—Two countries decide to
cooperate more closely together, usually in the
form of tariff reductions
Regional integration—A group of countries located
in the same geographic proximity decide to
cooperate, as with the European Union
Copyright © 2018 Pearson Education Limited. All Rights Reserved.
GATT: Predecessor to WTO
Objective 7-2
• GATT
– General Agreement on Tariffs and Trade
• Trade without Discrimination
– Fundamental principle: each member nation must open
its markets equally to every other member nation.
Notes:Learning Objective 2: Explain what the World Trade Organization is and how it is working to reduce
trade barriers on a global basis
• In 1947, 23 countries formed the General Agreement on Tariffs and Trade (GATT) under the auspices of
the United Nations to abolish quotas and reduce tariffs. By the time the WTO replaced GATT in 1995, 125
nations had become members. Many believe that GATT’s contribution to trade liberalization enabled the
expansion of world trade in the second half of the twentieth century.
The fundamental principle of GATT was that each member nation must open its markets equally to every
other member nation. This principle of “trade without discrimination” was embodied in GATT’s most-favored-
nation (MFN) clause—once a country and its trading partners had agreed to reduce a tariff, that tariff cut
was automatically extended to every other member country, irrespective of whether the country was a
signatory to the agreement.
• GATT lead to the creation of the WTO, discussed next
• Restoring an effective means for trade liberalization led officials to create the World Trade Organization
(WTO) in 1995.
Copyright © 2018 Pearson Education Limited. All Rights Reserved.
The WTO Objective 7-2
• WTO :World Trade Organization
– Purpose ;
• WTO and “most favored nation clause”: The WTO continued the MFN clause of
GATT, which implies that member countries should trade without discrimination,
basically giving foreign products “national treatment.”

• WTO and Dispute Settlement: One function of the WTO that is garnering growing
attention is the organization’s dispute settlement mechanism, in which countries may
bring charges of unfair trade practices to a WTO panel, and accused countries may
appeal.
Notes:Learning Objective 2: Explain what the World Trade Organization is and how it is working to reduce trade barriers on a global
basis
• The WTO adopted the principles and trade agreements reached under the auspices of GATT but expanded its mission to include
trade in services, investment, intellectual property, sanitary measures, plant health, agriculture, and textiles, as well as technical
barriers to trade.
• The World Trade Organization is the major body for
• reciprocal trade negotiations,
• enforcement of trade agreements.
• Most Favored Nation: The WTO continued the MFN clause of GATT, which implies that member countries should trade without
discrimination, basically giving foreign products “national treatment.” Although the WTO restricts this privilege to official members,
some exceptions are allowed, especially for developing countries or countries that are part of a regional or bilateral trading group.
• Dispute Settlement: One function of the WTO that is garnering growing attention is the organization’s dispute settlement
mechanism, in which countries may bring charges of unfair trade practices to a WTO panel, and accused countries may appeal.
There are time limits on all stages of deliberations, and the WTO’s rulings are binding. If an offending country fails to comply with
the panel’s judgment, its trading partners have the right to compensation.
If this penalty is ineffective, then the offending country’s trading partners have the right to impose countervailing sanctions.
Copyright © 2018 Pearson Education Limited. All Rights Reserved.
Regional Economic Integration
Bilateral agreements: can be between two individual countries
or may involve one country dealing with a group of other
countries.
Geography matters:
Free trade area – no internal tariffs
Customs union – no internal tariffs plus common external tariffs.
Common Market customs union plus factor mobility.
Notes Regional Economic Integration
Bilateral Agreements can be between two individual countries or may involve one country dealing with a group of other
countries. Regional trade agreements are reciprocal pacts between two or more partners that lie somewhat between
bilateral treaties and the WTO. Some of the best known RTA are the European Union, the North American Free Trade
Agreement (NAFTA), the ASEAN (Association of Southeast Asian Nations) Free Trade Area (AFTA)

Geographic proximity is an important reason for economic integration.


Major types of economic integration :
• Free trade area – no internal tariffs
• Customs union – no internal tariffs plus common external tariffs.
• Common market – customs union plus factor mobility.

Regional integration has social, cultural, political and economic effects.


Copyright © 2018 Pearson Education Limited. All Rights Reserved.
The effects of Integration Objective 7-3
• Static Effects: are the shifting of resources from inefficient to efficient companies as trade
barriers fall

• - Trade Creation

• - Trade Diversion

• Dynamic Effects: are the overall growth in the market and the impact on a company caused
by expanding production and by its ability to achieve greater economies of scale.

- Economies of Scale

- Increased Competition
Notes Learning Objective 3: Summarize the major benefits of regional economic integration
• Static and Dynamic Effects Regional economic integration reduces or eliminates barriers for member countries, producing both static and
dynamic effects. Static Effects are the shifting of resources from inefficient to efficient companies as trade barriers fall. Dynamic effects
are the overall growth in the market and the impact on a company caused by expanding production and by its ability to achieve greater
economies of scale.
• Static Effects:
• Trade Creation: Production shifts to more efficient producers for reasons of comparative advantage, allowing consumers access
to more goods at lower prices than would have been possible without integration. Companies protected in their domestic markets
face real problems when the barriers are eliminated and they attempt to compete with more efficient producers.
• Trade Diversion: Trade shifts to countries in the group at the expense of trade with other countries, even though the nonmember
companies might be more efficient in the absence of trade barriers.
• Dynamic Effects:
• Economies of Scale Dynamic effects of integration occur when trade barriers come down and markets grow. The average cost
per unit falls as the number of units produced rises; occurs in regional integration because of the growth in the market size.
• Increased Competition Another important effect of an RTA is greater efficiency due to increased competition. Many MNEs in
Europe have attempted to grow through mergers and acquisitions to achieve the size necessary to compete in the larger market.

Copyright © 2018 Pearson Education Limited. All Rights Reserved.


The Impact of Trade Agreements
Notes Learning Objective 3:
Objective 7-3 Summarize the major benefits of
regional economic integration

Figure 7.1 Impact of Free Trade Agreements This figure shows the impact of trade
agreements.

Copyright © 2018 Pearson Education Limited. All Rights Reserved.


The European Union (EU) Objective 7-4
• The European Union: Changed from the European Economic Community to
the European Community to the European Union (EU). It is the largest and
most successful regional trade group.
• Characteristics of the EU:
• Free trade of goods, services, capital, and people
• Common external tariff
• Common currency
Notes European Union gradually began abolishing internal tariffs but eventually established and external tariff while integrating in other ways such as facilitating the free
movement of workers, establishing a common agricultural policy, and agreeing on a value added tax system. the formation of the European parliament and the
establishment of a common currency, the euro, make the EU the most ambitious of all the regional trade groups.
Learning Objective 4: Compare and contrast different regional trading groups

• The European Union: Changed from the European Economic Community to the European Community to the European Union (EU). It is the largest and most
successful regional trade group. Characteristics of the EU:
• Free trade of goods, services, capital, and people
• Common external tariff
• Common currency

• Because of the economic and human destruction left by World War II, European political leaders realized that greater cooperation among their countries would help
speed up recovery. Many organizations were formed, including the European Economic Community (EEC), which eventually emerged as the organization that would
bring together the countries of Europe into the most powerful trading bloc in the world.

• The EU encompasses many governing bodies, among which are the European Commission, European Council, European Parliament, European Court of Justice,
and European Central Bank
• The European Commission providers political leadership, drafts laws, and runs the various daily program of the EU.
• The Council of the European union, or European Summit, is composed of the heads of state of each member country.
• The three major responsibilities of the European Parliament are legislative power, control over the budget, and supervision of executive decisions.
• The European Court of Justice ensures consistent interpretation and application of EU treaties.
Copyright © 2018 Pearson Education Limited. All Rights Reserved.
• WHY EU?
• Because of the economic and human destruction
left by World War II, European political leaders
realized that greater cooperation among their
countries would help speed up recovery. Many
organizations were formed, including the
European Economic Community (EEC), which
eventually emerged as the organization that would
bring together the countries of Europe into the
most powerful trading bloc in the world.
Copyright © 2018 Pearson Education Limited. All Rights Reserved.

Other Aspects
Antitrust Investigations
of the EU Objective 7-4
• Monetary Union

• Schengen Area

• Expansion

• Bilateral Agreements

• Transatlantic Trade and Investment Partnership


Notes Learning Objective 4: Compare and contrast different regional trading groups

• Antitrust Investigations The EU has been very aggressive in enforcing antitrust laws in a variety of areas, including high-tech companies like Microsoft and Google on
charges that they were harming competitors because of their dominant market positions, Apple and Amazon on suspicion that they were receiving unfair tax advantages
from Ireland and Luxembourg respectively, and Facebook on allegations that it was violating privacy policies.

• Monetary Union: The Euro In 1992, the members of the EU signed the Treaty of Maastricht in part to establish a monetary union. The decision to move to a common
currency, the euro, in Europe has eliminated currency as a trade barrier for its adopters. As of 2016, 19 of 28 EU members had adopted the euro
• The euro is a common currency in Europe., and is administered by the European Central Bank which was established on 1 st January 1999, resulted in new banknotes in
2002.

• The Schengen Area In order to facilitate the free flow of people from country to country within the EU, the Schengen Agreement was signed in 1990 with gradual
implementation allowing citizens to cross internal borders without having to go through border checks. Migration and terrorism are threatening the open borders that are at
the heart of the Schengen Agreement.

• Expansion One of the EU’s major challenges is expansion. Official candidates for future membership currently include Turkey, Montenegro, Serbia, and the former
Yugoslav Republic of Macedonia. Turkey is an interesting candidate since it straddles Europe and Asia.
The EU expanded from 15 to 25 countries in 2004 with countries from mostly Central and Eastern Europe. Romania and Bulgaria were admitted in 2007 and Croatia in 2013,
bringing the number to 28.

• Bilateral Agreements In addition to reducing trade barriers for member countries, the EU has signed numerous bilateral free trade agreements with other countries outside
the region.

• The Transatlantic Trade and Investment Partnership (T-TIP) One of the more intriguing potential agreements involves the United States and the EU. Even though tariffs
between the two superpowers are already low (the United States and the EU have the world’s largest trading relationship and account for nearly half of the world’s
economic output), the new agreement would eliminate the remaining tariffs, boost trade between the regions, and aid in harmonizing product standards between them.
Copyright © 2018 Pearson Education Limited. All Rights Reserved.
How to Do business with the EU –
Implications for the Corporate
Strategy
• Determining where to produce
• Determining whether to grow through new
investments, through expanding existing
investments, through joint ventures or mergers.
• Balancing “common “denominators with national
differences.
Notes:Implications for the corporate strategy are:
• Companies need to determine where to produce products.
• Companies need to determine what their entry strategy will be.
• Companies need to balance the commonness of the EU with national
differences.

Copyright © 2018 Pearson Education Limited. All Rights Reserved.


The European Union Countries Objective 7-4
Notes Learning Objective 4: Compare and contrast different regional trading groups
This map shows the participating countries in the EU.
Yellow part is the members of the European union
Purple part is the EU candidate countries
Red part are the Members of the European Free Trade Association (EFTA)
Map 7.1 European Trade and Economic Integration

Copyright © 2018 Pearson Education Limited. All Rights Reserved.



NAFTA
What is NAFTA?
Objective 7-4
– North American Free Trade Agreement

• Reasons and Goals for NAFTA:

• NAFTA calls for the elimination of tariff and nontariff barriers, the harmonization of trade rules, the liberalization
of restrictions on services and foreign investment, the enforcement of intellectual property rights, and a dispute
settlement process.

Notes Various forms of mutual economic cooperation have historically existed between the US and Canada such as the Canada –
U.S. Free Trade Agreement of 1989, which eliminated all tariffs on bilateral trade. In February 1991, Mexico approached the US to
establish a free trade agreement, Canada was included in the formal negotiations and the resulting North American Free Trade
Agreements (NAFTA) became effective on January 1, 1994.

Learning Objective 4: Compare and contrast different regional trading groups

• The North American Free Trade Agreement


• includes Canada, the United States, and Mexico;
• went into effect on January 1, 1994;
• involves free trade in goods, services, and investment;
• is a large trading bloc but includes countries of different sizes and wealth.

• NAFTA rationale:
• U.S.–Canadian trade is the largest bilateral trade in the world.
• The United States is Mexico’s and Canada’s largest trading partner.

• NAFTA calls for the elimination of tariff and nontariff barriers, the harmonization of trade rules, the liberalization of restrictions on
services and foreign investment, the enforcement of intellectual property rights, and a dispute settlement process.
• NAFTA is a good example of trade diversion; some U.S. trade with and investment in Asia has been diverted to Mexico.

Copyright © 2018 Pearson Education Limited. All Rights Reserved.


Other Aspects of NAFTA Objective 7-4
• Rules of Origin Goods and services must originate in North America to get
access to lower tariffs.
• Impact of NAFTA:
• There are pros and cons to any trade agreement, and NAFTA is no exception. It is
obvious that trade and investment have increased significantly since the agreement
was signed in 1994.. As trade in agriculture increased with the advent of NAFTA, more
than a million farm jobs disappeared in Mexico due to U.S. competition.

• A major challenge to NAFTA is illegal immigration.


Notes:Learning Objective 4: Compare and contrast different regional trading groups

• Rules of Origin and Regional Content An important component of NAFTA is the concept of rules of origin and regional content.
Because it is a free trade agreement and not a customs union, each country sets its own tariffs to the rest of the world. Rules of
origin” ensure that only goods that have been the subject of substantial economic activity within the free trade area are eligible for
the more liberal tariff conditions created by NAFTA. This is a major contrast with the EU, which is a customs union rather than just an
FTA.
Regional Value Content Requirement One aspect of rules of origin in NAFTA refers to the Regional Value Content requirement.
According to regional content rules, at least 50 percent of the net cost of components, raw materials, and labor of most products
must come from the NAFTA region to qualify for the FTA.

• The Impact of NAFTA There are pros and cons to any trade agreement, and NAFTA is no exception. It is obvious that trade and
investment have increased significantly since the agreement was signed in 1994. U.S. goods and services trade with NAFTA totaled
$1.6 trillion in 2009 (according to the latest data available). U.S. goods trade with the two partners totaled $918 billion in 2010, with
the United States recording a trade deficit in goods. Immigration A major challenge to NAFTA is immigration. As trade in agriculture
increased with the advent of NAFTA, more than a million farm jobs disappeared in Mexico due to U.S. competition.
• A major challenge to NAFTA is illegal immigration.
Copyright © 2018 Pearson Education Limited. All Rights Reserved.
Economic Integration: MERCOSUR, Pacific
Alliance and CAN Objective 7-4
• What is Mercosur?

Mercosur is a customs union among Argentina, Brazil, Parguay, and Urguay.

• What is the Pacific Alliance?

Pacific Alliance includes Mexico, Colombia, Peru and Chile.

• What is CAN?

The Andean Community is one of the original regional economic groups but has not been
successful in achieving its original goals.
Notes Learning Objective 4: Compare and contrast different regional trading groups

• Mercosur The major trade group in South America is Mercosur, which was established in 1991 by Brazil, Argentina,
Paraguay, and Uruguay. Its major goal is to become a customs union with free trade within the bloc and a common
external tariff. Mercosur is classified as a customs union by the WTO for trade in goods and as an economic integration
agreement for trade in services.

• Pacific Alliance Mercosur has problems. It included Venezuela as a full member, and temporarily suspended Paraguay.
Brazil and Argentina have serious problems with protectionism. Frustration over these and other issues in both CAN and
Mercosur has lead to the creation in 2012 of the Pacific Alliance, comprising Mexico, Colombia, Peru, and Chile. These
countries refer to themselves as more hospitable to trade and investment due to their adherence to democracy and the
rule of law rather than the more populist and protectionist philosophies of other countries in CAN and Mercosur.

• Andean Community (CAN) Although the Andean Community (CAN) is not as significant economically as Mercosur, it is
the second most important official regional group in South America
Copyright © 2018 Pearson Education Limited. All Rights Reserved.
Asia’s Economic Integration Agreements
Objective 7-4
• ASEAN Free Trade Area – is a successful trade agreement among
countries in Southeast Asia.
• Asia Pacific Economic Cooperation (APEC) – comprises 21 countries
that border the Pacific Rim; progress toward free trade is hampered by size
and geographic distance between member countries and by the lack of a
treaty.
Notes Learning Objective 4: Compare and contrast different regional trading groups

• ASEAN Free Trade Area: On January 1, 1993, ASEAN officially formed the ASEAN Free Trade Area (AFTA) with the goal of
cutting tariffs on all intra-zonal trade to a maximum of 5 percent by January 1, 2008. The weaker ASEAN countries would be
allowed to phase in their tariff reductions over a longer period. By 2005, most products traded among the AFTA countries were
subject to duties from 0 to 5 percent, so AFTA has been successful in its objectives.

• Asia Pacific Economic Cooperation (APEC): Formed in November 1989 to promote multilateral economic cooperation in
trade and investment in the Pacific Rim,27 Asia Pacific Economic Cooperation (APEC) is composed of 21 countries that
border both Asia and the Americas. All but three members of AFTA are members of APEC, plus Canada, the United States,
Mexico, Peru, and Chile in the Americas; Australia and New Zealand; and China, Japan, Korea, Russia, and Chinese Taipei. It
is a large and powerful organization that is focused on a wide range of activities related to trade and investment, security,
energy, sustainability, anticorruption, and transparency, among other things.

• Trans-Pacific Partnership (TPP): The TPP was initiated by the United States to spur economic growth and create jobs, and it
involves Australia, Brunei, Canada, Chile, Japan, Malaysia, Mexico, New Zealand, Peru, Singapore, the United States, and
Vietnam. The formation of the initiative was announced in 2011, the agreement was concluded in October 2015 and signed by
the trade ministers in February 2016. However, the TPP will not actually come into effect until it is approved by the government
of each member, which is a difficult political task, especially during a contentious election year in the United States.
Copyright © 2018 Pearson Education Limited. All Rights Reserved.
• Trans-Pacific Partnership (TPP): The TPP was
initiated by the United States to spur economic growth and
create jobs, and it involves Australia, Brunei, Canada,
Chile, Japan, Malaysia, Mexico, New Zealand, Peru,
Singapore, the United States, and Vietnam. The formation
of the initiative was announced in 2011, the agreement
was concluded in October 2015 and signed by the trade
ministers in February 2016.
• However, the TPP will not actually come into effect until it
is approved by the government of each member, which is a
difficult political task, especially during a contentious
election year in the United States.
Copyright © 2018 Pearson Education Limited. All Rights Reserved.
Regional Economic Integration in
Africa
• Africa is truly the new frontier. The UN keeps
revising its estimates of population growth in
Africa.
• There are several African trade groups, but they
reply more on their former colonial powers and
other developed markets for trade than they do on
each other.
• Africa is complicated because of large number of
countries on the continent and the fact is that
there are 3 monetary regional unions and 17
trade blocs. Copyright © 2018 Pearson Education Limited. All Rights Reserved.
The United Nations Objective 7-4
• The first form of cooperation worth exploring is the United Nations, which was
established in 1945 in response to the devastation of World War II to promote
international peace and security and to help solve global problems in such diverse
areas as economic development, antiterrorism, and humanitarian actions.

• If the UN performs its responsibilities, it should improve the environment in which


MNEs operate around the world, reducing risk and providing greater opportunities
Notes Learning Objective 4: Compare and contrast different regional trading groups

• The United Nations The first form of cooperation worth exploring is the United Nations, which was established
in 1945 in response to the devastation of World War II to promote international peace and security and to help
solve global problems in such diverse areas as economic development, antiterrorism, and humanitarian
actions. If the UN performs its responsibilities, it should improve the environment in which MNEs operate
around the world, reducing risk and providing greater opportunities

• The UN family of organizations is too large to list, but it includes the WTO, the International Monetary Fund,
and the World Bank (the latter two discussed in subsequent chapters). These organizations are all part of the
Economic and Social Council, one of six principal organs of the UN System, which also includes the General
Assembly, the Security Council, and the International Court of Justice. The UN has 193 member states
represented in the General Assembly, including 15 that compose the Security Council.
• UNCTAD (The UN Conference on Trade and Development) was established to help developing countries
participate in international trade.
• NGOs (Nongovernmental Organizations) – Private non profit institutions that are independent of the
government.

Copyright © 2018 Pearson Education Limited. All Rights Reserved.


• UN Family of Organizations
• The UN family of organizations is too large to list, but it
includes the WTO, the International Monetary Fund, and
the World Bank .These organizations are all part of the
Economic and Social Council, one of six principal organs
of the UN System, which also includes the General
Assembly, the Security Council, and the International Court
of Justice. T
• he UN has 193 member states represented in the General
Assembly, including 15 that compose the Security Council.

Copyright © 2018 Pearson Education Limited. All Rights Reserved.


• UNCTAD (The UN Conference on Trade and
Development) was established to help developing
countries participate in international trade.
• NGOs (Nongovernmental Organizations) – Private
non profit institutions that are independent of the
government.

Copyright © 2018 Pearson Education Limited. All Rights Reserved.


Commodity Agreements Objective 7-5
• What are commodities?

• refer to raw materials or primary products that enter into trade, such as metals or
agricultural products. Primary commodity exports—such as crude petroleum, natural
gas, copper, iron ore, tobacco, coffee, cocoa, tea, and sugar—are still important to
developing countries.
Notes: The attempts of countries to stabilize commodity prices through producer alliance and commodity
agreements have been largely unsuccessful.

Learning Objective 5: Describe the forces that affect the prices of commodities and their impact on commodity
agreements.

• Commodities refer to raw materials or primary products that enter into trade, such as metals or agricultural
products. Primary commodity exports—such as crude petroleum, natural gas, copper, iron ore, tobacco, coffee,
cocoa, tea, and sugar—are still important to developing countries.
• Many commodity agreements now exist for the purpose of
• discussing issues,
• disseminating information, and
• improving product safety.

• The Organization of the Petroleum Exporting Countries (OPEC) is an example of a producer cartel that
relies on quotas to influence prices. It is a group of 13 oil-producing countries that have significant control over
supply and band together to control output and price. Its members include Algeria, Angola, Ecuador, Indonesia,
Iran, Iraq, Kuwait, Libya, Nigeria, Qatar, Saudi Arabia, the United Arab Emirates, and Venezuela. Several.

Copyright © 2018 Pearson Education Limited. All Rights Reserved.


• Purpose of Commodities Agreements
• discussing issues,
• disseminating information, and
• improving product safety.
• OPEC
• Organization of the Petroleum Exporting Countries
(is a producers’ alliance in oil that has been
successful in using quotas to keep oil prices high.)
Copyright © 2018 Pearson Education Limited. All Rights Reserved.
International Business
Sixteenth Edition, Global Edition

Chapter 12
Strategies for
International Business

Copyright © 2018 Pearson Education Limited. All Rights Reserved.


IB Strategy Objective 12-1
• Strategy in the MNE
• Evolving customer preferences, innovative competitors,
changing market structures, and shifting institutional contexts
create opportunities and threats.
• The job of the strategist is identifying the implications of these
situations to which products to make, where to make them,
where to sell them, how to compete, and, all the while, earn a
profit.
Notes: Learning Objective 1: Explain the idea of strategy in the MNE.

• In principle, strategy is an integrated set of choices and commitments that supports and sustains an MNE’s
competitiveness. It defines and communicates an MNE’s plan on how it will use its resources, capabilities, and
competencies to compete in different countries.

• Strategy maps an MNE’s plan to create value, both for itself and its stakeholders. Importantly, strategy
specifies what an MNE will do and what it will not do. Strategy calls on managers to deal with the questions
and complexities that follow from crosschecking opportunities with competencies, assessing competitive
threats, and setting and sustaining superior performance. As tough as that sounds, the performance track
record of MNEs consistently confirms that managers make it happen, formulating strategies that build
endlessly clever new ways to build productive and profitable enterprises.
Copyright © 2018 Pearson Education Limited. All Rights Reserved.
• In principle, strategy is an integrated set of choices
and commitments that supports and sustains an
MNE’s competitiveness. It defines and communicates
an MNE’s plan on how it will use its resources,
capabilities, and competencies to compete in different
countries.

Copyright © 2018 Pearson Education Limited. All Rights Reserved.


• Components of Strategy
Strategy Process Notes Learning Objective 1: Explain
the idea of strategy in the MNE.
Objective 12-1 • This figure outlines the strategy
Figure 12.1 The Role of Strategy in IB process for IB.

Copyright © 2018 Pearson Education Limited. All Rights Reserved.


Mission and Vision

• Vision is the idealization of what an MNE firm


wants to be. It expresses, in broad terms, its
ultimate goal.

• The MNE’s mission defines its business, its


objectives, and its approach to achieve them.

Copyright © 2018 Pearson Education Limited. All Rights Reserved.


Examples of Mission and Visons
Notes Learning Objective 1: Explain the idea of strategy in the MNE.
Objective 12-1
This table outlines some examples of mission/vision statements.

Table 12.1 Vision and Mission Statements, Leading MNEs

Copyright © 2018 Pearson Education Limited. All Rights Reserved.


Making sense to make strategy Objective 12-2
How do Executives Create Strategy?
• The complexity of the global business environment spurs
managers to integrate sensemaking perspectives into strategic
planning.
• Industrial Organizations Outlook: sets the external environment
as the primary determinant of an MNE’s strategic plan. (the
threats of new entrants, scale of entry barriers or power of
suppliers and buyers)
Notes Learning Objective 2: Profile how executives make strategy.

• Industrial Organization (IO) The IO outlook sets the external environment as the primary determinant of an
MNE’s strategic plan. It emphasizes the determinism of industry, structure given the thesis that its
characteristics (for instance, the threat of new entrants, scale of entry barriers, or power of buyers and supplies)
directly influence the potential profitability of an MNE’s strategy.

• Great by Choice Outlook In reality, some industries are, and persistently remain, far from- perfectly
competitive. In these settings, proprietary advantages, high entry barriers, or oligopolistic dynamics, for
instance, produce market imperfections. Consequently, some MNEs earn above-average, risk-adjusted returns,
while others in the same industry underperform. In this context, industry structure shapes, but does not
determine, a firm’s strategic performance.
Copyright © 2018 Pearson Education Limited. All Rights Reserved.
• Great by Choice Outlook: some industries are, and
persistently remain, far-from-perfectly competitive. In
these settings, proprietary advantages, high entry
barriers, or oligopolistic dynamics, for instance,
produce market imperfections.
• Consequently, some MNEs earn above-average, risk-
adjusted returns, while others in the same industry
under-perform. In this context, industry structure
shapes, but does not determine, a firm’s strategic
performance
Copyright © 2018 Pearson Education Limited. All Rights Reserved.
Tangible and Intangible Resources
Objective 12-3 Notes Learning Objective 3: Differentiate resources, capabilities, and
core competencies.

This table shows types of tangible and intangible resources.

Table 12.2 Resources of the Firm: Specification and Profile

Copyright © 2018 Pearson Education Limited. All Rights Reserved.


Resources, Capabilities and Core Competencies
Objective 12-3
• Resources are inputs into an MNE production process.
• A capability is the capacity for resources to perform an activity in an
integrated manner.
• Managers combine resources into capabilities, they also transform
resources and capabilities into core competencies. Difficult to define
precisely, most see a core competency as the special outlook, skill, or
technology that, by synthesizing links between resources and capabilities,
sets and sustains the firm’s capacity to create superior
• value.
Notes Learning Objective 3: Differentiate resources, capabilities, and core competencies.
• Collectively, resources represent the stocks of available factors that managers bundle together
into capabilities.

• Managers also transform resources and capabilities into core competencies. Difficult to define
precisely, most see a core competency as the special outlook, skill, or technology that, by
synthesizing links between resources and capabilities, sets and sustains the firm’s capacity to
create superior value

Copyright © 2018 Pearson Education Limited. All Rights Reserved.


Creation of Value Objective 12-4
• An MNE can create value by perfecting processes and products in order to
do things more efficiently than others, thereby making products for lower
costs than can competitors (the strategy of cost leadership).
• Alternatively, an MNE can create value by doing something no one else
can do, and doing it effectively, thereby making products for which
consumers pay a premium price (the strategy of differentiation).
Notes Learning Objective 4: Assess approaches to create value .

• The authors follow convention and define value in economic terms, specifying it as the difference between the cost of
making a product and the price that customers are willing to pay for it. If an MNE can sell its product for more than the
costs incurred to make it, it generates profits, and hence, creates value.

• An MNE implementing a cost leadership strategy aims to make a product at the lowest cost, relative to those offered
by rivals, which appeals to the largest number of potential customers.

• An MNE implements a differentiation strategy when it aims to do something no other firm can do, and, besides
doing it, doing it effectively. Just like cost leadership, a differentiation strategy is an integrated set of choices to make a
good or provide a service. Unlike the cost leadership strategy, the differentiation strategy requires designing and
delivering products that customers see as different in ways that are important to them—and thus are willing to pay a
premium price.

• Successfully implementing the integrated cost leadership/differentiation strategy requires an MNE adapt quickly to
change, particularly when disruptive innovations call for new capabilities. Production must optimize efficiency in order
to generate the funds that support differentiation.

Copyright © 2018 Pearson Education Limited. All Rights Reserved.


The Value Chain Objective 12-5
Figure 12.2 Visualizing the Value Chain

Copyright © 2018 Pearson Education Limited. All Rights Reserved.


Notes slide 12
Learning Objective 5: Diagram the features and functions of the value chain.

• Primary activities represent the core business functions that make and move products.

• Its organization follows from that of designing a product and building the operations that make it onward
through the tasks of logistics, marketing, distribution, and service. Primary activities reflect classic
business activities and managerial orientations. Thus, they carry functional labels such as operations or
marketing.

• Support activities. These represent the infrastructure of the firm, identifying the activities that support
the work done in carrying out primary activities. Human resources, for example, are needed for each
primary activity, from supervising warehousing materials, to directing production, to shipping products, to
serving customers.

• An MNE’s option to go anywhere in the world to perform a primary or support activity gives tremendous
choice of location. How an MNE distributes value activities around the world is the matter of
configuration—essentially, the task of deciding which activity to do where. In theory, configuration
ranges from concentrated (the MNE performs all value chain activities in one location) to dispersed
(the MNE performs different value-chain activities in different locations).

• Location Advantages Differing environmental conditions, given differing political, legal, and market
features, means costs differ from country to country. The option to go anywhere to do anything pushes
MNEs to exploit location advantages. Labor, capital, and resources costs are traditional determinants of
location advantages. Increasingly, the matters of digitization and cluster effects moderate configuration
choices.

• Economies of Scale The degree that an MNE concentrates or disperses its value-chain activities
reflects the importance of efficiency to its quest to create value.
Copyright © 2018 Pearson Education Limited. All Rights Reserved.
• Value-chain analysis helps managers understand the
potential and performance of resources and capabilities,
thereby clarifying cost structures and value creation

Copyright © 2018 Pearson Education Limited. All Rights Reserved.


Configuration Choices Objective 12-5
• Configuration definition: How an MNE distributes value activities around the world is
the matter of configuration essentially, the task of deciding which activity to do where.
– Factors that influence the configuration of a value-chain include
– business environment,
– • economies of scale,
– • innovation context,
– • logistics,resource costs,
Notes:Learning Objective 5: Diagram the features and functions of the value chain.
• An MNE’s option to go anywhere in the world to perform a primary or support activity gives tremendous choice of location. How an
MNE distributes value activities around the world is the matter of configuration—essentially, the task of deciding which activity to
do where. In theory, configuration ranges from concentrated (the MNE performs all value chain activities in one location) to
dispersed (the MNE performs different value-chain activities in different locations).

• Location Advantages Differing environmental conditions, given differing political, legal, and market features, means costs differ
from country to country. The option to go anywhere to do anything pushes MNEs to exploit location advantages. Labor, capital,
and resources costs are traditional determinants of location advantages. Increasingly, the matters of digitization and cluster effects
moderate configuration choices.

• Economies of Scale The degree that an MNE concentrates or disperses its value-chain activities reflects the importance of
efficiency to its quest to create value.

• Experience and Learning Effects Industry and firm conduct confirms that low costs create strategic advantage. Hence, MNEs look
to capitalize on the scale and scope of their operations to exploit potential cost minimization via experience and learning effects.

• The Risks of Configuration Choices Configuration decisions face the risks of unpredictable market change. Disruptions, such as
a regime change, material shortages, labor unrest, or currency instability, can quickly convert an efficient location into a costly one.

Copyright © 2018 Pearson Education Limited. All Rights Reserved.


Global Integration or Local Responsiveness?
Notes Learning Objective 6: Compare global integration and local responsiveness.
Objective 12-6
• Table 12.6 shows the motivations and characteristics of global integration versus local responsiveness .

• Competing in the global marketplace puts an MNE on the horns of a dilemma: should it single- mindedly
standardize products and processes and resolutely exploit location effects in order to maximize the efficiency
gains of global integration? Or, should it adapt products and processes to the unique situations in each
market in order to maximize the effectiveness benefits of local responsiveness.

Table 12.6 Motivations of Global Integration and Local Responsiveness

Copyright © 2018 Pearson Education Limited. All Rights Reserved.


Considerations with Global Objective 12-6
Integration or Local Responsiveness
• Global integration standardizes worldwide activities to
maximize efficiency, whereas
• local responsiveness adapts local activities to
optimize effectiveness.
Notes Learning Objective 6: Compare global integration and local responsiveness.
• The greater the potential to standardize value activities, the greater the importance of global integration to an MNE’s
competitiveness.

• Responding to local customers’ preferences requires customizing products and processes. Adaptation reduces the
efficiencies of standardization, thereby aggravating the liability of foreignness, inflating operational costs, and reducing
value creation. Hence, MNEs oppose adapting operations unnecessarily. Still, local imperatives often compel them to do
so.

• A third factor, institutional agents and their policy agendas, can, at different times, support either scenario. Various
transnational institutions, such as the IMF, WTO, and World Bank, build an increasingly seamless global business
environment. Systematically opening national borders to trade and investment creates greater potential for MNEs to
build, expand, and integrate global operations.

• Mapping the Interaction: Operating internationally calls for configuring and coordinating operations in ways that
reconcile the competing demands of global integration and local responsiveness. The Integration-Responsiveness
(IR) Grid provides a straightforward framework to organize analysis (shown on next slide).
Copyright © 2018 Pearson Education Limited. All Rights Reserved.
The Integration Responsiveness Grid
Objective 12-6
Notes Learning Objective 6: Compare global integration and local responsiveness.
• Figure 12.4 shows components to making a decision on local responsiveness
versus global integration.
Figure 12.4 The Integration-Responsiveness Grid

Copyright © 2018 Pearson Education Limited. All Rights Reserved.


International Corporate Level Strategies
• International Strategy Objective 12-6
• The international strategy transfers home-country-based competencies, such as
production expertise, design skills, or brand power, to foreign markets. Ultimate
control resides with headquarters, since senior executives understand the optimal
bundling of the company’s resources and capabilities.
Notes Learning Objective 7: Differentiate the types of strategies used by MNEs.
• MNEs competing in markets marked by low pressures for global integration and local responsiveness (the lower-left quadrant of
the IR Grid) have the flexibility to sell products designed for their home market, with minimal, if any, customization for foreign
markets. Moreover, they often face few, if any, rivals that offer a competitive product. In addition, their superior competitiveness
creates the flexibility to arbitrage location effects. Given this scenario, managers position the MNE to implement the international
strategy.

• Some MNE faces higher pressure for local responsiveness, but lower pressure to reduce costs via global integration (the lower-
right quadrant of the IR Grid). Cultural, political, legal, and economic conditions in foreign markets require MNEs, like J&J,
McDonald’s, Nestlé, or HSBC, adapt products and processes to local circumstances. Cost pressures shape, but do not determine,
local competitiveness. Still, offsetting the liability of foreignness as well as providing a reasonably priced alternative to local
products requires an MNE to optimize productivity. Efficiently customizing products and process from market to market, if
headquarters aims to direct activity despite high responsiveness pressures, is costly and complicated. Instead, an MNE adopts a
localization strategy, orienting its vision, mission, and plans to provide customers products that fit their distinctive preferences.

• MNEs competing in industries of the sort found in the upper-left quadrant of the IR Grid face high pressures for global integration
yet low pressures for local responsiveness. Industry effects press these firms to adopt a global strategy that is keenly sensitive to
the economics of efficiency, advantages of standardization, and the imperative of integration. Productivity pressures push an MNE
to, ideally, be the cost leader or, minimally, be competitive with the industry pacesetter.

• The asymmetric demands of maximizing efficiencies through cost leadership and effectiveness via differentiation make it difficult
to pursue both simultaneously. Still, some companies successfully adopt an integrated cost leadership/differentiation business
level strategy. The corporate-level analogue, the transnational strategy, takes this idea globally.

Copyright © 2018 Pearson Education Limited. All Rights Reserved.


• Localization Strategy
encourages decentralized decision-making so that
local subsidiaries can adjust value activities to local
circumstances
• The transnational strategy
targets the efficiency of global integration, the
effectiveness of local responsiveness and the
systematic diffusion of innovations.

Copyright © 2018 Pearson Education Limited. All Rights Reserved.


International Business
Sixteenth Edition, Global Edition

Chapter 13
Evaluation of Countries
for Operations

Copyright © 2018 Pearson Education Limited. All Rights Reserved.


THE IMPORTANCE OF LOCATION
Objective 13-1
• Because companies have limited resources, they
must be careful in choosing among countries when
making the following decisions:
✓The location of sales, production, and administrative
and auxiliary services, such as R&D.
✓The sequence for entering different countries.
✓The portion of resources and efforts to allocate to
each country where they operate.

Copyright © 2018 Pearson Education Limited. All Rights Reserved.


COMPARING COUNTRIES THROUGH
Objective 13-2
• Managers use scanning techniques to examine and
compare countries on broad indicators of opportunities
and risks.
• Why Is Scanning Important?
• Scanning is like seeding widely and then weeding out;
it is useful insofar as a company might otherwise
consider too few or too many possibilities. However,
comparison among countries is not always practical.

Notes Learning Objective 2: Illustrate why comparing countries through


scanning is important and how it connects to final location choices.
• This figure illustrates the location decision scanning process .
Copyright © 2018 Pearson Education Limited. All Rights Reserved.
Scanning and Detailed Analysis
Objective 13-2
• Scanning : Step 1
• During the scanning phase, information gathered might include:
• Yes or no For a question like, “Does the country allow 100
percent ownership of foreign direct investments?” the answer
is “yes” or “no.”
• Direct statistics For a question such as, “What is the highest
marginal tax rate on corporate earnings?” direct information is
available from tax schedules.
• Indirect indicators For a question such as, “What are the
potential sales for my product?” estimates must use indirect
indicators, such as those based on per capita GDP and
population size. Copyright © 2018 Pearson Education Limited. All Rights Reserved.
• Qualitative assessment For a question akin to, “What will
be the future political leaders’ philosophy about IB?” a
qualitative assessment is necessary based on different
opinions and indirect Indicators.
• Detailed Analysis: Step 2
• If Managers need to decide where best to emphasize
sales, they will likely need to visit the shortlisted countries
to observe the market and visit with distributors.

Copyright © 2018 Pearson Education Limited. All Rights Reserved.


Factors to Analyze Regarding Risk
Objective 13-3
➢Companies and their managers differ in their perceptions of
what is risky, how tolerant they are of taking risk, the returns
they expect, and the portion of their assets they are willing to
put at risk.
➢One company’s risk may be another’s opportunity. For example,
companies offering security solutions (e.g., alarm systems,
guard services, insurance, and armaments) may find their
biggest sales opportunities where other companies find only
risks. Companies offering risk-assessment services do better
when the perception of risk increases.
Notes Learning Objective 3: Discern major opportunity and risk variables and how to prioritize
and relate them when deciding whether and where to expand abroad.
• Factors to Consider in Analyzing Risk:

• Types of risk might include political risk, foreign exchange risk, natural disaster risk, competitive
risk,
Copyright © 2018 Pearson Education Limited. All Rights Reserved.
• Companies may reduce their risks by means other than
avoiding locations, such as by insuring. But all these options
incur costs.

• Risks may occur for suppliers and within suppliers’ supply


chains, thus companies need to examine the complex external
dependencies and vulnerabilities of its suppliers

Copyright © 2018 Pearson Education Limited. All Rights Reserved.


Shortcomings of Comparative Country
Information Objective 13-4
• Inaccuracy: for a variety of reasons, data could be outdated,
published results could be misleading, and questionable
methodology could be used.
▪ Non-Comparability: Countries do not necessarily publish
reports for the same length of time periods or at the same time
as each other. So a company must extrapolate in order to
estimate how countries compare.

Notes: Learning Objective 4: Summarize the sources and shortcomings


of comparative country information.

• Inaccuracy; Non comparability: Countries do not necessarily publish


reports for the same length of time periods or at the same time as each
other. So a company must extrapolate in order to estimate how countries
compare.
Copyright © 2018 Pearson Education Limited. All Rights Reserved.
Strategies for International Expansion:
Diversification, Concentration, Reinvesting
Objective 13-5
• Diversification Strategy: diversification strategy in the context
of IB location describes a company’s rapid movement into many
foreign markets, gradually increasing its commitment within
each one.
Notes: Learning Objective 5:Explain alternative
considerations and means for companies to allocate
resources among countries.
• Although any move abroad means some geographic
diversification, the term At the other extreme, with a

Copyright © 2018 Pearson Education Limited. All Rights Reserved.


• concentration strategy: the company will first move
to only one or a few foreign countries, not going
elsewhere until it develops a very strong involvement
and competitive position.
• Reinvesting and Harvesting: Reinvestment
Decisions Once committed to a given locale, a
company may need to reinvest its earnings there. The
failure to expand might result in not attaining its target
growth objectives.

Copyright © 2018 Pearson Education Limited. All Rights Reserved.


Moreover, headquarters management may delegate certain
investment decisions to experienced foreign subsidiary managers
because they believe that subsidiary management is the best judge
of what the operation needs.
Harvesting Companies commonly reduce commitments in some
countries because they have poorer performance prospects than
do others—a process known as harvesting (or divesting). Burger
King, for example, sold off underperforming operations in Korea
and Slovakia so as to have funds for more promising ventures in
the Chinese and Russian markets.

Copyright © 2018 Pearson Education Limited. All Rights Reserved.


NON-COMPARATIVE DECISIONS
Objective 13-6
• Decision Making Process:
• Most companies examine proposals one at a time, and
accept them if they meet minimum-threshold criteria,
because unforeseen opportunities give little time to
make decisions, and because of difficulty in
incorporating global performance into single analysis.

Notes:Learning Objective 6: Recognize why companies make


non-comparative decisions when choosing where to operate
abroad.
• Decision Making process:
Copyright © 2018 Pearson Education Limited. All Rights Reserved.
• Go-no-go decisions:
• One might expect companies to maintain a storehouse of
ranked foreign operating proposals, undertaking the best,
second best, etc. until they could make no further
commitments, but this is usually not the case
.
• They make go-no-go decisions by examining one
opportunity at a time and pursuing it if it meets some
threshold criteria.

Copyright © 2018 Pearson Education Limited. All Rights Reserved.


• Interdependence
• another factor inhibiting country operating comparison is
their interdependence. Profit figures from individual
operations may obscure the real impact on overall
company performance

Copyright © 2018 Pearson Education Limited. All Rights Reserved.


International Business: Environments and Operations, 15e (Daniels et al.)


Chapter 5 International Trade and Factor-Mobility Theory

1) Factor mobility refers to the movement of which of the following combinations?


A) capital, technology and people
B) currency, profits and raw materials
C) labor, capital and management
D) trade, profits and materials
Answer: A
Diff: 2
Learning Outcome: Define the fundamental concepts of international business
Skill: Concept
Objective: 4
AACSB: Application of knowledge

2) In the text, Costa Rica devised a strategic trade policy to accomplish what goals?
A) attract industries that promised high growth and higher wages
B) develop and grow local industries to keep capital in Costa Rica
C) attract industries that were "green" and produced sustainable products
D) attract companies that were acceptable to the U.S.
Answer: A
Diff: 2
Learning Outcome: Define the fundamental concepts of international business
Skill: Concept
Objective: 3
AACSB: Application of knowledge

3) Which of the following statements about Costa Rica and its approach to trade and factor-
mobility strategy is true?
A) Costa Rica decided to select industries that would produce competitive products and improve
workers' skills.
B) Costa Rica decided to attract industries acceptable to its major trading partners.
C) Costa Rica decided to select industries on a least cost basis.
D) Costa Rica decided to invite industries that were highly technical.
Answer: A
Diff: 3
Learning Outcome: Compare and contrast theories of international trade
Skill: Concept
Objective: 3
AACSB: Application of knowledge

1
Copyright © 2015 Pearson Education, Inc.

4) All countries face the questions of what, how much, and with whom they should import and
export. How they answer these questions primarily affects whether ________.
A) nontradable goods become tradable
B) products go through a lengthy life cycle
C) companies adhere to laissez-faire export policies
D) a company's present production location will be competitive
Answer: D
Diff: 2
Skill: Concept
Objective: 1

5) The trade theory that says a country should export more than it imports is known as ________.
A) mercantilism
B) absolute advantage
C) comparative advantage
D) import substitution
Answer: A
Diff: 1
Learning Outcome: Compare and contrast theories of international trade
Skill: Concept
Objective: 1
AACSB: Application of knowledge

6) Under mercantilism, governments sought to influence trade by ________.


A) establishing bilateral trading agreements with other countries
B) limiting exports
C) limiting imports and subsidizing exports
D) encouraging the development of manufacturing in their colonies
Answer: C
Diff: 2
Learning Outcome: Compare and contrast theories of international trade
Skill: Concept
Objective: 1
AACSB: Application of knowledge

7) Which of the following UNDESIRABLE results will most likely occur for a country running a
favorable balance of trade?
A) higher unemployment
B) higher domestic interest rates
C) fewer funds to invest abroad
D) granting credit that may be risky
Answer: D
Diff: 2
Skill: Application
Objective: 1
AACSB: Analytical thinking

2
Copyright © 2015 Pearson Education, Inc.

8) Neomercantilism describes the approach of countries that try to run a favorable balance of
trade to ________.
A) build up gold reserves
B) achieve a social or political objective
C) lower their rates of inflation
D) buy raw materials more cheaply
Answer: B
Diff: 1
Learning Outcome: Compare and contrast theories of international trade
Skill: Concept
Objective: 1
AACSB: Application of knowledge

9) According to Adam Smith's theory of absolute advantage, specialization allows countries to


increase their efficiency for each of the following reasons EXCEPT ________.
A) labor could become more skilled by repeating the same tasks
B) transportation costs could be lowered by producing closer to markets
C) labor would not lose time in switching from the production of one kind of product to another
D) higher production would provide incentives for the development of more effective working
methods
Answer: B
Diff: 2
Learning Outcome: Compare and contrast theories of international trade
Skill: Concept
Objective: 1
AACSB: Application of knowledge

10) Which of the following is most likely a basis for a Jamaican natural advantage in
international trade?
A) product technology developed by a Jamaican company
B) high literacy rates among Jamaican citizens
C) the use of English as the primary language
D) beautiful beaches and climate
Answer: D
Diff: 2
Skill: Application
Objective: 1
AACSB: Reflective thinking

3
Copyright © 2015 Pearson Education, Inc.

11) As a leading exporter of luxury automobiles, Germany has built a strong reputation in
engineering. Germany's trade most likely relies on a(n) ________.
A) outward immigration restriction
B) natural advantage
C) acquired advantage
D) neomercantilist policy
Answer: C
Diff: 2
Skill: Application
Objective: 1
AACSB: Application of knowledge

12) Assume the following conditions: In the United States it takes 5 units of resources to produce
a ton of potatoes and 10 to produce a ton of coal. In Canada it takes 6 units of resources to
produce a ton of potatoes and 9 to produce a ton of coal. According to the theory of absolute
advantage, ________.
A) the United States should export potatoes to Canada and import coal from Canada
B) the United States should export coal to Canada and import potatoes from Canada
C) the United States should import both potatoes and coal from Canada, while concentrating on
production of more valuable goods
D) there would be no basis for trade
Answer: A
Diff: 2
Learning Outcome: Compare and contrast theories of international trade
Skill: Application
Objective: 1
AACSB: Analytical thinking

13) Comparative advantage differs from absolute advantage in that the former ________,
whereas the latter ________.
A) holds that countries should specialize their production; does not
B) holds that trade should be kept as nearly in balance as possible; says countries should seek a
favorable balance of trade
C) bases trade on natural advantages; bases trade on acquired advantage
D) states that there is a basis of trade even if one country can produce everything more
efficiently than another country; does not deal with this issue
Answer: D
Diff: 2
Learning Outcome: Compare and contrast theories of international trade
Skill: Concept
Objective: 1
AACSB: Analytical thinking

4
Copyright © 2015 Pearson Education, Inc.

14) The comparative advantage theory holds that a country will gain from trade ________.
A) when it exports products for which it has an acquired advantage and imports products for
which another country has a natural advantage
B) if it exports goods it can produce more efficiently than other countries and imports goods
other countries can produce more efficiently than it can
C) even though it can produce all goods more efficiently than other countries
D) if it exports products using its abundant production factors in exchange for products for which
it has scarce production factors
Answer: C
Diff: 1
Learning Outcome: Compare and contrast theories of international trade
Skill: Concept
Objective: 1
AACSB: Application of knowledge

15) According to the theory of comparative advantage, a country gains from foreign trade even
though it may have an absolute advantage in the production of all products because ________.
A) the country will forego producing its less efficient output in order to produce its more
efficient output
B) workers become more efficient through specialization
C) economies of scale will reduce cost
D) there will be more incentive to develop cost-saving technologies
Answer: A
Diff: 2
Learning Outcome: Compare and contrast theories of international trade
Skill: Concept
Objective: 1
AACSB: Analytical thinking

16) Assume the following conditions: In the United States it takes 4 units of resources to produce
a ton of potatoes and 5 to produce a ton of coal. In Canada it takes 6 units of resources to
produce a ton of potatoes and 10 to produce a ton of coal. According to the theory of
comparative advantage, ________.
A) there would be no basis for trade
B) the United States should import potatoes from Canada and export coal to Canada
C) the United States should export both potatoes and coal to Canada
D) the United States should export potatoes to Canada and import coal from Canada
Answer: B
Diff: 2
Learning Outcome: Compare and contrast theories of international trade
Skill: Concept
Objective: 1
AACSB: Analytical thinking

5
Copyright © 2015 Pearson Education, Inc.

17) Which of the following assumptions was made in the original theories of absolute and
comparative advantage?
A) Specialized labor seeks efficiency.
B) Labor resources are fully employed.
C) Countries pursue objectives other than economic efficiency.
D) Production networks enable countries to concentrate on particular functions.
Answer: B
Diff: 2
Learning Outcome: Compare and contrast theories of international trade
Skill: Concept
Objective: 1
AACSB: Analytical thinking

18) The free trade theories of specialization primarily assume that ________.
A) specialization leads to unemployment, but production gains compensate for job losses
B) resources can move internationally from the production of one good to another
C) resources can move domestically from the production of one good to another
D) countries have objectives other than economic efficiency
Answer: C
Diff: 2
Learning Outcome: Compare and contrast theories of international trade
Skill: Concept
Objective: 2
AACSB: Application of knowledge

19) The free trade theories of specialization primarily assume that ________.
A) domestic resources are unable to move from the production of one good to another
B) countries have objectives other than economic efficiency
C) specialization triggers unemployment
D) resources are immobile internationally
Answer: D
Diff: 2
Learning Outcome: Compare and contrast theories of international trade
Skill: Concept
Objective: 2
AACSB: Analytical thinking

20) Nontradable goods are best defined as ________.


A) products and services for which exporting costs are excessive
B) factors of production that exceed safety regulations
C) goods used for national defense
D) products that comprise a portion of the inputs for finished products
Answer: A
Diff: 2
Skill: Concept
Objective: 3
AACSB: Application of knowledge

6
Copyright © 2015 Pearson Education, Inc.

21) Countries with varied climates and varied natural resources generally ________ than
countries with less varied climates and natural resources.
A) have lower per capita incomes
B) depend less heavily on trade
C) have more ethnic subgroups
D) have higher endowments of capital relative to labor
Answer: B
Diff: 2
Skill: Concept
Objective: 3
AACSB: Analytical thinking

22) Countries with large land areas are generally less dependent on trade than countries with
small land areas because of ________.
A) highly restricted economic scales
B) self-sufficiency with natural resources
C) higher transportation costs related to foreign trade
D) the development of unique products that have a limited demand
Answer: C
Diff: 2
Learning Outcome: Compare and contrast theories of international trade
Skill: Concept
Objective: 3
AACSB: Analytical thinking

23) Eight of the top 10 exporting and importing countries are countries with ________.
A) cheap labor forces
B) small land masses
C) natural advantages
D) developed economies
Answer: D
Diff: 2
Skill: Concept
Objective: 3
AACSB: Application of knowledge

7
Copyright © 2015 Pearson Education, Inc.

24) The trade theory that says countries should concentrate production on those products using
their most abundant production factors is the ________.
A) factor proportions theory
B) theory of comparative advantage
C) theory of absolute advantage
D) theory of nontradable goods
Answer: A
Diff: 1
Learning Outcome: Compare and contrast theories of international trade
Skill: Concept
Objective: 3
AACSB: Application of knowledge

25) El Salvador has a population density of about 620 people per square mile and neighboring
Honduras a population density of about 115 people per square mile. According to the factor
proportions theory of trade, one would expect El Salvador's exports to Honduras to ________.
A) have a lower labor-to-land ratio than its imports from Honduras
B) have a higher labor-to-land ratio than its imports from Honduras
C) embody more capital per square mile than its imports from Honduras
D) embody more capital per worker than its imports from Honduras
Answer: B
Diff: 3
Learning Outcome: Compare and contrast theories of international trade
Skill: Critical Thinking
Objective: 3
AACSB: Reflective thinking

26) Tests to substantiate the factor proportions theory have had mixed results most likely because
________.
A) labor migration quickly outdates any studies
B) most countries have a favorable balance of trade
C) labor skills and education are not homogeneous
D) large and small countries have different trade policies
Answer: C
Diff: 2
Learning Outcome: Compare and contrast theories of international trade
Skill: Concept
Objective: 3
AACSB: Analytical thinking

8
Copyright © 2015 Pearson Education, Inc.

27) Most world trade takes place ________.


A) between developed countries and developing countries
B) among developed countries
C) among developing countries
D) between raw material exporters and manufacturing exporters
Answer: B
Diff: 2
Skill: Concept
Objective: 3
AACSB: Application of knowledge

28) One way that developed countries specialize in order to gain acquired advantages is by
________.
A) allocating research efforts more heavily in specific sectors
B) emphasizing production in natural resource endowments
C) restricting imports to those in the service sector
D) subsidizing the transport of exports
Answer: A
Diff: 3
Skill: Concept
Objective: 3
AACSB: Analytical thinking

29) Why is the United States both an exporter and importer of such products as vehicles and
passenger aircraft?
A) Buyers procure similar products for replacement parts.
B) Transportation costs and cultural differences limit exporters.
C) Companies differentiate products to appeal to different consumers.
D) Bilateral trading agreements require this interchange for a number of products.
Answer: C
Diff: 2
Skill: Concept
Objective: 3
AACSB: Analytical thinking

30) Which of the following statements is most likely FALSE?


A) Developing countries trade mainly with developed countries.
B) Developed countries trade mainly with other developed countries.
C) Cultural similarity among countries enhances their trade with each other.
D) The greater the geographic distance between countries, the greater the trade.
Answer: D
Diff: 2
Skill: Concept
Objective: 3
AACSB: Reflective thinking

9
Copyright © 2015 Pearson Education, Inc.

31) According to the product life cycle theory, production and sales are primarily domestic in the
introductory stage because ________.
A) businesses need quick market feedback
B) tariff reductions remain under negotiation
C) international transport costs are too high
D) international patents have not been approved
Answer: A
Diff: 2
Learning Outcome: Compare and contrast theories of international trade
Skill: Concept
Objective: 4
AACSB: Application of knowledge

32) According to the PLC theory, at an early stage of a product's life cycle the product is likely
to be made in a more ________ method than in its later stages.
A) capital-intensive
B) labor-intensive
C) land-intensive
D) low-cost
Answer: B
Diff: 2
Learning Outcome: Compare and contrast theories of international trade
Skill: Concept
Objective: 4
AACSB: Analytical thinking

33) According to the PLC theory, what is the most likely reason that companies manufacture
products in locations with high labor rates during the introductory stage of a product's life cycle?
A) Doing so allows use of long production runs using capital-intensive methods.
B) Many consumers are willing to pay high costs for the newest products.
C) Transportation costs are reduced by focusing on markets in developed countries.
D) Import restrictions prevent production in countries other than the ones making product
innovations.
Answer: B
Diff: 3
Learning Outcome: Compare and contrast theories of international trade
Skill: Concept
Objective: 4
AACSB: Analytical thinking

10
Copyright © 2015 Pearson Education, Inc.

34) According to the PLC theory, developing countries have their best production advantage in
which stage of the product life cycle?
A) growth
B) maturity
C) decline
D) introduction
Answer: C
Diff: 2
Skill: Concept
Objective: 4
AACSB: Application of knowledge

35) Which of the hypothetical new products, if successful, would most likely diffuse its
production and sales according to the product life cycle theory?
A) a Ferrari sports car selling for $200,000 to a niche, upper-end market
B) a Sony television that receives global transmissions without a satellite dish or cable
connection, introduced at a high price but targeted eventually for sale to a mass market
C) a new Diet Coca-Cola soft drink flavored with cranberries
D) a Kyocera plastic chip carrier, which is expected to be quickly obsolete because of
innovations
Answer: B
Diff: 2
Learning Outcome: Compare and contrast theories of international trade
Skill: Application
Objective: 4
AACSB: Reflective thinking

36) All of the following are types of products that are usually exceptions to what is predicted by
the product life cycle theory EXCEPT ________.
A) trendy clothing
B) luxury items
C) differentiated products
D) consumer durables
Answer: D
Diff: 2
Skill: Concept
Objective: 4
AACSB: Analytical thinking

11
Copyright © 2015 Pearson Education, Inc.

37) Contrary to the product life cycle theory, there has been an increased tendency for companies
to ________.
A) sell products only in their home markets throughout the cycle
B) produce and sell products in countries where counterfeiting is low
C) introduce new products simultaneously in domestic and foreign markets
D) sell new products in developing countries before expanding into developed countries
Answer: C
Diff: 2
Learning Outcome: Compare and contrast theories of international trade
Skill: Concept
Objective: 4
AACSB: Analytical thinking

38) All of the following are features of the diamond of national advantage theory EXCEPT
________.
A) firm strategy, structure, and rivalry
B) strategic trade policy regulations
C) related and supporting industries
D) demand conditions
Answer: B
Diff: 1
Learning Outcome: Compare and contrast theories of international trade
Skill: Concept
Objective: 4
AACSB: Analytical thinking

39) The diamond of national advantage would be best used to answer which of the following
questions?
A) How do developed countries prevent the trade of blood diamonds?
B) How can developing countries create a significant trade surplus?
C) Why do specialized competitive advantages differ among countries?
D) Why do most innovative products originate in developed countries?
Answer: C
Diff: 2
Learning Outcome: Compare and contrast theories of international trade
Skill: Application
Objective: 4
AACSB: Reflective thinking

12
Copyright © 2015 Pearson Education, Inc.

40) According to the diamond of national advantage theory, the domestic existence of all four
conditions best explains ________.
A) the essence of an industry's development
B) the position of a product in its life cycle
C) where globally competitive firms develop and sustain themselves
D) why countries rely on abundant factor endowments
Answer: C
Diff: 2
Learning Outcome: Compare and contrast theories of international trade
Skill: Concept
Objective: 4
AACSB: Reflective thinking

41) Costa Rica applied the concepts of the diamond of national advantage theory to help
transform its economy by ________.
A) building domestic demand for its products and services
B) looking globally to develop favorable conditions
C) following import substitution policies
D) concentrating on nontradable goods
Answer: B
Diff: 2
Skill: Application
Objective: 4
AACSB: Application of knowledge

42) Which of the following best supports a nation's decision to implement a strategic trade
(industrial) policy?
A) The policies have usually resulted in big payoffs.
B) Governments, rather than entrepreneurs, should take the risks of developing new industries.
C) Consumer needs would otherwise not be met.
D) Government actions should target industries that are believed to give the country its best
export advantages.
Answer: D
Diff: 3
Learning Outcome: Compare and contrast theories of international trade
Skill: Critical Thinking
Objective: 4
AACSB: Analytical thinking

13
Copyright © 2015 Pearson Education, Inc.

43) A governmental strategic trade (industrial) policy is one that ________.


A) lessens dependence on foreign military goods
B) seeks reciprocal free trade agreements between developed and developing countries
C) targets the resources needed to support industries that seem to fit best with the country's
advantages
D) limits imports to encourage domestic production of what would otherwise be imported
Answer: C
Diff: 2
Skill: Concept
Objective: 4
AACSB: Application of knowledge

44) Which of the following statements most likely undermines the arguments for a strategic trade
policy?
A) Entrepreneurs, rather than governments, should take business risks.
B) Developed countries have production lead time over developing countries.
C) If big companies can figure out what products to develop, so can governments.
D) Governments have limited resources, so industries with greater growth potential should be
targeted.
Answer: A
Diff: 3
Skill: Critical Thinking
Objective: 4
AACSB: Analytical thinking

45) Between now and 2050, countries undergoing a simultaneous population reduction and an
increased percentage of retirees in the population most likely will need more ________.
A) imports
B) immigrants
C) domestic products
D) favorable balances of trade
Answer: B
Diff: 2
Skill: Concept
Objective: 5
AACSB: Reflective thinking

46) The most internationally mobile factor of production is ________.


A) labor
B) management
C) long-term capital
D) short-term capital
Answer: D
Diff: 1
Skill: Concept
Objective: 5
AACSB: Application of knowledge
14
Copyright © 2015 Pearson Education, Inc.

47) All the following are reasons for the lower international mobility of people than capital
EXCEPT which one?
A) The differences in economic return between countries are lower for people than for capital.
B) It is more expensive to move people than capital.
C) People may have to learn another language and adapt to a different culture.
D) International capital transfers have less cumbersome legal restrictions than international
movements of people.
Answer: A
Diff: 2
Skill: Application
Objective: 5
AACSB: Analytical thinking

48) All of the following are examples of international labor mobility EXCEPT ________.
A) a manager assigned by an MNE to work short-term abroad
B) a refugee who takes up employment in another country
C) a college student on a study abroad program
D) an illegal immigrant working in another country
Answer: C
Diff: 2
Skill: Application
Objective: 5
AACSB: Analytical thinking

49) Labor and capital mobility are intertwined because ________.


A) illegal workers sometimes pay large sums to people who smuggle them into another country
B) people cannot emigrate to another country unless they bring capital with them
C) companies receive tax benefits for hiring immigrants
D) immigrants bring an investment in human capital with them
Answer: D
Diff: 2
Skill: Concept
Objective: 5
AACSB: Analytical thinking

50) Brain drain is a term that describes ________.


A) a country's loss of educated, productive people
B) a nation's shift from an emerging market to a developed economy
C) the exportation of high-technology products in exchange for low-technology products
D) the unauthorized use of patents, copyrights, and technology by the counterfeiting industry
Answer: A
Diff: 2
Skill: Concept
Objective: 5
AACSB: Application of knowledge

15
Copyright © 2015 Pearson Education, Inc.

51) Which of the following is the LEAST likely result of outward migration?
A) Emigrants remit capital used to start businesses in their home countries.
B) Nations receive more foreign aid.
C) Nations lose highly educated and productive workers.
D) Countries receive remittances from emigrants.
Answer: B
Diff: 3
Skill: Concept
Objective: 5
AACSB: Reflective thinking

52) Country X brings in a large number of unskilled immigrants to meet its labor needs. Which
of the following is the LEAST likely to occur in Country X as a result?
A) Country X will develop a long-term class of "have-nots" if the children of immigrants remain
unskilled.
B) Country X will need more unskilled people in the future if immigrant children become
skilled.
C) The continual supply of unskilled labor will delay the development of labor saving
technology in Country X.
D) Country X will experience a significant transfer of knowledge and a resulting rise in start-up
businesses.
Answer: D
Diff: 3
Skill: Application
Objective: 5
AACSB: Reflective thinking

53) In this example, assume that both trade and production factors are internationally mobile.
Using domestic labor, the labor cost per silicon chip is $.10 in Japan and $.20 in the United
States. Using domestic capital, the capital cost per chip is $.10 in Japan and $.05 in the United
States. Chip transportation in either direction is $.10. Japanese labor is willing to work in the
United States for $.15 per chip including the workers' transport cost. U.S. capital will go to Japan
at a cost of $.08 per chip including transaction costs. Which silicon chips will the United States
buy?
A) those made in Japan with Japanese labor and capital
B) those made in the United States with U.S. labor and capital
C) those made in Japan with Japanese labor and U.S. capital
D) those made in the United States with Japanese labor and U.S. capital
Answer: D
Diff: 3
Skill: Application
Objective: 6
AACSB: Analytical thinking

16
Copyright © 2015 Pearson Education, Inc.

54) The inability of a company to gain foreign production factors to use in its domestic
operations most likely ________.
A) entices the company to export its own production factors
B) stimulates the company to adopt efficient substitute methods of production
C) stimulates foreign companies to invest in that domestic market
D) causes the company's products to move more rapidly through their life cycles
Answer: B
Diff: 2
Skill: Concept
Objective: 6
AACSB: Analytical thinking

55) All of the following are reasons that foreign investment may stimulate exports from the
home (donor) country EXCEPT which of the following?
A) The investor often sends home-country components to its facilities abroad.
B) The investor often sends home-country equipment to its facilities abroad.
C) Home-country aid usually goes where the investments are made.
D) The foreign facility is adept at selling the investors' home country products.
Answer: C
Diff: 2
Skill: Concept
Objective: 6
AACSB: Analytical thinking

56) Analysts conclude that the finite supply of natural resources will most likely ________.
A) cause a higher proportion of world trade to occur among developed countries
B) cause a higher proportion of world trade to occur among developing countries
C) serve as an advantage for developing countries in their sales to developed countries
D) create a greater portion of world trade in the form of services
Answer: C
Diff: 2
Skill: Concept
Objective: 6
AACSB: Analytical thinking

57) Research shows that urbanization leads to higher productivity. Which of the following is the
LEAST likely reason for this?
A) Urbanization establishes an inflow of immigrants who will work for less money.
B) In urban areas, firms can more easily find people with the exact skills they need.
C) Urbanization allows for economies in moving supplies and finished products.
D) In urban areas, knowledge flows more easily from one company to another and from one
industry to another.
Answer: A
Diff: 2
Skill: Concept
Objective: 6
AACSB: Analytical thinking
17
Copyright © 2015 Pearson Education, Inc.

58) The premise that there will be more finely tuned specialization of production among
countries is most likely based on the idea that companies will ________.
A) depend more on production of nontradable goods and services
B) encourage urbanization in emerging countries that lack infrastructure
C) establish facilities in dispersed locations to hinder domestic competition
D) take advantage of wage and skill differences among countries
Answer: D
Diff: 2
Skill: Concept
Objective: 6
AACSB: Analytical thinking

59) Costa Rica's recent economic transformation to depend more on high-tech manufacturing is
largely due to its adoption of ________ trade policy.
A) an import substitution
B) a comparative advantage
C) a factor proportions
D) a strategic
Answer: D
Diff: 2
Skill: Application
Objective: 1
AACSB: Reflective thinking

60) All of the following factors most likely led to Costa Rica's attraction of high-tech FDI
EXCEPT the country's ________.
A) demand for high-tech products
B) literacy rate
C) quality of life
D) political stability
Answer: A
Diff: 2
Learning Outcome: Explain the implications of foreign direct investment for both host and home
country
Skill: Concept
Objective: 1
AACSB: Reflective thinking

61) Countries enact trade policies based on trade theories, which, in turn, affect companies'
optimum production locations.
Answer: TRUE
Diff: 1
Learning Outcome: Compare and contrast theories of international trade
Skill: Concept
Objective: 1
AACSB: Reflective thinking

18
Copyright © 2015 Pearson Education, Inc.

62) Understanding trade theories helps companies know if they will need to compete against
goods and services produced abroad.
Answer: TRUE
Diff: 1
Learning Outcome: Compare and contrast theories of international trade
Skill: Concept
Objective: 1
AACSB: Reflective thinking

63) Under mercantilism, colonial powers sought to run trade surpluses with their colonies by
preventing the colonies from manufacturing.
Answer: TRUE
Diff: 2
Learning Outcome: Compare and contrast theories of international trade
Skill: Concept
Objective: 1
AACSB: Analytical thinking

64) An objective of neomercantilism is to build up gold reserves.


Answer: FALSE
Diff: 1
Learning Outcome: Compare and contrast theories of international trade
Skill: Concept
Objective: 1
AACSB: Application of knowledge

65) The theory of absolute advantage holds that there are advantages to trade because different
countries can produce different goods more efficiently than others.
Answer: TRUE
Diff: 1
Learning Outcome: Compare and contrast theories of international trade
Skill: Concept
Objective: 1
AACSB: Application of knowledge

66) Under the theory of absolute advantage, countries hold two types of advantages—acquired
advantages and technological advantages.
Answer: FALSE
Diff: 2
Learning Outcome: Compare and contrast theories of international trade
Skill: Concept
Objective: 1
AACSB: Analytical thinking

19
Copyright © 2015 Pearson Education, Inc.

67) The comparative advantage theory holds that a country will gain from trade even though it
can produce all goods more efficiently than other countries.
Answer: TRUE
Diff: 1
Learning Outcome: Compare and contrast theories of international trade
Skill: Concept
Objective: 1
AACSB: Analytical thinking

68) Comparative advantage theory holds that gains from trade are the result of capital
movements from countries with low interest rates to countries with higher interest rates.
Answer: FALSE
Diff: 2
Learning Outcome: Compare and contrast theories of international trade
Skill: Concept
Objective: 1
AACSB: Analytical thinking

69) The free trade theories of specialization assume that countries will be better off even though
some people are unemployed.
Answer: FALSE
Diff: 2
Learning Outcome: Compare and contrast theories of international trade
Skill: Concept
Objective: 1
AACSB: Application of knowledge

70) According to theories of specialization in international trade, gains occur because specialized
workers move to countries that can use their skills more effectively.
Answer: FALSE
Diff: 2
Learning Outcome: Compare and contrast theories of international trade
Skill: Concept
Objective: 1
AACSB: Reflective thinking

71) Countries with large land areas are generally less dependent on trade than countries with
small land areas.
Answer: TRUE
Diff: 1
Skill: Concept
Objective: 3
AACSB: Reflective thinking

20
Copyright © 2015 Pearson Education, Inc.

72) The top 10 exporting countries are dominated by developing countries.


Answer: FALSE
Diff: 2
Skill: Concept
Objective: 3
AACSB: Application of knowledge

73) The factor proportions theory holds that countries should concentrate production on those
products that use their most abundant production factors.
Answer: TRUE
Diff: 2
Learning Outcome: Compare and contrast theories of international trade
Skill: Concept
Objective: 1
AACSB: Application of knowledge

74) The factor proportions theory holds that countries should improve their competitiveness by
importing capital and skilled employees from abroad.
Answer: FALSE
Diff: 2
Learning Outcome: Compare and contrast theories of international trade
Skill: Concept
Objective: 1
AACSB: Analytical thinking

75) Most world trade takes place between raw material exporters and manufacturing exporters.
Answer: FALSE
Diff: 2
Skill: Concept
Objective: 3
AACSB: Application of knowledge

76) Trade occurs more between culturally dissimilar countries than between culturally similar
countries.
Answer: FALSE
Diff: 2
Skill: Concept
Objective: 3
AACSB: Analytical thinking

21
Copyright © 2015 Pearson Education, Inc.

77) Many products' location of production will shift internationally as the products go through
their life cycle.
Answer: TRUE
Diff: 1
Learning Outcome: Compare and contrast theories of international trade
Skill: Concept
Objective: 4
AACSB: Analytical thinking

78) According to the PLC theory of trade, most new products are produced in countries where
wage rates are low.
Answer: FALSE
Diff: 2
Learning Outcome: Compare and contrast theories of international trade
Skill: Concept
Objective: 4
AACSB: Application of knowledge

79) Luxury products are the most likely types of products to behave according to the product life
cycle theory of trade.
Answer: FALSE
Diff: 2
Learning Outcome: Compare and contrast theories of international trade
Skill: Concept
Objective: 4
AACSB: Application of knowledge

80) An exception to the PLC theory in terms of production location is often a product with very
rapid change in innovation.
Answer: TRUE
Diff: 2
Learning Outcome: Compare and contrast theories of international trade
Skill: Concept
Objective: 4
AACSB: Application of knowledge

81) The four favorable domestic conditions of the diamond of national advantage help to explain
how and where globally competitive companies develop and sustain themselves.
Answer: TRUE
Diff: 2
Learning Outcome: Compare and contrast theories of international trade
Skill: Concept
Objective: 4
AACSB: Application of knowledge

22
Copyright © 2015 Pearson Education, Inc.

82) The existence of the four favorable conditions of the diamond of national advantage does not
guarantee that an industry will develop in a given locale.
Answer: TRUE
Diff: 2
Learning Outcome: Compare and contrast theories of international trade
Skill: Concept
Objective: 4
AACSB: Analytical thinking

83) A strategic trade policy is one that develops industries to lessen dependence on foreign
military goods.
Answer: FALSE
Diff: 1
Skill: Concept
Objective: 4
AACSB: Application of knowledge

84) Companies and individuals transfer capital internationally primarily because of expectations
of earning higher returns.
Answer: TRUE
Diff: 1
Skill: Concept
Objective: 5
AACSB: Application of knowledge

85) The international mobility of labor includes workers, students, tourists, and retirees who
travel to another country.
Answer: FALSE
Diff: 2
Skill: Concept
Objective: 5
AACSB: Analytical thinking

86) The foreign-born population as a percentage of total population is substantial for some
countries and insignificant for others.
Answer: TRUE
Diff: 2
Skill: Concept
Objective: 5
AACSB: Application of knowledge

23
Copyright © 2015 Pearson Education, Inc.

87) The term brain drain describes the export of high-technology products in exchange for low-
technology products.
Answer: FALSE
Diff: 1
Skill: Concept
Objective: 5
AACSB: Application of knowledge

88) The combination of free trade and free movement of production factors offers maximum
production efficiency.
Answer: TRUE
Diff: 2
Skill: Concept
Objective: 6
AACSB: Analytical thinking

89) The finite nature of natural resources should work as a disadvantage for the export prices of
developing countries.
Answer: FALSE
Diff: 2
Skill: Concept
Objective: 6
AACSB: Reflective thinking

90) The growth of small-scale production technology will most likely enable small countries to
produce products efficiently for their own consumption.
Answer: TRUE
Diff: 2
Skill: Concept
Objective: 6
AACSB: Analytical thinking

91) As an international business manager, how can you benefit from an understanding of
international trade theories?
Answer: Trade policies have an impact on business because they affect which countries can
produce given products more efficiently and whether countries will permit imports to compete
against their domestically produced goods and services. In turn, a country's policies influence
which products companies might export to given countries, as well as what and where companies
can produce in order to sell in the given countries.
Diff: 3
Learning Outcome: Compare and contrast theories of international trade
Skill: Application
Objective: 1
AACSB: Reflective thinking

24
Copyright © 2015 Pearson Education, Inc.

92) In a short essay, discuss the theory of mercantilism, and discuss favorable and unfavorable
balances of trade as they apply to international business.
Answer: Mercantilism holds that a country's wealth is measured by its holdings of treasure,
which usually meant gold. According to the theory, countries should export more than they
import and, if successful, receive gold from countries that run deficits. To export more than they
imported, governments imposed restrictions on most imports and subsidized production of many
products that could otherwise not compete in domestic or export markets.

A favorable balance of trade indicates that a country is exporting more than it is importing. An
unfavorable balance of trade indicates that a country is importing more than it is exporting,
which is known as a deficit. However, it is not necessarily beneficial to run a trade surplus nor is
it necessarily disadvantageous to run a trade deficit. A country that is running a surplus, or
favorable balance of trade, is, for the time being, importing goods and services of less value than
those it is exporting. In effect, the surplus country is granting credit to the deficit country. If that
credit cannot eventually buy sufficient goods and services, the so-called favorable trade balance
actually may turn out to be disadvantageous for the country with the surplus.
Diff: 3
Learning Outcome: Compare and contrast theories of international trade
Skill: Application
Objective: 1
AACSB: Reflective thinking

93) In a short essay, discuss the theory of absolute advantage and the reasons a country's
efficiency improves based on this theory.
Answer: The theory of absolute advantage holds that different countries produce some goods
more efficiently than other countries; thus, global efficiency can increase through free trade.
Developed by Adam Smith, the theory of absolute advantage says the real wealth of a country
consists of the goods and services available to its citizens. Smith reasoned that if trade were
unrestricted, each country would specialize in those products that gave it a competitive
advantage. Each country's resources would shift to the efficient industries because the country
could not compete in the inefficient ones. Through specialization, countries could increase their
efficiency because of three reasons:
a. Labor could become more skilled by repeating the same tasks.
b. Labor would not lose time in switching from the production of one kind of product to
another.
c. Long production runs would provide incentives for the development of more effective
working methods.
Diff: 3
Learning Outcome: Compare and contrast theories of international trade
Skill: Synthesis
Objective: 1, 2
AACSB: Analytical thinking

25
Copyright © 2015 Pearson Education, Inc.

94) What is the difference between the free trade theories of absolute advantage and comparative
advantage? How can free trade improve global efficiency?
Answer: Absolute advantage holds that different countries produce some goods more efficiently
than other countries; thus, global efficiency can increase through free trade. Based on this theory,
Adam Smith questioned why the citizens of any country should have to buy domestically
produced goods when they could buy those goods cheaper abroad. But what happens when one
country can produce all products at an absolute advantage? David Ricardo examined this
question and expanded on Adam Smith's theory of absolute advantage to develop the theory of
comparative advantage. Ricardo reasoned that there may still be global efficiency gains from
trade if a country specializes in products that it can produce more efficiently than other
products—regardless of whether other countries can produce those same products even more
efficiently.
Diff: 3
Learning Outcome: Compare and contrast theories of international trade
Skill: Synthesis
Objective: 1, 2
AACSB: Analytical thinking

95) What assumptions underlie the theories of specialization in international trade? What are the
limitations of these assumptions?
Answer: The assumptions that underlie the theories of specialization in international trade
include the following:
a. Full employment: When countries have many unemployed or unused resources, they may
seek to restrict imports to employ or use idle resources.
b. Economic efficiency: Countries may pursue objectives other than output efficiency. They
may avoid overspecialization because of the vulnerability created by changes in technology.
c. Division of gains: If a country perceives a trading partner is gaining too large a share of
benefits, it may forgo absolute gains for itself so as to prevent relative losses.
d. Two countries, two commodities: Two countries trading only two commodities is unrealistic.
e. Transport costs: If it costs more to transport the goods than is saved through specialization,
then the advantages of trade are negated.
f. Statics and dynamics: The relative conditions that give countries advantages or
disadvantages in the production of given products are dynamic, not static, as the theories view
countries' advantages.
g. Services: An increasing portion of world trade is in services, and the theories deal with
commodities.
h. Production networks: Specialization may take place by function or by component as well as
by final product.
i. Mobility: The assumption that resources can move domestically from the production of one
good to another, and at no cost, is not completely valid.
Diff: 3
Learning Outcome: Compare and contrast theories of international trade
Skill: Critical Thinking
Objective: 2
AACSB: Analytical thinking

26
Copyright © 2015 Pearson Education, Inc.

96) What is the theory of country size? How is country size determined? How does country size
affect national trade patterns?
Answer: The theory of country size says that countries with large land areas are more likely to
have varied climates and an assortment of natural resources than smaller countries, thus making
them more self-sufficient. Although land area is the most obvious way of measuring a country's
size, countries also can be compared on the basis of economic size. Distance to foreign markets
affects trade patterns in large and small countries differently. Normally, the farther the distance,
the higher the transport costs, the longer the inventory carrying time, and the greater the
uncertainty and unreliability of timely product delivery. In addition, large countries' production
and market centers are more likely to be located at a greater distance from other countries,
raising the transport costs of foreign trade.
Diff: 3
Learning Outcome: Compare and contrast theories of international trade
Skill: Synthesis
Objective: 1, 3
AACSB: Analytical thinking

97) What is the country similarity theory? According to this theory, what factors affect trade
patterns?
Answer: Observations of trade patterns reveal that most of the world's trade occurs among
countries that have similar characteristics, specifically among industrial, or developed, countries.
The country similarity theory says that once a company has developed a new product in response
to observed market conditions in the home market, it will turn to markets it sees as most similar
to those at home. In addition, markets in industrial countries can support products and their
variations. Thus, companies from different countries produce different product models, and each
may gain some markets abroad.
Diff: 2
Learning Outcome: Compare and contrast theories of international trade
Skill: Synthesis
Objective: 1, 3
AACSB: Analytical thinking

27
Copyright © 2015 Pearson Education, Inc.

98) In a short essay, discuss the four stages of the international product life cycle.
Answer: The international product life cycle theory of trade states that certain kinds of products
go through a continuum, or cycle, that consists of four stages—introduction, growth, maturity,
and decline. The location of production will shift internationally depending on the stage of the
cycle.
a. Introduction: Most new products are produced in and exported from developed countries
because of their combined demand conditions and labor skills. Many reasons account for the
dominant position of developed countries, including competition, demanding consumers, the
availability of scientists and engineers, and high incomes. Early production also generally occurs
in a domestic location so the company can obtain rapid market feedback, as well as save
transport costs.
b. Growth: As sales of the new product grow, competitors enter the market. At the same time,
demand is likely to grow substantially in foreign markets, particularly in other developed
countries. In fact, demand may be sufficient to justify producing in some foreign markets to
reduce or eliminate transport charges, but the output at this stage is likely to stay almost entirely
in the foreign country with the additional manufacturing unit. The original producing country
will also increase its exports in this stage but lose certain key export markets in which
competitors commence local production.
c. Maturity: In this stage, worldwide demand begins to level off, although it may be growing in
some countries and declining in others. There is often a "shake-out" of producers such that
product models become highly standardized, making cost an important competitive weapon.
Longer production runs become possible for foreign plants, which in turn reduce per-unit cost for
their output. The lower per-unit costs create demand in developing countries.
d. Decline: As a product moves to the decline stage, those factors occurring during the mature
stage continue to evolve. The markets in developed countries decline more rapidly than those in
developing countries as affluent customers demand newer products. By this time, market and
cost factors have dictated that almost all production is in developing countries, which export to
the declining or small-niche markets in developed countries. In other words, the country in which
the innovation first emerged and exported from then becomes the importer.
Diff: 3
Learning Outcome: Compare and contrast theories of international trade
Skill: Application
Objective: 4
AACSB: Analytical thinking

28
Copyright © 2015 Pearson Education, Inc.

99) What are the arguments for and against nations developing and implementing strategic trade
policies?
Answer: The two basic approaches to government policy are to alter conditions that will affect
industry in general—a non-strategic approach—and to alter conditions that will affect a targeted
industry—a strategic approach. Regardless of whether a government takes a general or specific
approach, it may alter the competitive positions of specific companies and production locations.

The first approach means altering conditions that affect factor proportions, efficiency, and
innovation. A country may upgrade production factors by improving human skills through
education, providing infrastructure, promoting a highly competitive environment so that
companies must make improvements, and inducing consumers to demand a higher quality of
products and services.

The second approach is to target specific industries. This approach has usually resulted in only
small payoffs, largely because governments find it difficult to identify and target the right
industries. Moreover, too many countries may identify the same industries.
Diff: 3
Learning Outcome: Compare and contrast theories of international trade
Skill: Critical Thinking
Objective: 1
AACSB: Analytical thinking

100) From an economic standpoint, why do production factors move from one country to
another? How does factor movement affect international trade?
Answer: Capital, especially short-term capital, is the most internationally mobile production
factor. Companies and private individuals primarily transfer capital because of differences in
expected return. Short-term capital is more mobile than long-term capital, especially direct
investment, because there is more likely to be an active market through which investors can
quickly buy foreign holdings and sell them if they want to transfer capital back home or to
another country. Furthermore, investors feel more certain about short-term political and
economic conditions in a foreign country than about long-term ones. People are also
internationally mobile. Unlike funds that can be cheaply transferred by wire, people must usually
incur high transportation costs to work in another country. Although international mobility of
production factors may be a substitute for trade, the mobility may stimulate trade through sales
of components, equipment, and complementary products. If trade could not occur and production
factors could not move internationally, a country would have to either forego consuming certain
goods or produce them differently, which in either case would usually result in decreased
worldwide output and higher prices. In some cases, however, the inability to gain sufficient
access to foreign production factors may stimulate efficient methods of substitution, such as the
development of alternatives for traditional production methods.
Diff: 3
Skill: Synthesis
Objective: 5, 6
AACSB: Analytical thinking

29
Copyright © 2015 Pearson Education, Inc.
International Business: Environments and Operations, 15e (Daniels et al.)
Chapter 6 Governmental Influence on Trade

1) Protectionism will most likely affect which aspect of a company's operations?


A) selecting employees
B) investing short-term capital
C) acquiring foreign supplies
D) relocating personnel overseas
Answer: C
Diff: 2
Learning Outcome: Define the fundamental concepts of international business
Skill: Concept
Objective: 1
AACSB: Application of knowledge

2) The term protectionism, when applied to international trade, refers to ________.


A) governmental restrictions and competitive support actions to affect trade flows
B) payments to dock workers to prevent pilferage of imported shipments
C) border checks to prevent entry of illegal aliens
D) methods used to prevent intellectual property theft
Answer: A
Diff: 1
Learning Outcome: Define the fundamental concepts of international business
Skill: Concept
Objective: 1
AACSB: Application of knowledge

3) Why should managers have an understanding of trade protectionism?


A) Trade protectionism targets factor endowments, thus affecting the best country to locate
production.
B) Trade protectionism affects a company's ability to sell abroad and ability to compete at
home.
C) Trade protectionism affects the number of people permitted to practice a specific
profession.
D) Trade protectionism prevents companies' enactment of merger and acquisition
agreements.
Answer: B
Diff: 2
Learning Outcome: Define the fundamental concepts of international business
Skill: Concept
Objective: 1
AACSB: Analytical thinking
4) Managers should understand the effect of trade protectionism because ________.
A) trade protectionism may limit the number of people permitted to practice a specific
profession
B) trade protectionism requires the payment of high insurance rates to transport goods
internationally
C) trade protectionism may prevent companies' enactment of merger and acquisition
agreements
D) trade protectionism may make it difficult for a company to buy what it needs from foreign
suppliers
Answer: D
Diff: 2
Learning Outcome: Define the fundamental concepts of international business
Skill: Concept
Objective: 1
AACSB: Application of knowledge

5) The government of Country X imposes import restrictions on steel to help the domestic
steel industry in depressed areas. What is the most likely result of such restrictions?
A) damaging other industries in Country X
B) devaluing the currency of Country X
C) lowering steel prices in Country X
D) triggering boycotts in Country X
Answer: A
Diff: 2
Learning Outcome: Discuss arguments for and against government intervention in
international business
Skill: Application
Objective: 1
AACSB: Analytical thinking

6) Assume a government places restrictions on a specific product from a specific foreign


country. What would be the government's most likely concern about the foreign country's
response?
A) the foreign producers raising the prices of their exports
B) the foreign country restricting its own imports
C) the foreign country restricting its exports in that industry
D) the foreign producers seeking other markets
Answer: B
Diff: 2
Learning Outcome: Discuss arguments for and against government intervention in
international business
Skill: Application
Objective: 1
AACSB: Analytical thinking
7) Assume a government is considering import restrictions on sugar because sugar imports
are hurting the domestic industry. Which of the following groups is LEAST likely to speak
out on the subject?
A) politicians in areas that grow sugar
B) employees of sugar companies
C) sugar company owners
D) sugar consumers
Answer: D
Diff: 2
Skill: Application
Objective: 2
AACSB: Analytical thinking

8) What is the most likely reason that consumers rarely protest import restrictions that raise
the prices they pay for a specific product?
A) They reason that if the import restrictions are removed, the foreign producers will raise
their prices to those of the domestic producers anyway.
B) Many countries prohibit consumers to band together to influence political actions.
C) Typically, although the added costs to consumers for a given product are high in
aggregate, they are fairly trivial for most individual consumers.
D) They reason that if they do something to hurt domestic employment for one product, the
displaced workers will then do something that will hurt their own employment.
Answer: C
Diff: 3
Skill: Concept
Objective: 2
AACSB: Analytical thinking

9) Unemployed workers are most apt to form a pressure group to support ________.
A) export restrictions
B) import restrictions
C) price limitations
D) import subsidies
Answer: B
Diff: 1
Skill: Concept
Objective: 2
AACSB: Application of knowledge
10) Successful trade retaliation is most likely achieved ________.
A) on agricultural products
B) on manufactured products
C) by a large trading country
D) by a small trading country
Answer: C
Diff: 2
Skill: Concept
Objective: 2
AACSB: Reflective thinking

11) Imports can stimulate exports by ________.


A) redistributing the work force
B) curtailing domestic competition
C) generating more tax revenue
D) increasing foreign income
Answer: D
Diff: 2
Learning Outcome: Discuss the roles of exporting, importing, and countertrade in
international business
Skill: Concept
Objective: 2
AACSB: Application of knowledge

12) All of the following are generally true about trade-displaced workers EXCEPT which
one?
A) They move abroad to take new jobs.
B) They earn less in their new jobs than they earned in their old ones.
C) They spend their unemployment benefits on living rather than retraining.
D) They have difficulty finding new work because of educational deficiencies.
Answer: A
Diff: 2
Skill: Concept
Objective: 2
AACSB: Analytical thinking
13) The rationale for the infant-industry argument for trade protection is that ________.
A) incubator centers in which business, government, and academia cooperate will develop
entrepreneurial companies
B) a country should give one firm in an industry a monopoly status so that it will grow large
enough to be competitive internationally
C) it takes time for an industry to become competitive in world markets, so protection is
needed to help this industry pass through the critical period
D) lower restrictions should be placed on products coming from countries where a
government has a large sphere of political influence
Answer: C
Diff: 2
Learning Outcome: Discuss arguments for and against government intervention in
international business
Skill: Concept
Objective: 1
AACSB: Reflective thinking

14) Which of the following statements most likely undermines the infant-industry argument?
A) Total unit costs decrease through economies of scale.
B) High tariffs to prevent foreign competition increase government revenues in the protected
country.
C) Domestic entrepreneurs need assistance to compete in industries with high entry barriers.
D) Experience of operating over time triggers higher productivity and global competitiveness.
Answer: B
Diff: 3
Learning Outcome: Discuss arguments for and against government intervention in
international business
Skill: Critical Thinking
Objective: 1
AACSB: Analytical thinking

15) A problem that can arise in using trade protectionism to develop international
competitiveness for a domestic industry is that ________.
A) it is difficult to identify industries that have a high probability of reaching competitiveness
B) assistance should be given only if entry barriers to new firms are very low
C) the protecting countries lose too much revenue from import duties
D) a short product life cycle makes the industry quickly noncompetitive
Answer: A
Diff: 3
Learning Outcome: Discuss arguments for and against government intervention in
international business
Skill: Concept
Objective: 3
AACSB: Application of knowledge
16) Which of the following is a problem with the infant-industry argument for protection?
A) Most developed countries increasingly have a larger portion of retirees than youth.
B) Consumer groups become very active in protesting the higher prices that result during
infancy.
C) Other countries retaliate by limiting exports of technology needed by the infant industry
producers.
D) If the industry does not lower costs sufficiently to be competitive, it becomes a formidable
pressure group for continued protection.
Answer: D
Diff: 3
Learning Outcome: Discuss arguments for and against government intervention in
international business
Skill: Concept
Objective: 3
AACSB: Application of knowledge

17) The industrialization argument for trade protection in developing countries is based on
the assumption that ________.
A) the protected industry will become competitive over time with economies of scale
B) unemployment and underemployment exist in rural areas, so little agricultural output is
lost as people move into industrial jobs
C) subsidizing production is a better means of protection than limiting imports
D) it is better to depend on buying agricultural surpluses from developed countries than to
produce these agricultural goods
Answer: B
Diff: 2
Learning Outcome: Compare and contrast theories of international trade
Skill: Concept
Objective: 1
AACSB: Analytical thinking

18) Unlike the infant-industry argument, the industrialization argument for trade protection
________.
A) requires independence from other countries
B) stresses labor-intensive production methods despite high costs
C) emphasizes use of locally available raw materials for manufacturing inputs
D) presumes that economic growth will occur even if domestic manufactured prices are not
globally competitive
Answer: D
Diff: 3
Learning Outcome: Compare and contrast theories of international trade
Skill: Concept
Objective: 1
AACSB: Analytical thinking
19) Developing countries have sometimes adopted policies to shift people out of agriculture
and into industry by protecting manufactured production. One of the problems they have
encountered is that ________.
A) people have been too reluctant to leave rural areas to go to the cities
B) food shortages have increased in rural areas because of worker shortages
C) demand for social and political services has increased excessively in the cities
D) developed countries have retaliated with protection of products from the developing
countries
Answer: C
Diff: 3
Learning Outcome: Discuss arguments for and against government intervention in
international business
Skill: Concept
Objective: 3
AACSB: Application of knowledge

20) Terms of trade refers to ________.


A) the quantity of imports that a given quantity of a country's exports can buy
B) specific requirements placed on imports and exports at the port of entry or exit
C) requirements agreed upon by two countries to regulate bilateral trade
D) an account statement showing a country's annual imports and exports
Answer: A
Diff: 2
Learning Outcome: Define the fundamental concepts of international business
Skill: Concept
Objective: 3
AACSB: Application of knowledge

21) Which term refers to restricting imports in order to boost domestic production and
consumption of goods that would otherwise be imported?
A) import substitution
B) terms-of-trade
C) most-favored nation
D) in-sourcing
Answer: A
Diff: 1
Learning Outcome: Compare and contrast theories of international trade
Skill: Concept
Objective: 3
AACSB: Application of knowledge
22) Export-led development refers to ________.
A) a country's efforts to promote its exports in order to reduce its trade deficits
B) a policy to promote domestic production of goods that would otherwise be imported
C) a program to promote industries with export potential
D) decreases in infrastructure development due to the loss of revenue from export tariffs
Answer: C
Diff: 2
Learning Outcome: Compare and contrast theories of international trade
Skill: Concept
Objective: 3
AACSB: Application of knowledge

23) Which of the following best explains why the experience of countries such as Taiwan and
South Korea are used to support export-led development policies?
A) their low inflation
B) their increased FDI
C) their industry diversity
D) their rapid economic growth
Answer: D
Diff: 2
Learning Outcome: Compare and contrast theories of international trade
Skill: Concept
Objective: 1
AACSB: Analytical thinking

24) The relationship between import substitution policies and export-led development
policies is best characterized by which of the following?
A) Import-substitution policies are more likely to lead to production of mature products,
whereas export-led development policies result in production of growth products.
B) The two are hard to distinguish because production under import substitution may
eventually be exported.
C) Production under import substitution policies, as opposed to export-led development
policies, is more likely to be located in urban areas.
D) Production under import substitution policies, as opposed to export-led development
policies, is likely to be more labor-intensive.
Answer: B
Diff: 3
Learning Outcome: Compare and contrast theories of international trade
Skill: Concept
Objective: 1
AACSB: Application of knowledge
25) Advocates of the comparable access argument for trade protection primarily assert that
domestic industries ________.
A) will deteriorate in countries that have lower import restrictions than their trading partners
B) should be required to implement export-led development policies on foreign competitors
C) are entitled to the same access to foreign markets as foreign industries have to their
markets
D) are forced to lower prices for domestic consumers because of foreign import restrictions
Answer: C
Diff: 2
Learning Outcome: Compare and contrast theories of international trade
Skill: Concept
Objective: 3
AACSB: Application of knowledge

26) It is sometimes contended that by imposing import controls a country might be able to
increase its exports. This contention is premised on ________.
A) the country's simultaneous currency depreciation, which decreases the price of its exports
B) using the import taxes to institute efficiency measures in potential export industries
C) raising domestic prices in one industry so that the excess domestic profits in that industry
can compensate for the cost of dumping products from another industry into foreign markets
D) getting other countries to maintain or relax their current import restrictions instead of
escalating restrictions in a trade war
Answer: D
Diff: 3
Learning Outcome: Discuss arguments for and against government intervention in
international business
Skill: Concept
Objective: 3
AACSB: Application of knowledge

27) Country X wants to eliminate its balance of trade deficit while simultaneously keeping
prices low for imported essentials. Which of the following methods would most likely
achieve these dual objectives?
A) devaluing its currency
B) enacting selective import restrictions
C) using tight monetary policies to deflate price levels
D) spurring productivity increases through general tax breaks for industry
Answer: B
Diff: 3
Learning Outcome: Discuss arguments for and against government intervention in
international business
Skill: Application
Objective: 3
AACSB: Reflective thinking
28) All of the following are reasons a country might institute import restrictions to improve
its balance of trade position with other countries EXCEPT to ________.
A) maintain essential industries
B) reduce imports and encourage exports
C) get comparable access for its companies
D) bargain away restrictions by other countries
Answer: A
Diff: 2
Learning Outcome: Discuss arguments for and against government intervention in
international business
Skill: Concept
Objective: 3
AACSB: Reflective thinking

29) Country X is withholding goods from international markets in an attempt to raise prices
abroad. Such actions will be most effective for Country X if the nation ________.
A) supports the development of substitutions
B) provides domestic industries with tax breaks
C) receives low-interest loans from foreign banks
D) holds a monopoly on the product or resource
Answer: D
Diff: 2
Skill: Application
Objective: 3
AACSB: Analytical thinking

30) Countries most likely establish export restrictions to ________.


A) ensure their population obtain the goods first
B) raise prices in foreign markets
C) encourage substitute products
D) reduce domestic production
Answer: B
Diff: 2
Skill: Concept
Objective: 3
AACSB: Application of knowledge

31) Export restrictions have a tendency to ________.


A) favor domestic consumers
B) protect employment in the export-restricted industries
C) lower prices in foreign markets
D) encourage the development of substitutes in the restricting country
Answer: A
Diff: 2
Skill: Concept
Objective: 3
AACSB: Analytical thinking

32) All of the following are potential problems of using export controls EXCEPT which one?
A) There is an incentive for other countries to develop production of their own.
B) Domestic producers may have less incentive to increase output.
C) Prices go up in the country imposing the controls.
D) There is more incentive for smuggling.
Answer: C
Diff: 2
Learning Outcome: Discuss arguments for and against government intervention in
international business
Skill: Concept
Objective: 3
AACSB: Reflective thinking

33) Exporting below cost or below the home country price is called ________.
A) countertrade
B) an export-led development policy
C) a strategic trade policy
D) dumping
Answer: D
Diff: 1
Learning Outcome: Define the fundamental concepts of international business
Skill: Concept
Objective: 3
AACSB: Application of knowledge

34) There are several reasons for a company to sell products abroad at either below cost or
below the price in the home country. Which of the following is one of these reasons?
A) encouraging foreign consumers to try new products
B) improving the exporting country's terms of trade
C) gaining imports that are sold below cost
D) following import substitution objectives
Answer: A
Diff: 2
Learning Outcome: Define the fundamental concepts of international business
Skill: Concept
Objective: 3
AACSB: Analytical thinking
35) Countries sometimes fear that foreign producers are pricing their exports artificially low.
This fear is most likely based on the assumption that ________.
A) foreign companies will lack the earnings to repay their foreign debt
B) insufficient earnings will be available to improve product technology
C) foreign producers will charge exorbitant prices after putting competitors out of business
D) developing countries will be unable to maintain critical industries needed in times of war
Answer: C
Diff: 3
Skill: Concept
Objective: 3
AACSB: Analytical thinking

36) According to the optimum tariff theory, a foreign producer will most likely ________.
A) ship highly taxed goods internationally on a per-unit basis
B) lower its export prices if the importing country imposes an import tax on its products
C) assess a tax on goods shipped internationally based on a percentage of the goods' value
D) seek import tariffs by using the comparable access argument
Answer: B
Diff: 2
Learning Outcome: Compare and contrast theories of international trade
Skill: Concept
Objective: 3
AACSB: Application of knowledge

37) An argument against limiting exports to unfriendly countries is that ________.


A) the costs of the sanctions are borne by innocent people rather than by leaders
B) markets cannot be regained after the countries become friendly
C) one country's essential product is superfluous to another
D) the exporting nation's cultural identity is harmed
Answer: A
Diff: 2
Learning Outcome: Discuss arguments for and against government intervention in
international business
Skill: Concept
Objective: 3
AACSB: Ethical understanding and reasoning
38) A possible drawback to the essential industry argument for import protectionism is
________.
A) that such protection hurts the protecting country's balance of payments
B) in times of military emergency, almost any product could be considered essential
C) other countries find supplies elsewhere
D) unemployment increases in the protecting country
Answer: B
Diff: 2
Skill: Concept
Objective: 3
AACSB: Application of knowledge

39) Defense arguments are sometimes used to prevent exports to unfriendly countries. This
runs the risk of the targeted country ________.
A) becoming politically destabilized
B) removing import restrictions
C) finding alternative sources of supply
D) banding with other countries to form a cartel
Answer: C
Diff: 2
Learning Outcome: Discuss arguments for and against government intervention in
international business
Skill: Concept
Objective: 3
AACSB: Analytical thinking

40) What is the main motive for countries' protection of their film/cinema industries?
A) to keep prices low for their citizens
B) to improve their balance of payments
C) to diversify their economies
D) to maintain their cultural sovereignty
Answer: D
Diff: 2
Skill: Concept
Objective: 3
AACSB: Reflective thinking

41) The most common type of tariff is the ________ tariff.


A) export
B) import
C) transit
D) ad valorem
Answer: B
Diff: 1
Skill: Concept
Objective: 4
AACSB: Application of knowledge

42) An import tariff may be protective ________.


A) only if it is on imports the country produces domestically
B) if it serves primarily to restrict entry of hazardous materials
C) if it does not generate significant tax revenue for essential industries
D) even though the importing country does not produce the product
Answer: D
Diff: 2
Skill: Concept
Objective: 4
AACSB: Application of knowledge

43) In addition to protection, tariffs serve to ________.


A) generate revenue
B) subsidize exports
C) subsidize imports
D) increase consumption
Answer: A
Diff: 2
Skill: Concept
Objective: 4
AACSB: Application of knowledge

44) Which term refers to a tariff or duty assessed as a percentage of an item's value?
A) specific duty
B) effective tariff
C) ad valorem duty
D) compound tariff
Answer: C
Diff: 1
Skill: Concept
Objective: 4
AACSB: Application of knowledge
45) What is the primary difficulty associated with dismantling developed countries'
agricultural subsidies?
A) Rural areas in the United States, the European Union, and Japan are disproportionately
represented in their governments.
B) Developing countries put pressure on developed countries to maintain the subsidies so that
they receive food products at a lower price.
C) The subsidies are used to counter the commodity agreements set up mainly by developing
countries.
D) The subsidies are at the consumer, rather than producer, level, and everyone benefits from
the lower prices.
Answer: A
Diff: 3
Learning Outcome: Discuss arguments for and against government intervention in
international business
Skill: Concept
Objective: 4
AACSB: Application of knowledge

46) In most cases, which type of government protection assistance is most controversial?
A) business development services
B) foreign business contacts
C) trade expositions
D) tariffs
Answer: D
Diff: 2
Learning Outcome: Discuss arguments for and against government intervention in
international business
Skill: Concept
Objective: 4
AACSB: Analytical thinking

47) Tied aid requires a recipient to ________.


A) donate a portion of the funds to the donor country's infrastructure needs
B) use the capital in any way as long as donor country approval is granted
C) employ local workers in management positions
D) spend the funds in the donor country
Answer: D
Diff: 2
Learning Outcome: Define the fundamental concepts of international business
Skill: Concept
Objective: 4
AACSB: Application of knowledge
48) Most countries have agreed on how to assess values when their customs agents levy
tariffs. Which of the following best expresses this agreement?
A) They should use the value of similar goods arriving at about the same time.
B) They should use the declared invoice price unless they doubt its authenticity.
C) They should assess a value based on local costs to produce a similar product.
D) They should assess a value based on the expected final consumer sales price.
Answer: B
Diff: 2
Skill: Concept
Objective: 4
AACSB: Reflective thinking

49) In international trade, what is a quota?


A) a guarantee by one country to buy some minimum amount from another
B) a quantitative limit on the amount of a product that can be imported or exported
C) a countertrade arrangement that establishes the value of imports and exports
D) a bilateral agreement calling for mutual access to markets
Answer: B
Diff: 1
Skill: Concept
Objective: 4
AACSB: Application of knowledge

50) A voluntary export restriction (VER) refers to ________.


A) an agreement between two countries to reciprocally restrict exports to one another
B) requests by governments for companies to limit exports of militarily useful technology
C) limiting companies' exports to increase domestic supplies
D) limits placed on exports by a government of an exporting country at the request of the
government of an importing country
Answer: D
Diff: 1
Skill: Concept
Objective: 4
AACSB: Application of knowledge

51) An import license is ________.


A) an agreement whereby one country gives another country permission to use a patent that a
company has registered there
B) a requirement that exporters take merchandise in lieu of money as payment for their sales
C) a requirement that permission be secured from governmental authorities before
importation can be undertaken
D) a government prohibition of imports from a specific country
Answer: C
Diff: 1
Skill: Concept
Objective: 4
AACSB: Application of knowledge

52) Why are offsets considered protectionist measures?


A) Exporters must often find markets for goods outside their lines of expertise.
B) Exports must be sold at a certain percentage price below the price of domestic producers.
C) Companies must submit samples to government authorities before receiving export
permission.
D) Trading companies must incur additional inventory carrying charges and pay significantly
higher tariffs.
Answer: A
Diff: 3
Skill: Concept
Objective: 4
AACSB: Analytical thinking

53) Which of the following hypothetical examples would be a restriction on the import of
services?
A) The U.S. restricts foreign companies from carrying cargo between two U.S. cities.
B) Japan restricts North Koreans from visiting Tokyo Disneyland.
C) China does not allow the importation of rice from Thailand.
D) Canada does not allow Air Canada to buy Brazilian aircraft.
Answer: A
Diff: 2
Skill: Application
Objective: 4
AACSB: Reflective thinking

54) The fact that there are few reciprocal agreements among countries on the licensing of
professionals most likely means that ________.
A) universities' study abroad programs do not enable students to obtain dual degrees and
licenses from more than one country
B) more service functions are being handled as not-for-profit
C) people immigrate to those countries with the highest standards so as to more easily
become licensed anywhere else
D) there is an effective limitation on trade in services
Answer: D
Diff: 2
Learning Outcome: Discuss the roles of exporting, importing, and countertrade in
international business
Skill: Application
Objective: 4
AACSB: Reflective thinking
55) A physician, who is a citizen of and licensed in Country A, meets the professional
licensing requirements of Country B. The physician will most likely ________.
A) have to pass a language proficiency exam before being allowed to work in Country B
B) not be allowed to work in the not-for-profit sector in Country A or Country B for a set
period
C) have to get a work permit from Country B's immigration authorities to work in Country B
D) have to work in the not-for-profit sector in Country B for a period of time before being
permitted to work for a profit-seeking organization
Answer: C
Diff: 3
Skill: Application
Objective: 4
AACSB: Reflective thinking

56) The U.S. automobile industry has attempted to counter import competition in all the
following ways EXCEPT ________.
A) concentrating on market niches that initially had less import competition
B) lobbying for customs deposits so that importers' costs would be raised
C) moving some production to lower-cost countries and exporting to the United States
D) effecting internal adjustments, such as cost efficiencies and improved quality
Answer: B
Diff: 2
Learning Outcome: Discuss the roles of exporting, importing, and countertrade in
international business
Skill: Concept
Objective: 5
AACSB: Analytical thinking

57) Companies with ________ would most likely oppose global protectionist measures.
A) internationally integrated supply chains
B) domestically focused supply chains
C) multidomestic production facilities
D) product differentiation
Answer: A
Diff: 2
Skill: Concept
Objective: 5
AACSB: Analytical thinking
58) In nearly half the cases in which U.S. firms have requested protection from imports, one
or more U.S. companies in the industry opposed the protection. What was the reason for
opposing protection?
A) They did not want consumers to have to pay higher prices that would result from
protection.
B) These were foreign-owned companies that saw the opportunity to serve the U.S. market.
C) They feared that they would lose foreign export markets because of retaliation.
D) They believed that they could compete against global and domestic rivals.
Answer: D
Diff: 3
Learning Outcome: Discuss arguments for and against government intervention in
international business
Skill: Concept
Objective: 5
AACSB: Analytical thinking

59) Which of the following is NOT causing greater complexity in the regulation of trade?
A) growth in export tariffs
B) services available over the Internet
C) heightened concern about product safety
D) development of new products that must be classified
Answer: A
Diff: 2
Learning Outcome: Discuss arguments for and against government intervention in
international business
Skill: Concept
Objective: 5
AACSB: Reflective thinking

60) The U.S. catfish industry petitioned the U.S. government for increased taxes on imported
Vietnamese fish, claiming that the fish were being sold below the cost of production. The
U.S. catfish industry was accusing the Vietnamese fish industry of ________.
A) dumping
B) using an embargo
C) subsidizing
D) using offsets
Answer: A
Diff: 2
Skill: Application
Objective: 4
AACSB: Application of knowledge
61) The U.S. catfish industry successfully petitioned the U.S. government to require that
catfish varieties imported from Vietnam be labeled as tra, basa, or pangasius. This is an
example of which of the following?
A) an embargo
B) a tariff
C) a nontariff barrier
D) a direct price influence
Answer: C
Diff: 2
Skill: Application
Objective: 4
AACSB: Application of knowledge

62) People who argue for lifting the U.S. trade embargo with Cuba claim all of the following
EXCEPT which one?
A) The embargo has not achieved its purpose of changing Cuba's economic and political
system.
B) U.S. companies lose Cuban sales to competitors from other countries.
C) Increased exposure to the United States would be a more effective force of change.
D) Cuba has largely become a market economy already.
Answer: D
Diff: 2
Learning Outcome: Discuss arguments for and against government intervention in
international business
Skill: Concept
Objective: 5
AACSB: Application of knowledge

63) People who argue for keeping the U.S. trade embargo with Cuba claim all of the
following EXCEPT which one?
A) There is not much economic potential from trade given Cuba's small population and low
per capita income.
B) Removal of the embargo will cause much more Cuban immigration to the United States.
C) If the Cuban economy is weakened just a bit more, the Cuban political-economic system
cannot be sustained.
D) There will be a backlash among countries supplying such commodities as sugar to the
United States if the U.S. buys them from Cuba instead.
Answer: B
Diff: 2
Learning Outcome: Discuss arguments for and against government intervention in
international business
Skill: Concept
Objective: 5
AACSB: Application of knowledge
64) The term protectionism, when applied to international trade, refers to governmental
restrictions and incentives to affect trade flows.
Answer: TRUE
Diff: 1
Learning Outcome: Discuss arguments for and against government intervention in
international business
Skill: Concept
Objective: 1
AACSB: Application of knowledge

65) In most cases, trade protectionism makes it easier for a company to buy what it needs and
to sell products in global markets.
Answer: FALSE
Diff: 1
Learning Outcome: Discuss arguments for and against government intervention in
international business
Skill: Concept
Objective: 1
AACSB: Application of knowledge

66) The group most likely to become involved in disputes concerning trade protectionism is
consumers.
Answer: FALSE
Diff: 2
Skill: Concept
Objective: 2
AACSB: Analytical thinking

67) Helping a struggling domestic company through import restrictions frequently causes
other countries to retaliate.
Answer: TRUE
Diff: 2
Learning Outcome: Discuss arguments for and against government intervention in
international business
Skill: Concept
Objective: 2
AACSB: Analytical thinking

68) The countries most likely to be successful at using trade retaliation are large trading
countries.
Answer: TRUE
Diff: 2
Skill: Concept
Objective: 2
AACSB: Application of knowledge
69) On average, workers displaced by imports earn higher wages in the new jobs they accept.
Answer: FALSE
Diff: 2
Learning Outcome: Discuss arguments for and against government intervention in
international business
Skill: Concept
Objective: 3
AACSB: Analytical thinking

70) The infant-industry argument for trade protection holds that an industry needs
government protection from imports until it becomes competitive enough in world markets.
Answer: TRUE
Diff: 1
Learning Outcome: Discuss arguments for and against government intervention in
international business
Skill: Concept
Objective: 1
AACSB: Application of knowledge

71) Infant-industry protection requires some segment of the economy to incur the higher
costs when local production is inefficient.
Answer: TRUE
Diff: 2
Learning Outcome: Discuss arguments for and against government intervention in
international business
Skill: Concept
Objective: 1
AACSB: Application of knowledge

72) The argument for using protectionism to bring about industrialization in developing
countries presumes that gains will occur because the industry will become internationally
competitive.
Answer: FALSE
Diff: 2
Learning Outcome: Discuss arguments for and against government intervention in
international business
Skill: Concept
Objective: 1
AACSB: Reflective thinking

73) Export prices of primary products fluctuate less than export prices of manufactured
products.
Answer: FALSE
Diff: 2
Learning Outcome: Discuss arguments for and against government intervention in
international business
Skill: Concept
Objective: 3
AACSB: Application of knowledge
74) Import substitution is a program promoting local production of products that would
otherwise be imported.
Answer: TRUE
Diff: 1
Skill: Concept
Objective: 1
AACSB: Application of knowledge

75) Export-led development refers to the off-shoring of production.


Answer: FALSE
Diff: 1
Skill: Concept
Objective: 1
AACSB: Application of knowledge

76) The argument for using import controls to promote exports is partially premised on the
assumption that other countries will remove their import restrictions as a result.
Answer: TRUE
Diff: 2
Skill: Concept
Objective: 3
AACSB: Analytical thinking

77) The comparable access argument for import restrictions is a more valid economic
argument for products using small-scale technology than for products requiring substantial
economies of scale to be competitive.
Answer: FALSE
Diff: 2
Skill: Concept
Objective: 3
AACSB: Reflective thinking

78) Countries typically establish export restrictions to encourage the development of


substitute products.
Answer: FALSE
Diff: 2
Learning Outcome: Discuss arguments for and against government intervention in
international business
Skill: Concept
Objective: 3
79) Export controls are highly effective for digital products, such as computers, TVs, and
cameras.
Answer: FALSE
Diff: 2
Learning Outcome: Discuss arguments for and against government intervention in
international business
Skill: Concept
Objective: 3
AACSB: Application of knowledge

80) The lowering of a foreign producer's price as a result of an imposed import tax is known
as an optimum tariff.
Answer: TRUE
Diff: 1
Skill: Concept
Objective: 3
AACSB: Application of knowledge

81) Home country consumers are typically active in preventing their domestic companies
from dumping products into foreign markets.
Answer: FALSE
Diff: 2
Learning Outcome: Discuss arguments for and against government intervention in
international business
Skill: Synthesis
Objective: 2, 3
AACSB: Analytical thinking

82) The essential-industry argument holds that industries with potential export capabilities
should be protected.
Answer: FALSE
Diff: 1
Learning Outcome: Discuss arguments for and against government intervention in
international business
Skill: Concept
Objective: 3
AACSB: Application of knowledge

83) Import trade controls, but not export trade controls, can be used as a weapon of foreign
policy.
Answer: FALSE
Diff: 2
Learning Outcome: Discuss arguments for and against government intervention in
international business
Skill: Concept
Objective: 4
AACSB: Application of knowledge

84) The most common type of tariff is the export tariff.


Answer: FALSE
Diff: 1
Skill: Concept
Objective: 4
AACSB: Application of knowledge

85) An effective tariff is the sum of the ad valorem tariff plus the specific duty.
Answer: FALSE
Diff: 1
Skill: Concept
Objective: 4
AACSB: Analytical thinking

86) Agricultural subsidies by developed countries impede the competitiveness of agricultural


exports by developing countries.
Answer: TRUE
Diff: 2
Learning Outcome: Discuss arguments for and against government intervention in
international business
Skill: Concept
Objective: 4
AACSB: Application of knowledge

87) When customs officials set a value on which to place an import tariff, they ordinarily use
the declared invoice price unless they doubt its authenticity.
Answer: TRUE
Diff: 2
Skill: Concept
Objective: 4
AACSB: Application of knowledge

88) A quota is a quantitative limit on the amount of a product that can be traded.
Answer: TRUE
Diff: 1
Skill: Concept
Objective: 4
AACSB: Application of knowledge

89) The purpose of "Made in" labels on imported products is to enable countries to keep
records of the origin of imports.
Answer: FALSE
Diff: 2
Skill: Concept
Objective: 4
AACSB: Analytical thinking

90) Governments sometimes prohibit operations of private companies, foreign or domestic, in


some sectors because they feel these services should not be sold at a profit.
Answer: TRUE
Diff: 2
Skill: Concept
Objective: 4
AACSB: Application of knowledge

91) At present there is little reciprocal recognition of professional licensing among countries.
Answer: TRUE
Diff: 2
Skill: Concept
Objective: 4
AACSB: Application of knowledge

92) Companies that have integrated their supply chains internationally tend to lobby their
home governments for increased protectionist measures.
Answer: FALSE
Diff: 2
Skill: Concept
Objective: 5
AACSB: Application of knowledge

93) When a company is seeking protection from imports, it can usually improve its chances
of getting that protection if it allies with most of the companies in the industry.
Answer: TRUE
Diff: 1
Skill: Concept
Objective: 5
AACSB: Analytical thinking

94) The international regulatory situation for trade is becoming more, rather than less,
complex.
Answer: TRUE
Diff: 1
Skill: Concept
Objective: 5
AACSB: Application of knowledge

95) Every time countries enter a new trading agreement, service trade tends to grow more
rapidly than merchandise trade.
Answer: FALSE
Diff: 1
Skill: Concept
Objective: 5
AACSB: Reflective thinking

96) What are the disadvantages of import restrictions in regards to creating domestic
employment opportunities?
Answer: One problem with restricting imports in order to create jobs is that other countries
might retaliate with their own restrictions. New import restrictions by a major country have
usually brought quick retaliation, sometimes causing more job losses than gains in industries
protected by the new restrictions. Even if no country retaliates, the restricting country will
gain jobs one place and lose them somewhere else, such as in import-handling jobs. Imports
may also help create jobs in other industries, and these industries may form pressure groups
against protectionism.
Diff: 3
Skill: Critical Thinking
Objective: 3
AACSB: Analytical thinking

97) Explain the rationale for and problems with making the infant-industry argument work as
intended.
Answer: The infant-industry argument holds that a government should guarantee an
emerging industry a large share of the domestic market until it becomes efficient enough to
compete against imports. Developing countries still use this argument to support their
protectionist policies. The infant-industry argument is based on the logic that although the
initial output costs for an industry in a given country may be so high as to make it
noncompetitive in world markets; over time the costs will decrease to a level sufficient to
achieve efficient production. The cost reductions may occur for two reasons: As companies
gain economies of scale and employees become more efficient through experience, total unit
costs drop to competitive levels.

Although it is reasonable to expect costs to decrease over time, they may not go down
enough, which poses two problems for protecting an industry. First, governments have
difficulty identifying those industries that have a high probability of success. If infant-
industry protection goes to an industry that does not reduce costs enough to make it
competitive against imports, chances are its owners, workers, and suppliers will constitute a
formidable pressure group that may prevent the importation of a cheaper competitive product.
Second, even if policymakers can ascertain which industries are likely to succeed, it does not
necessarily mean that companies in those industries should receive governmental assistance.
Entrepreneurs may incur the costs and reap the benefits instead. For the infant-industry
argument to be fully viable, future benefits should exceed early costs.
Diff: 3
Learning Outcome: Discuss arguments for and against government intervention in
international business
Skill: Synthesis
Objective: 1, 3
AACSB: Analytical thinking
98) Why do developing countries sometimes impose import restrictions to increase their
levels of industrialization?
Answer: Countries with a large manufacturing base generally have higher per capita incomes
than do countries without such a base. Moreover, a number of countries, such as the United
States and Japan, developed an industrial base while largely preventing competition from
foreign-based production. Many developing countries use protection to increase their level of
industrialization because of industrial countries' economic success and experience.
Specifically, they believe:
a. Surplus workers can more easily increase manufacturing output than they can increase
agricultural output.
b. Inflows of foreign investment in the industrial area will promote growth.
c. Prices and sales of traditional agricultural products and raw materials fluctuate too much,
harming economies that depend on too few of them.
d. Markets and prices for industrial products will grow faster than those for agricultural
products.
Diff: 3
Learning Outcome: Discuss arguments for and against government intervention in
international business
Skill: Critical Thinking
Objective: 1
AACSB: Analytical thinking

99) What is the difference between import substitution policies and export-led development
policies? What are the potential effects of each?
Answer: Developing countries promote industrialization by restricting imports in order to
encourage local production for local consumption goods which they formerly imported. This
is known as import substitution. If the protected industries do not become efficient,
consumers may have to support them by paying higher prices or higher taxes. In contrast to
import substitution, some countries have achieved rapid economic growth by promoting
export industries, an approach known as export-led development. These countries try to
develop industries for which export markets should logically exist. Industrialization may
result initially in import substitution, yet export-led development of the same products may
be feasible later.
Diff: 3
Learning Outcome: Discuss arguments for and against government intervention in
international business
Skill: Synthesis
Objective: 1, 3
AACSB: Analytical thinking
100) Many companies and industries argue that they should have the same access to foreign
markets as foreign industries and companies have to their markets. In a short essay, discuss
this issue of "comparable access," or "fairness."
Answer: From an economic standpoint, comparable access argues that in industries in which
increased production will greatly decrease cost, either from scale economies or learning
effects, producers that lack equal access to a competitor's market will have a disadvantage in
gaining enough sales to be cost-competitive. The argument for equal access also is presented
as one of fairness. There are at least two arguments against this fairness doctrine. First, there
are advantages of freer trade, even if imposed unilaterally. Restrictions may deny one's own
consumers lower prices. Second, governments would find it cumbersome and expensive to
negotiate separate agreements for each of the many thousands of different products and
services that might be traded.
Diff: 3
Learning Outcome: Discuss arguments for and against government intervention in
international business
Skill: Synthesis
Objective: 1, 3
AACSB: Analytical thinking

101) What are common reasons that governments enact export restrictions? What are the
possible negative consequences of such restrictions?
Answer: A country may limit exports of a product that is in short supply worldwide in order
to favor domestic consumers. Typically, greater supply drops local prices beneath those in the
intentionally undersupplied world markets. However, this discourages domestic producers
from increasing output and encourages them to smuggle output to sell abroad. It also
encourages foreign producers to develop substitutes or production of their own. Countries
also fear that foreign producers will price their exports so artificially low that they drive
domestic producers out of business, after which they charge monopoly prices. However,
competition among foreign producers limits their ability to charge exorbitant prices. The
ability to price low abroad may result from high domestic prices due to a lack of competition
at home or from home country governmental subsidies.
Diff: 3
Learning Outcome: Discuss arguments for and against government intervention in
international business
Skill: Synthesis
Objective: 1, 3
AACSB: Analytical thinking
102) What is dumping? What are the possible effects of dumping on a country's economy?
Answer: When companies export below cost or below their home country price, this is called
dumping. Most countries prohibit imports of dumped products, but enforcement usually
occurs only if the imported product disrupts domestic production. If there is no domestic
production, then the only host country effect is a low price to its consumers. Companies may
dump because they cannot otherwise build a market abroad. They can afford to dump if the
competitive landscape allows them to charge high domestic prices or if their home country
government subsidizes them. They may also incur short-term losses abroad if they believe
they can recoup those losses after eliminating competitors in the market. Home country
consumers or taxpayers seldom realize that they are, in effect, paying so that foreign
consumers have low prices. A company believing it is competing against dumped products
may ask its government to restrict the imports.
Diff: 3
Skill: Application
Objective: 3
AACSB: Analytical thinking
103) Briefly discuss the four noneconomic rationales for governmental intervention in the
free movement of trade: maintaining essential industries, preventing shipments to unfriendly
countries, maintaining or extending spheres of influence, and preserving national identity.
Answer:
a. Maintenance of essential industries (especially defense): A major consideration behind
governmental action on trade is the protection of essential domestic industries during
peacetime so that a country is not dependent on foreign sources of supply during war. This is
called the essential-industry argument. This argument for protection has much appeal in
rallying support for import barriers. However, in times of real crisis or military emergency,
almost any product could be essential. Because of the high cost of protecting an inefficient
industry or a higher-cost domestic substitute, the essential-industry argument should not be
accepted without a careful evaluation of costs, real needs, and alternatives. Once an industry
becomes protected, that protection is difficult to terminate because protected companies and
their employees support politicians who will support their protection from imports.
b. Prevention of shipments to unfriendly countries: Groups concerned about security often
use defense arguments to prevent exports, even to friendly countries, of strategic goods that
might fall into the hands of potential enemies or that might be in short supply domestically.
Export constraints may be valid if the exporting country assumes there will be no retaliation
that prevents it from securing even more essential goods from the potential importing
country. Trade controls on nondefense goods also may be used as a weapon of foreign policy
to try to prevent another country from easily meeting its economic and political objectives.
c. Maintenance or extension of spheres of influence: Governments frequently give aid and
credits to, and encourage imports from, countries that join a political alliance or vote a certain
way within international bodies. A country's trade restrictions may also coerce governments
to follow certain political actions or punish companies whose governments do not follow the
actions.
d. Conservation of activities that help preserve a national identity: Countries are held
together partially through a common sense of identity that sets their citizens apart from other
nationalities. To protect this "separateness," countries limit foreign products and services in
certain sectors, particularly the media.
Diff: 2
Learning Outcome: Discuss arguments for and against government intervention in
international business
Skill: Critical Thinking
Objective: 1
AACSB: Diverse and multicultural work environments
104) Describe and compare the different types of tariffs (duties).
Answer: A tariff, or duty, the most common type of trade control, is a tax that a government
levies on a good shipped internationally. If collected by the exporting country, it is known as
an export tariff; if collected by a country through which the goods have passed, it is a transit
tariff; if collected by the importing country, it is an import tariff. The import tariff is by far
the most common. Import tariffs primarily serve as a means of raising the price of imported
goods so that domestically produced goods will gain a relative price advantage.

A tariff may be protective even though there is no domestic production in direct competition.
Tariffs also serve as a source of governmental revenue. Import tariffs are of little importance
to large industrial countries, but are a major source of revenue in many developing countries.
Transit tariffs were once a major source of revenue for countries, but they have been nearly
abolished through governmental treaties. A government may assess a tariff on a per-unit
basis, in which case it is a specific duty. It may assess a tariff as a percentage of the value of
the item, in which case it is an ad valorem duty. If it assesses both specific and an ad valorem
duty on the same product, the combination is a compound duty. A specific duty is easy for
customs officials who collect duties to assess because they do not need to determine a good's
value on which to calculate a percentage tax. Because an ad valorem tariff is based on the
total value of the product, meaning the raw materials and the processing combined,
developing countries argue that the effective tariff on the manufactured portion turns out to
be higher than the published tariff rate.
Diff: 3
Skill: Application
Objective: 4
AACSB: Analytical thinking

105) In a short essay, list and discuss the nontariff barriers that relate to direct price
influences: subsidies, aid and loans, customs valuations, and other direct price influences.
Answer:
a. Subsidies: Countries sometimes make direct payments (called subsidies) to domestic
companies to reduce their costs or compensate them for losses incurred from selling abroad.
b. Aid and loans: Governments also give aid and loans to other countries. If the recipient is
required to spend the funds in the donor country, some products can compete abroad that
might otherwise be noncompetitive.
c. Customs valuation: Most countries have agreed on a procedure for assessing values when
their customs agents levy tariffs, but customs must ascertain whether the invoice correctly
identifies the product, its price, and its origin.
d. Other direct price influences: Countries frequently use other means to affect prices,
including special fees, requirements that customs deposits be placed in advance of shipment,
and minimum price levels at which goods can be sold after they have customs clearance.
Diff: 3
Skill: Application
Objective: 4
AACSB: Analytical thinking
106) List and define the types of nontariff barriers that limit the quantity of goods traded:
quotas, embargoes, buy local legislation, standards and labels, specific permission
requirements, administrative delays, and reciprocal requirements.
Answer:
a. Quotas: The most common type of import or export restriction based on quantity is the
quota. From the standpoint of imports, a quota most frequently limits the quantity of a
product allowed to be imported in a given year. The amount frequently reflects a guarantee
that domestic producers will have access to a certain percentage of the domestic market in
that year.
b. Embargoes: An embargo is a specific type of quota that prohibits all trade on a whole
category of products or on all products from a given country. Governments use embargoes in
an attempt to use economic means to achieve political goals.
c. "Buy Local" legislation: Another form of quantitative trade control is "buy local"
legislation. If government purchases are a large part of total expenditures within a country,
they comprise an important part of the market. Most governments favor domestic producers
in their purchases of goods. Sometimes they specify a content restriction—in which a certain
percentage of the product is of local origin.
d. Standards and labels: Countries commonly have set classification, labeling, and testing
standards in a manner that allows the sales of domestic products but inhibits that of foreign-
made ones. The purpose of testing standards is to protect the safety or health of the domestic
population. However, there have been situations where exporters have argued that such
restrictions protect domestic producers instead.
e. Specific permission requirements: Some countries require that potential importers or
exporters secure permission from governmental authorities before conducting trade
transactions, a requirement known as an import license.
f. Administrative delays: Closely related to specific permission requirements are intentional
administrative delays, which create uncertainty and raise the cost of carrying inventory.
g. Reciprocal requirements: Governments sometimes require that exporters take
merchandise in lieu of money or that they promise to buy merchandise or services in the
country to which they export. This requirement is common in the aerospace and defense
industries—sometimes because the importer is short of foreign currency to purchase what it
wants, and sometimes because the sales are so large the buyer has strong negotiating power.
Diff: 3
Skill: Application
Objective: 4
AACSB: Analytical thinking
107) What are the main arguments for limiting trade in services? What is your opinion on
limiting trade in services?
Answer: Countries restrict trade in services for three reasons:
a. Essentiality: Countries judge certain service industries to be essential because they serve
strategic purposes or because they provide social assistance to their citizens. They sometimes
prohibit private companies, foreign or domestic, in some sectors because they feel the
services should not be sold for profit.
b. Standards: Governments limit foreign entry into many service professions to ensure
practice by qualified personnel. The licensing standards of these personnel vary by country.
At present, there is little reciprocal recognition in licensing from one country to another
because occupational standards and requirements differ substantially.
c. Immigration: Satisfying the standards of a particular country does not guarantee that a
foreigner can then work there. Governmental regulations often require that an organization—
domestic or foreign—search extensively for qualified personnel locally before it can even
apply for work permits for personnel it would like to bring in from abroad.
Diff: 3
Skill: Critical Thinking
Objective: 4
AACSB: Analytical thinking
International Business: Environments and Operations, 15e (Daniels et al.)
Chapter 7 Cross-National Cooperation and Agreements

1) ________ integration is the political and economic agreements among countries that give
preference to member countries to the agreement.
A) Global
B) Economic
C) Bilateral
D) Regional
Answer: B
Diff: 1
Learning Outcome: Define the fundamental concepts of international business
Skill: Concept
Objective: 1

2) Country A and Country B make an agreement to cooperate more closely and implement
tariff reductions. Which of the following best describes this agreement?
A) double commodity integration
B) multilateral integration
C) regional integration
D) bilateral integration
Answer: D
Diff: 1
Learning Outcome: Define the fundamental concepts of international business
Skill: Application
Objective: 1

3) The European Union is an example of ________ integration.


A) regional
B) relative
C) global
D) bilateral
Answer: A
Diff: 2
Skill: Concept
Objective: 1
AACSB: Diverse and multicultural work environments
4) Global integration occurs as countries from all over the world decide to cooperate through
the ________.
A) EU
B) NAFTA agreement
C) WTO
D) CARICOM
Answer: C
Diff: 1
Skill: Concept
Objective: 1
AACSB: Diverse and multicultural work environments

5) Which of the following statements most accurately describes the relationship between
trading groups and MNEs?
A) Trading groups have no influence on the size of the regional market.
B) Regional trading groups can define the rules under which companies must operate within
that region.
C) Companies never need to change their organizational structure to take advantage of
regional trading groups.
D) Regional trading groups have no influence on an MNE's strategy since they operate
worldwide.
Answer: B
Diff: 2
Skill: Concept
Objective: 1

6) Anderson Enterprises is a U.S. firm that manufactures light fixtures for commercial and
residential consumers. Anderson is looking to expand internationally, so the firm should most
likely ________.
A) focus on developing new recruitment strategies and selection tests
B) change their organizational structure to fit the norms of the foreign country
C) disregard changes in trading groups because they have little effect on a company
D) look to expand beyond the triad regions currently controlled by the top 500 MNEs
Answer: B
Diff: 2
Learning Outcome: Discuss factors affecting the organizational structures of international
businesses
Skill: Application
Objective: 1
7) Baldani Manufacturing, an Italian firm, plans to expand into Asia. In order to take
advantage of regional trading groups, Baldani will most likely need to change its ________.
A) operating strategies
B) selection test methods
C) bilateral trade agreements
D) economic integration methods
Answer: A
Diff: 2
Skill: Application
Objective: 1

8) Which of the following is NOT one of the regions included in the triad regions of the
world?
A) South America
B) Europe
C) North America
D) Asia
Answer: A
Diff: 2
Learning Outcome: Define the fundamental concepts of international business
Skill: Concept
Objective: 1
AACSB: Dynamics of the global economy

9) The ________ clause embodied the fundamental principle of GATT—trade without


discrimination.
A) most-favored-nation
B) nontariff barriers
C) free rider
D) normal trade relations
Answer: A
Diff: 1
Skill: Concept
Objective: 1

10) Which of the following accurately identifies a difference between GATT and the WTO?
A) GATT could enforce member compliance with agreements, but the WTO cannot.
B) At its inception, GATT had more member nations than the WTO currently has.
C) GATT withdrew the most-favored-nation clause, but the WTO reinstated it.
D) GATT rules did not cover trade in services, but the rules of the WTO do.
Answer: D
Diff: 2
Skill: Concept
Objective: 1
11) Under the WTO agreement, ________.
A) a dispute resolution mechanism allows countries to bring grievances to the WTO against
countries that levy inappropriate trade discrimination measures
B) there is no dispute resolution mechanism except for trade involving environmental
products
C) countries are allowed to place trade barriers on member countries with no particular
justification, because like GATT the WTO has no enforcement mechanism
D) tariffs are permitted to be levied by developed countries against developing countries but
not against each other
Answer: A
Diff: 2
Skill: Concept
Objective: 1
AACSB: Ethical understanding and reasoning

12) A major problem with the Doha Round is that ________.


A) developing countries want developed countries to better protect their intellectual property
B) developing countries want a reduction in agricultural subsidies maintained by the
developed countries
C) the WTO does not want the developing countries to liberalize their investment rules
D) because of security issues, not all countries were able to attend the meetings, so it was
impossible to get a consensus vote
Answer: B
Diff: 2
Learning Outcome: Discuss arguments for and against regional economic integration
Skill: Concept
Objective: 1

13) Most trade groups contain countries in the same area of the world. Why is this so?
A) The distances that goods need to travel between such countries are short.
B) Distribution channels are not easily established in adjacent countries.
C) Adjacent countries are reluctant to coordinate policies.
D) Neighboring countries usually lack a common history and interests.
Answer: A
Diff: 3
Learning Outcome: Discuss arguments for and against regional economic integration
Skill: Concept
Objective: 4
AACSB: Dynamics of the global economy
14) Which of the following groups of countries is most likely to form a regional trading
group?
A) India, Argentina, France
B) Canada, United States, Mexico
C) Brazil, Ukraine, Japan
D) Spain, South Africa, Mongolia
Answer: B
Diff: 2
Skill: Application
Objective: 4
AACSB: Dynamics of the global economy

15) The 27 member EU negotiates trade agreements as one. The EU and Brazil have
negotiated a strategic trade alliance which is an example of a ________.
A) customs union
B) global free trade agreement
C) bilateral agreement
D) multilateral agreement
Answer: C
Diff: 2
Learning Outcome: Discuss arguments for and against regional economic integration
Skill: Application
Objective: 2
AACSB: Dynamics of the global economy

16) The goal of a ________ is to abolish all tariffs among member countries.
A) customs union
B) common market
C) free trade agreement
D) common internal tariff
Answer: C
Diff: 1
Learning Outcome: Discuss arguments for and against regional economic integration
Skill: Concept
Objective: 4

17) Of the following possibilities, the European Union is best described as a ________.
A) customs union
B) domestic organization
C) global bargaining unit
D) common language agreement
Answer: A
Diff: 2
Learning Outcome: Discuss arguments for and against regional economic integration
Skill: Concept
Objective: 4
AACSB: Diverse and multicultural work environments

18) A ________ results when free mobility of factors of production is added to a customs
union.
A) customs union
B) common market
C) free trade agreement
D) regional trade agreement
Answer: B
Diff: 1
Learning Outcome: Discuss arguments for and against regional economic integration
Skill: Concept
Objective: 4

19) Members of the Andean Community (CAN) have a common external tariff. CAN is most
likely a ________.
A) free trade agreement
B) customs union
C) domestic trade zone
D) free trade sovereignty
Answer: B
Diff: 1
Learning Outcome: Discuss arguments for and against regional economic integration
Skill: Application
Objective: 4

20) ________ effects of trade agreements are the shifting of resources from inefficient to
efficient companies as trade barriers fall.
A) Dynamic
B) Static
C) Economic
D) Barrier
Answer: B
Diff: 1
Learning Outcome: Discuss arguments for and against regional economic integration
Skill: Concept
Objective: 3

21) ________ effects of economic integration are the overall growth in the market and the
impact on a company caused by expanding production and by the company's ability to
achieve greater economies of scale.
A) Dynamic
B) Static
C) Economic
D) Barrier
Answer: A
Diff: 1
Skill: Concept
Objective: 3

22) Trade shifting to countries within a regional trade agreement at the expense of trade with
countries not in the agreement is called ________.
A) a dynamic effect
B) trade creation
C) trade diversion
D) economy of scale
Answer: C
Diff: 2
Skill: Concept
Objective: 3

23) When the European Union was formed, the size of the market increased for European
companies. This is most likely an example of a ________.
A) dynamic effect
B) static effect
C) trade diversion
D) trade reflection
Answer: A
Diff: 2
Learning Outcome: Discuss arguments for and against regional economic integration
Skill: Application
Objective: 3
AACSB: Dynamics of the global economy

24) Since production has shifted to more efficient producers due to comparative advantage,
consumers in Country X have had access to more goods at lower prices. Which of the
following most likely exists?
A) trade specialization
B) trade diversion
C) trade creation
D) trade internalization
Answer: C
Diff: 2
Learning Outcome: Discuss arguments for and against regional economic integration
Skill: Application
Objective: 3

25) A free trade agreement is likely to increase efficiency because of ________.


A) decreased competition
B) increased competition
C) decreased trade diversion
D) decreased trade creation
Answer: B
Diff: 2
Learning Outcome: Discuss arguments for and against regional economic integration
Skill: Concept
Objective: 3

26) Assume that U.S. companies are importing the same product from Mexico and Taiwan.
The United States enters into an FTA with Mexico but not with Taiwan. Consequently, the
United States begins to import more goods from Mexico (due to lower tariffs) than from
Taiwan, even though the Mexican products are not any better or cheaper. This is most likely
an example of ________.
A) trade specialization
B) trade internalization
C) trade creation
D) trade diversion
Answer: D
Diff: 2
Learning Outcome: Discuss arguments for and against regional economic integration
Skill: Application
Objective: 3
AACSB: Dynamics of the global economy

27) Because the size of the market increases when trade barriers fall, companies can increase
their production, which will result in lower costs per unit. This phenomenon is known as
________.
A) trade creation
B) economies of scale
C) diseconomies of scale
D) increased competition
Answer: B
Diff: 1
Skill: Concept
Objective: 3

28) The ________ is the European Union's ultimate decision-making body and is composed
of the different ministers of the member countries.
A) European Commission
B) Council of the European Union
C) European Parliament
D) European Court of Justice
Answer: B
Diff: 2
Skill: Concept
Objective: 5
29) The three major responsibilities of the ________ are legislative power, control over the
budget, and supervision of executive decisions.
A) European Commission
B) European Council
C) European Parliament
D) European Court of Justice
Answer: C
Diff: 2
Skill: Concept
Objective: 5

30) The EU organization that provides political leadership, drafts laws, and runs the daily
programs of the EU is the ________.
A) European Commission
B) Council of Ministers
C) European Parliament
D) European Central Bureaucracy
Answer: A
Diff: 2
Skill: Concept
Objective: 5

31) The EU organization that ensures consistent interpretation and application of EU treaties
is the ________.
A) European Commission
B) Council of Ministers
C) Court of Justice
D) Council of Treaties and Laws
Answer: C
Diff: 2
Skill: Concept
Objective: 5
AACSB: Ethical understanding and reasoning

32) Which of the following was primarily responsible for establishing the euro?
A) Lisbon Treaty
B) Treaty of Maastricht
C) European Finance Act
D) Single European Act
Answer: B
Diff: 2
Skill: Concept
Objective: 5
33) Critics of the Lisbon Treaty primarily argue that the legislation will ________.
A) reduce national sovereignty
B) implement trade restrictions
C) develop a monetary union
D) weaken decision making
Answer: A
Diff: 2
Learning Outcome: Discuss arguments for and against regional economic integration
Skill: Application
Objective: 4

34) The ________ is the common currency of the European Union.


A) franc
B) pound
C) euro
D) mark
Answer: C
Diff: 1
Skill: Concept
Objective: 5

35) Which of the following statements about the euro is true?


A) It was adopted by all existing EU members when it was first initiated.
B) It must be adopted by countries as a precondition to joining the EU.
C) It cannot be used by countries that are not members of the EU.
D) It was designed to eliminate currency as a barrier to trade in the EU.
Answer: D
Diff: 3
Learning Outcome: Discuss arguments for and against regional economic integration
Skill: Concept
Objective: 5

36) Which of the following members of the European Union has NOT adopted the euro?
A) United Kingdom
B) Estonia
C) Germany
D) Greece
Answer: A
Diff: 2
Skill: Concept
Objective: 5
37) Hudson Manufacturing is an MNE based in the U.S. with operations in Asia. The firm is
considering expansion into the European Union. Which of the following questions is most
relevant to the decision?
A) What is the primary language of most workers?
B) Which country has the best production location?
C) Which currency has the most favorable value of the euro?
D) Which country has the lowest tariffs for manufactured products?
Answer: B
Diff: 2
Learning Outcome: Discuss arguments for and against regional economic integration
Skill: Critical Thinking
Objective: 5
AACSB: Reflective thinking

38) Hudson Manufacturing is an MNE based in the U.S. with operations in Asia. The firm is
considering expansion into the European Union. Executives at the firm are debating whether
central Europe or Eastern Europe would be best for the firm. Which of the following best
supports a decision to establish operations in Eastern Europe?
A) Hudson wants to implement a high-performance work system.
B) Hudson plans to staff the foreign facility with local managers.
C) Hudson wants to minimize costs by keeping wages low.
D) Hudson recently lost money in a joint venture.
Answer: C
Diff: 3
Learning Outcome: Discuss arguments for and against regional economic integration
Skill: Critical Thinking
Objective: 5

39) Korman Industries is a foreign multinational that recently established operations in the
European Union. What is the most likely advantage for Korman as a result?
A) Governance processes are streamlined because local governments have been eliminated.
B) Market size is larger because of the elimination of internal tariff barriers.
C) Differential external tariff barriers exist for product shipments.
D) The EU uses English as its official language.
Answer: B
Diff: 2
Learning Outcome: Discuss arguments for and against regional economic integration
Skill: Application
Objective: 5
40) Which of the following recently threatened the future of the EU's common currency?
A) refusal of the UK to use the euro
B) failed mergers and acquisitions
C) debt crisis in Greece
D) human rights issues
Answer: C
Diff: 2
Learning Outcome: Discuss arguments for and against regional economic integration
Skill: Concept
Objective: 5
AACSB: Dynamics of the global economy

41) Which of the following are members of NAFTA?


A) the United States, Canada, and Mexico
B) North America and Latin America
C) the United Kingdom, the United States, and Canada
D) the United States, Canada, and Brazil
Answer: A
Diff: 1
Skill: Concept
Objective: 5
AACSB: Diverse and multicultural work environments

42) NAFTA was primarily formed because the member nations have ________.
A) static and dynamic liberalization policies
B) similarly sized economies and resources
C) geographic proximity to each other
D) competing interests with the EU
Answer: C
Diff: 2
Learning Outcome: Discuss arguments for and against regional economic integration
Skill: Concept
Objective: 5

43) NAFTA is a good example of ________.


A) trade divestment
B) trade diversion
C) the theory of trade disruption
D) the theory of trade implementation
Answer: B
Diff: 2
Skill: Concept
Objective: 5
44) Compared to the European Union, the North American Free Trade Agreement ________.
A) has a stronger currency linkage due to the U.S. dollar
B) has more trade problems since it is a customs union
C) is significantly larger in population and total GNI
D) is slightly smaller in population and GDP
Answer: D
Diff: 2
Skill: Concept
Objective: 5
AACSB: Dynamics of the global economy

45) Under NAFTA, rules of origin ensure that ________.


A) only goods produced mostly within the region are eligible for liberal tariff conditions
B) only American products are shipped to Canada and Mexico duty-free
C) all members import products only from member nations
D) all members have the same external and internal tariffs
Answer: A
Diff: 2
Skill: Concept
Objective: 5

46) According to regional content rules, at least ________ of the net cost of most products
must come from the NAFTA region in order to get access to the tariff reductions of NAFTA.
A) 62.5%
B) 50%
C) 45%
D) 40.5%
Answer: B
Diff: 2
Skill: Concept
Objective: 5

47) Which of the following is a unique provision of NAFTA?


A) tariff elimination
B) common currency
C) environmental standards
D) immigration oversights and policies
Answer: C
Diff: 2
Learning Outcome: Discuss arguments for and against regional economic integration
Skill: Concept
Objective: 5
48) The major trade group in South America involving Brazil, Paraguay, Uruguay, and
Argentina is ________.
A) MERCOSUR
B) UNASUR
C) APEC
D) CACM
Answer: A
Diff: 1
Skill: Concept
Objective: 5

49) The ________ is a preferential trade agreement that was organized in 1967 and comprises
Brunei, Cambodia, Indonesia, Laos, Malaysia, Myanmar, the Philippines, Singapore,
Thailand, and Vietnam.
A) APEC
B) OAU
C) ASEAN
D) CAFTA
Answer: C
Diff: 1
Skill: Concept
Objective: 5

50) Which of the following is the primary focus of the African Union (AU)?
A) FDI influx
B) democracy
C) economic integration
D) intrazonal trade relations
Answer: B
Diff: 2
Learning Outcome: Discuss arguments for and against regional economic integration
Skill: Concept
Objective: 5

51) An NGO is ________.


A) an agency of the United Nations
B) a private institution independent of a government
C) any organization working on environmental issues
D) an organization that is concerned only with workers' rights
Answer: B
Diff: 1
Skill: Concept
Objective: 6
52) Which of the following is most likely a true statement about the relationship between the
United Nations and NGOs?
A) The UN must license an NGO for it to be involved in any international relief efforts.
B) A UN committee discusses issues of importance to NGOs but does not regulate their
activities.
C) The UN determines which NGOs can work on international humanitarian issues and relief
efforts.
D) NGOs typically operate in only one country, whereas UN agencies can operate anywhere
in the world.
Answer: B
Diff: 3
Skill: Concept
Objective: 6

53) Many NGOs, including Africa Now, Quaker Peace and Social Witness, and Save the
Children, are members of the ________.
A) Ethical Trading Initiative
B) International Red Cross Council
C) United Nations Global Compact
D) United Nations Economic and Social Council
Answer: A
Diff: 1
Skill: Concept
Objective: 6

54) Commodity agreements ________.


A) were initially established to attempt to stabilize commodity prices
B) are effective in regulating the price of grains but not minerals
C) are effective in regulating the prices of both grains and minerals
D) were disbanded by the United Nations for being non-competitive
Answer: A
Diff: 2
Skill: Concept
Objective: 6

55) The role of most commodity agreements now is to ________.


A) discuss issues and disseminate information about commodities
B) keep consumer countries from controlling commodity prices
C) allow producer countries to control inflation by keeping commodity prices low
D) stabilize prices through regional alliances between producer and consumer countries
Answer: A
Diff: 2
Skill: Concept
Objective: 6
56) An example of an effective commodity agreement is ________.
A) the Organization of Petroleum Exporting Countries
B) the Organization of Petroleum Importing Countries
C) the International Tin Alliance
D) the International Cotton Alliance
Answer: A
Diff: 2
Skill: Concept
Objective: 6

57) The Organization of Petroleum Exporting Countries ________.


A) uses import tariffs to control oil prices
B) stabilizes prices based on demand
C) uses quotas to control oil prices
D) produces all of the world's oil
Answer: C
Diff: 2
Learning Outcome: Discuss arguments for and against regional economic integration
Skill: Concept
Objective: 6

58) Which of the following primarily triggered Toyota's investment in the European Union?
A) Europeans demanded affordable, high quality cars from Japan.
B) A common currency gave Europeans more buying power.
C) The EU lowered trade barriers on foreign auto imports.
D) Japan and the EU formed a free trade agreement.
Answer: C
Diff: 2
Skill: Concept
Objective: 2
AACSB: Analytical thinking

59) Which of the following is Walmart's competitive advantage in Mexico?


A) Walmart purchased all Mexican retail chains and eliminated competitors.
B) Walmart is able to transfer its "everyday low prices" concept to Mexico.
C) Walmart offers affordable, unique products for niche markets in Mexico.
D) Customers prefer to buy from American companies instead of Mexican companies.
Answer: B
Diff: 2
Learning Outcome: Discuss arguments for and against regional economic integration
Skill: Concept
Objective: 2
60) What was the primary purpose of the formation of Sinergia?
A) charge tariffs on Walmart imports
B) purchase Walmart's major suppliers
C) better compete with Walmart on price
D) include Walmart in the regional supply chain
Answer: C
Diff: 2
Learning Outcome: Discuss arguments for and against regional economic integration
Skill: Concept
Objective: 2

61) Regional integration is better known as global integration through the World Trade
Organization.
Answer: FALSE
Diff: 1
Skill: Concept
Objective: 2

62) The European Union is an example of regional integration.


Answer: TRUE
Diff: 2
Skill: Concept
Objective: 2

63) As companies expand internationally, they must change their organizational structure and
operating strategies to take advantage of regional trading groups.
Answer: TRUE
Diff: 2
Skill: Concept
Objective: 2

64) Most MNEs generate a majority of their revenues in their home regions.
Answer: FALSE
Diff: 2
Skill: Concept
Objective: 2

65) GATT's contribution to trade liberalization made possible the expansion of world trade in
the second half of the twentieth century.
Answer: TRUE
Diff: 2
Skill: Concept
Objective: 1
66) The most-favored-nation policy is a WTO privilege that allows member nations to restrict
tariff cuts to members.
Answer: FALSE
Diff: 2
Skill: Concept
Objective: 1

67) One of the reasons that neighboring countries tend to ally is similar consumer tastes.
Answer: TRUE
Diff: 1
Skill: Concept
Objective: 2
AACSB: Dynamics of the global economy

68) Most trade groups contain countries in the same area of the world, even though
neighboring countries usually lack a common history and interests.
Answer: FALSE
Diff: 2
Skill: Concept
Objective: 2
AACSB: Dynamics of the global economy

69) The goal of a free trade agreement is to abolish all tariffs among member countries.
Answer: TRUE
Diff: 2
Learning Outcome: Discuss arguments for and against regional economic integration
Skill: Concept
Objective: 2

70) When free mobility of factors of production is added to a common market, the result is a
customs union.
Answer: FALSE
Diff: 2
Skill: Concept
Objective: 2

71) Static effects are the overall growth in the market and the impact on a company caused by
expanding production and by the company's ability to achieve greater economies of scale.
Answer: FALSE
Diff: 1
Skill: Concept
Objective: 3
72) The shifting of resources from inefficient to efficient companies as trade barriers fall
produces static effects.
Answer: TRUE
Diff: 1
Skill: Concept
Objective: 3

73) Trade creation allows consumers access to more goods at a lower price than would have
been possible without integration.
Answer: TRUE
Diff: 2
Learning Outcome: Discuss arguments for and against regional economic integration
Skill: Concept
Objective: 2

74) The shifting of trade to countries in a regional group at the expense of trade with
countries not in the group is known as trade internalization.
Answer: FALSE
Diff: 2
Learning Outcome: Discuss arguments for and against regional economic integration
Skill: Concept
Objective: 2

75) The European Commission is the European Union's ultimate decision-making body and
is composed of the different ministers of the member countries.
Answer: FALSE
Diff: 2
Skill: Concept
Objective: 5

76) The three major responsibilities of the European Parliament are legislative power, control
over the budget, and supervision of executive decisions.
Answer: TRUE
Diff: 2
Skill: Concept
Objective: 5

77) The introduction of the euro has eliminated currency as a barrier to trade in all European
countries.
Answer: FALSE
Diff: 2
Skill: Concept
Objective: 5
78) The Single European Act set steps to accomplish monetary union in the European Union,
including the creation of the euro.
Answer: FALSE
Diff: 2
Skill: Concept
Objective: 5

79) Production location is not an important choice when doing business in the European
Union.
Answer: FALSE
Diff: 1
Skill: Concept
Objective: 5

80) Although the European Union is a common market, member countries have different
economic growth rates.
Answer: TRUE
Diff: 2
Skill: Concept
Objective: 5

81) NAFTA is an example of a common market.


Answer: FALSE
Diff: 2
Skill: Concept
Objective: 5

82) The most important rationales for NAFTA are geographic proximity and trading
importance.
Answer: TRUE
Diff: 2
Skill: Concept
Objective: 5

83) Each country in NAFTA sets its own tariffs to the rest of the world.
Answer: TRUE
Diff: 2
Skill: Concept
Objective: 5

84) The major trade groups in South America are CAN and MERCOSUR.
Answer: TRUE
Diff: 1
Skill: Concept
Objective: 5
85) The formation of CARICOM has primarily been triggered by the desire of Jamaica,
Trinidad, and Tobago to expand the region's market size and attract more FDI.
Answer: TRUE
Diff: 1
Learning Outcome: Discuss arguments for and against regional economic integration
Skill: Concept
Objective: 5

86) An NGO is a private institution independent from the government.


Answer: TRUE
Diff: 2
Skill: Concept
Objective: 6

87) NGOs must be recognized by the United Nations in order to do humanitarian work in
developing countries.
Answer: FALSE
Diff: 2
Skill: Application
Objective: 6
AACSB: Analytical thinking

88) Very few commodity agreements are successful in bringing together supplier and
consumer countries to stabilize commodity prices.
Answer: TRUE
Diff: 2
Learning Outcome: Discuss arguments for and against regional economic integration
Skill: Concept
Objective: 6

89) OPEC is an example of a producer's cartel that is successful because of its ability to
institute tariffs on oil exports.
Answer: FALSE
Diff: 2
Skill: Concept
Objective: 6

90) Toyota has been successful in Europe by exporting cars to Europe as well as designing
and manufacturing cars in Europe for the European market.
Answer: TRUE
Diff: 2
Skill: Concept
Objective: 2
91) Describe the different types of regional economic integration and give an example of
each type.
Answer:
a. Free trade area (FTA): The goal of a free trade area is to abolish all tariffs among member
countries. Free trade agreements usually begin modestly by eliminating tariffs on goods that
already have low tariffs, and there is usually an implementation period over which all tariffs
are eliminated on all products. In addition, each member country maintains its own external
tariffs against non-FTA countries. Examples: the North American Free Trade Agreement, the
Association of South East Asian Nations
b. Customs union: In addition to eliminating internal tariffs, member countries levy a
common external tariff on goods being imported from nonmembers. Example: MERCOSUR
c. Common market: A common market has all the elements of a customs union, plus it
allows free mobility of production factors such as labor and capital. Example: the European
Union
Diff: 3
Learning Outcome: Discuss arguments for and against regional economic integration
Skill: Application
Objective: 4
AACSB: Reflective thinking

92) Explain the static effects and dynamic effects of economic integration. What is the
difference between trade creation and trade diversion resulting from economic integration?
Answer: Static effects are the shifting of resources from inefficient to efficient companies as
trade barriers fall. Dynamic effects are the overall growth in the market and the impact on a
company of expanding production and achieving greater economies of scale. Static effects
may develop when either of two conditions occurs:
a. Trade creation: Production shifts to more efficient producers for reasons of comparative
advantage, allowing consumers access to more goods at a lower price than would have been
possible without integration.
b. Trade diversion: Trade shifts to countries in the group at the expense of trade with
countries not in the group, even though the nonmember company might be more efficient in
the absence of trade barriers.
Dynamic effects of integration occur when trade barriers come down and the size of the
market increases, allowing companies to achieve economies of scale.
Diff: 3
Learning Outcome: Discuss arguments for and against regional economic integration
Skill: Critical Thinking
Objective: 3
AACSB: Analytical thinking
93) What are the functions of the European Commission, the European Parliament, the
Council, and the European Court of Justice?
Answer:
a. The European Commission provides the European Union's political leadership and
direction. The commission is composed of commissioners nominated by each member
government and approved by the European Parliament. It drafts laws that it submits to the
European Parliament and Council of the EU.
b. The three major responsibilities of the European Parliament are: legislative power, control
over the budget, and supervision of executive decisions. The commission presents
community legislation to the parliament. Parliament may approve legislation, amend it, or
reject it outright. Parliament also approves the EU's budget each year and monitors spending.
c. The Council is composed of the ministers of the member countries. The Council passes
laws and makes and enacts major policies. It works closely with the Commission and
Parliament in adopting policies.
d. The European Court of Justice ensures consistent interpretation and application of EU
treaties. Member states, community institutions, or individuals and companies may bring
cases to the court. The Court of Justice is an appeals court for individuals, firms, and
organizations fined by the commission for infringing treaty law. The Court of Justice is
relevant to MNEs because it deals mostly with economic matters.
Diff: 3
Skill: Application
Objective: 5
AACSB: Analytical thinking

94) What are the rules of origin and regional content provisions of NAFTA?
Answer: Because NAFTA is a free trade agreement and not a customs union, each country
sets its own tariffs for the rest of the world. Rules of origin ensure that only goods that have
been the subject of substantial economic activity within the free trade area are eligible for the
more liberal tariff conditions created by NAFTA. According to regional content rules, at least
50 percent of the net cost of most products must come from the NAFTA region. The
exceptions are 55 percent for footwear, 62.5 percent for passenger automobiles and light
trucks and the engines and transmissions for such vehicles, and 60 percent for other vehicles
and automotive parts.
Diff: 3
Skill: Application
Objective: 5
AACSB: Analytical thinking
95) What has been the impact of NAFTA on trade and employment in NAFTA nations?
Answer: Trade and investment among the NAFTA members has increased significantly
since the agreement was signed in 1994. The U.S. is the largest trade partner of Canada and
Mexico, and both countries are among the most important exporters and importers for the
U.S. Due to lower wages in Mexico, a lot of FDI has poured into Mexico, potentially
displacing jobs in the United States. U.S. firms have come under criticism for taking
advantage of cheaper wages and lax environmental standards. In addition, the agreement has
not stopped the flow of illegal immigrants from Mexico to the U.S.
Diff: 3
Learning Outcome: Discuss arguments for and against regional economic integration
Skill: Synthesis
Objective: 2, 5
AACSB: Reflective thinking

96) Identify and briefly compare the major regional trading groups in Latin America, Asia,
and Africa.
Answer:
a. The major trade group in South America is MERCOSUR. In 1991, Brazil, Argentina,
Paraguay, and Uruguay established MERCOSUR. MERCOSUR is significant because of its
size; it generates 75 percent of South America's GNP. Another major group in South America
is the Andean Group (CAN), which is composed of Bolivia, Colombia, Ecuador, and Peru.
There are three major regional trading groups in Central America and the Caribbean: the
Central American Common Market, the Central American Free Trade Agreement-Dominican
Republic (which includes the United States), and the Caribbean Community and Common
Market (CARICOM). These groups are hampered by their small markets and dependence on
the United States for trade.
b. In Asia, the key group is the Association of South East Asian Nations (ASEAN), which
was organized in 1967 and comprises Brunei, Cambodia, Indonesia, Laos, Malaysia,
Myanmar, the Philippines, Singapore, Thailand, and Vietnam. It is promoting cooperation in
many areas, including industry and trade. In 1993, the ASEAN countries formed the ASEAN
Free Trade Area (AFTA) to deal with the specific intrazonal trade issues.
c. The Asia Pacific Economic Cooperation (APEC) is massive since it includes every
country that borders the Pacific Ocean. In spite of the size of APEC, it does not engage in
treaties like the other trade agreements, so it has potential but not much teeth.
d. Africa is divided into many different trading groups based on geographic proximity and
links to former colonial powers. Most groups are hampered by poverty, small market size,
and dependence on former colonial powers. The African Union is modeled loosely on the
EU, but that type of integration will likely be very difficult.
Diff: 3
Learning Outcome: Discuss arguments for and against regional economic integration
Skill: Critical Thinking
Objective: 5
AACSB: Analytical thinking
97) Are commodity agreements effective? Why or why not?
Answer: Commodity agreements used to be influential in helping to stabilize commodity
prices, but now they are more involved in disseminating information and promoting research.
OPEC is an example of an effective producers' cartel that operates on quotas to try to
stabilize prices. In general, very little can be done outside of market forces to influence price.
Diff: 3
Skill: Critical Thinking
Objective: 6
AACSB: Reflective thinking

98) Why is geography important to most regional trade agreements? Provide examples of
RTAs to illustrate your answer.
Answer: There are a number of reasons why geography matters in the case of RTAs.
Neighboring countries often, though not always, share a common history, language, culture,
and currency. Unless the countries are at war with each other, they usually have already
developed trading ties. Close proximity reduces transportation costs, thereby making traded
products cheaper in general. Armenia has RTAs in force with Kazakhstan, Moldova, the
Russian Federation, Turkmenistan, and Ukraine. India has a number of trade agreements with
most of the countries in its region. Germany, a member of the European Union, exports 62.9
percent of its merchandise exports to other EU members and imports 58.3 percent from them.
Switzerland, which is not a member of the EU but which has a trade agreement with the EU,
exports 59.7 percent of its merchandise exports to EU countries and imports 78 percent from
them. NAFTA includes Canada, the United States, and Mexico.
Diff: 3
Learning Outcome: Discuss arguments for and against regional economic integration
Skill: Synthesis
Objective: 2, 5

99) In a brief essay, explain the roles of the World Trade Organization and the United
Nations in international trade.
Answer: The World Trade Organization (WTO) replaced GATT in 1995 as a continuing
means of trade negotiations that aspires to foster the principle of trade without discrimination
and to provide a better means of mediating trade disputes and of enforcing agreements. The
United Nations is composed of representatives of most of the countries in the world and
influences international trade and development in a number of significant ways. The UN
family of organizations is too large to list, but it includes the WTO, the International
Monetary Fund, and the World Bank. If the UN performs its responsibilities, it should
improve the environment in which MNEs operate around the world, reducing risk and
providing greater opportunities.
Diff: 3
Skill: Synthesis
Objective: 1, 6
AACSB: Dynamics of the global economy
100) What is the difference between a free trade agreement and a customs union? Provide
examples of each in your answer.
Answer: The goal of an FTA is to abolish all tariffs between member countries. It usually
begins modestly by eliminating tariffs on goods that already have low tariffs, and there is
usually an implementation period during which all tariffs are eliminated on all products.
Moreover, each member country maintains its own external tariffs against non-FTA
countries. NAFTA is an example of a free trade agreement. The EU is considered a customs
union by the WTO. In addition to eliminating internal tariffs, member countries levy a
common external tariff on goods being imported from nonmembers. For example, the EU
removed internal tariffs from 1959 to 1967, when it established a common external tariff.
Now it negotiates as one region in the WTO rather than as separate countries. Customs
unions account for less than 10 percent of the RTAs identified by the WTO.
Diff: 3
Learning Outcome: Discuss arguments for and against regional economic integration
Skill: Synthesis
Objective: 4, 5
AACSB: Dynamics of the global economy
International Business: Environments and Operations, 15e (Daniels et al.)
Chapter 12 The Strategy of International Business

1) Which of the following statements about Zara is most likely FALSE?


A) Zara puts fewer products on clearance racks than most of its competitors in the industry.
B) Zara's large advertising budget generates word-of-mouth and attracts new buyers.
C) New products and designs are delivered to Zara stores every three to four weeks.
D) Most of Zara's products move through the firm's distribution center in Spain.
Answer: B
Diff: 3
Learning Outcome: Describe the process of selecting and developing an international
business strategy
Skill: Concept
Objective: 4
AACSB: Analytical thinking

2) Political, legal, economic, monetary, and institutional forces comprise the ________ of
international business and influence managers' actions.
A) environment
B) competition
C) culture
D) threat
Answer: A
Diff: 2
Learning Outcome: Describe the process of selecting and developing an international
business strategy
Skill: Concept
Objective: 1
AACSB: Application of knowledge

3) Phillip is an international business manager with Corbin Manufacturing. Which of the


following serves as an external influence on the business decisions that Phillip makes?
A) production plant locations
B) host country monetary policy
C) supply chain linkages
D) product design standards
Answer: B
Diff: 2
Learning Outcome: Describe the process of selecting and developing an international
business strategy
Skill: Application
Objective: 1
AACSB: Analytical thinking
4) ________ is the framework that managers apply to determine the competitive moves and
business approaches that run the company.
A) Competition
B) Growth
C) Strategy
D) Vision
Answer: C
Diff: 1
Learning Outcome: Describe the process of selecting and developing an international
business strategy
Skill: Concept
Objective: 1
AACSB: Application of knowledge

5) Which force in Toyota's immediate environment would most likely have the greatest
impact on its strategy?
A) interest rate trends
B) technology developments
C) shifts in U.S. political attitudes
D) actions taken by Honda and Mercedes Benz
Answer: D
Diff: 2
Learning Outcome: Describe the process of selecting and developing an international
business strategy
Skill: Application
Objective: 1
AACSB: Reflective thinking

6) Which of the following is NOT among the five forces in the Five-Forces Model of
Industry Structure?
A) governments
B) substitute products
C) potential new entrants
D) suppliers of raw materials
Answer: A
Diff: 2
Learning Outcome: Describe the process of selecting and developing an international
business strategy
Skill: Concept
Objective: 1
AACSB: Application of knowledge
7) Which of the following is the best example of a product disruption driving industry
change?
A) redesign of Toyota's Prius
B) introduction of Apple's iPad
C) AT&T's purchase of T-Mobile
D) merger of United and Continental
Answer: B
Diff: 2
Learning Outcome: Describe the process of selecting and developing an international
business strategy
Skill: Application
Objective: 1
AACSB: Analytical thinking

8) The industry organization (IO) paradigm reports that, on average, the best predictor of firm
strategy is the ________.
A) company's stockpile of assets, skills, and capabilities
B) aggressiveness of a company's marketing objectives
C) link between a company's products and processes
D) structure of the industry in which it competes
Answer: D
Diff: 2
Learning Outcome: Describe the process of selecting and developing an international
business strategy
Skill: Concept
Objective: 1
AACSB: Application of knowledge

9) The Industry Organization (IO) paradigm assumes which of the following?


A) stable political and cultural trends
B) shifting foreign-exchange rates
C) perfect competition
D) unequal value
Answer: C
Diff: 2
Learning Outcome: Describe the process of selecting and developing an international
business strategy
Skill: Concept
Objective: 1
AACSB: Application of knowledge
10) Which of the following is the most important constraint on the explanatory power of the
IO paradigm?
A) Many industries are imperfectly competitive.
B) Many companies are locally but not internationally competitive.
C) Most customers have perfect knowledge of the products they select.
D) Most industries have many firms that each have small market shares.
Answer: A
Diff: 2
Learning Outcome: Describe the process of selecting and developing an international
business strategy
Skill: Concept
Objective: 1
AACSB: Application of knowledge

11) Improving explanatory power of the IO paradigm can be done by considering the
potential for ________ to lead to a company's sustained competitive advantage.
A) bright, motivated managers
B) political trends and events
C) cultural institutions
D) new markets
Answer: A
Diff: 2
Learning Outcome: Describe the process of selecting and developing an international
business strategy
Skill: Concept
Objective: 1
AACSB: Application of knowledge

12) As a result of the global economic crisis, the potential for profitability in all sorts of
industries has ________.
A) decreased
B) increased
C) become less predictable
D) become more projectable
Answer: C
Diff: 1
Learning Outcome: Describe the process of selecting and developing an international
business strategy
Skill: Concept
Objective: 1
AACSB: Application of knowledge
13) Which of the following has the greatest potential to transform an industry's structure?
A) a change in a competitor's management
B) a change in a competitor's pricing structure
C) the expansion of a distribution channel
D) the exit of a competitor from the industry
Answer: D
Diff: 2
Learning Outcome: Describe the process of selecting and developing an international
business strategy
Skill: Concept
Objective: 1
AACSB: Analytical thinking

14) Which of the following is the purpose of a company's value proposition?


A) to exceed customers' expectations
B) to force competitors into a price war
C) to identify consumers for whom the company creates products
D) to explain why a consumer should buy the company's products
Answer: D
Diff: 2
Learning Outcome: Describe the process of selecting and developing an international
business strategy
Skill: Concept
Objective: 1
AACSB: Application of knowledge

15) A strategy that focuses on lowering operating costs is referred to as a ________ strategy.
A) differentiation
B) cost leadership
C) niche market
D) diversification
Answer: B
Diff: 1
Learning Outcome: Describe the process of selecting and developing an international
business strategy
Skill: Concept
Objective: 1
AACSB: Application of knowledge
16) Value-Mart has achieved the status of overall low-cost producer in its industry, which
means that Value-Mart ________.
A) can strongly defend its market position in the event of a price war
B) will earn the largest profits of any company in the industry
C) makes the most appealing product in its industry
D) can block the entry of new firms into the industry
Answer: A
Diff: 2
Learning Outcome: Describe the process of selecting and developing an international
business strategy
Skill: Application
Objective: 1
AACSB: Analytical thinking

17) ________ are the two basic strategies for creating value and attaining a competitive
advantage in an industry.
A) Diversification and niche marketing
B) Industry leadership and market dominance
C) Customer satisfaction and product innovation
D) Cost leadership and differentiation
Answer: D
Diff: 1
Learning Outcome: Describe the process of selecting and developing an international
business strategy
Skill: Concept
Objective: 1
AACSB: Application of knowledge

18) A company that employs a ________ strategy creates value by generating customer
insights and translating those insights into innovations.
A) differentiation
B) diversification
C) cost leadership
D) innovation
Answer: A
Diff: 1
Learning Outcome: Describe the process of selecting and developing an international
business strategy
Skill: Concept
Objective: 1
AACSB: Analytical thinking
19) The essence of a differentiation strategy is to ________.
A) target the most sophisticated segment of the market
B) incorporate the greatest number of features into a product
C) offer a unique product that supports a premium price
D) outspend rivals on product advertising
Answer: C
Diff: 2
Learning Outcome: Describe the process of selecting and developing an international
business strategy
Skill: Concept
Objective: 1
AACSB: Application of knowledge

20) A differentiation strategy works well when ________.


A) a company designs a universal product for a broad market
B) buyers believe minor product differences are irrelevant to price
C) buyers are highly sensitive to price shifts and quality control
D) a company can continually develop products that have unique features
Answer: D
Diff: 2
Learning Outcome: Describe the process of selecting and developing an international
business strategy
Skill: Concept
Objective: 1
AACSB: Analytical thinking

21) Successful implementation of a ________ strategy requires a company to develop unique


competencies that rivals find hard, if not impossible, to match or copy.
A) differentiation
B) cost leadership
C) globalization
D) marketing
Answer: A
Diff: 2
Learning Outcome: Describe the process of selecting and developing an international
business strategy
Skill: Concept
Objective: 1
AACSB: Analytical thinking
22) A useful way to understanding the purpose of strategy is to think of the firm as a(n)
________, composed of a series of distinct activities, including production, marketing,
materials management, R&D, human resources, information systems, and the firm
infrastructure.
A) activity network
B) value constellation
C) task succession
D) value chain
Answer: D
Diff: 1
Learning Outcome: Describe the process of selecting and developing an international
business strategy
Skill: Concept
Objective: 2
AACSB: Reflective thinking

23) A company's value chain is best described as the ________.


A) variable sequence of converting product ideas into value for shareholders
B) mixture of activities taken to define product value standards and price points
C) discrete series of steps taken to move a product from conception to end-users
D) blueprint that must be followed to leverage the core capabilities of a global firm
Answer: C
Diff: 2
Learning Outcome: Describe the process of selecting and developing an international
business strategy
Skill: Concept
Objective: 2
AACSB: Application of knowledge

24) In the context of value chain analysis, the support activities of a firm include ________.
A) outbound logistics, human resources, and information systems
B) production, marketing, and operations
C) shipping and customer service
D) research and development
Answer: C
Diff: 2
Learning Outcome: Describe the process of selecting and developing an international
business strategy
Skill: Concept
Objective: 2
AACSB: Application of knowledge
25) Which of the following is considered a primary activity in a company's value chain?
A) shipping
B) product design
C) systems and solutions
D) materials and equipment
Answer: B
Diff: 2
Learning Outcome: Describe the process of selecting and developing an international
business strategy
Skill: Concept
Objective: 3
AACSB: Application of knowledge

26) Dispersing value activities where and in how many places in the world is the issue of
________.
A) logistics
B) clustering
C) coordination
D) configuration
Answer: D
Diff: 1
Learning Outcome: Describe the process of selecting and developing an international
business strategy
Skill: Concept
Objective: 3
AACSB: Application of knowledge

27) Citibank decides to open a call center in Mumbai, India because a detailed analysis of the
country-specific advantages suggests that India is the optimal place for responding to
customers' calls. Citibank is exploiting ________ by running a call center in India.
A) value configuration synergies
B) location economies
C) geographic arbitrage
D) value offshoring
Answer: B
Diff: 2
Learning Outcome: Describe the process of selecting and developing an international
business strategy
Skill: Application
Objective: 3
AACSB: Analytical thinking
28) Zara creates, produces, and distributes most of its products from its headquarters in
Spain. Zara is most likely using a ________ configuration.
A) concentrated
B) coordinated
C) dispersed
D) logical
Answer: A
Diff: 2
Learning Outcome: Describe the process of selecting and developing an international
business strategy
Skill: Application
Objective: 3
AACSB: Analytical thinking

29) Johnson Enterprises manufactures computer chips used in cell phones. The MNE has
long production runs that lower the per unit cost of each chip. The firm also purchases
materials in bulk from its suppliers with whom Johnson has long-term contracts. Johnson is
most likely benefiting from ________.
A) economies of scale
B) the cluster effect
C) digitization
D) location economies
Answer: A
Diff: 2
Learning Outcome: Describe the process of selecting and developing an international
business strategy
Skill: Application
Objective: 3
AACSB: Application of knowledge

30) A ________ is a special outlook, skill, capability, or technology that runs through the
firm's operations, weaving together all value activities into an integrated value chain.
A) core competency
B) value proposition
C) mission statement
D) learning curve
Answer: A
Diff: 1
Learning Outcome: Describe the process of selecting and developing an international
business strategy
Skill: Concept
Objective: 3
AACSB: Application of knowledge
31) Which of the following refers to the means by which management applies the systems
that link a company's value activities, whether those activities are performed in one or in
many countries?
A) configuration
B) logistics
C) coordination
D) core competency
Answer: C
Diff: 1
Learning Outcome: Describe the process of selecting and developing an international
business strategy
Skill: Concept
Objective: 3
AACSB: Application of knowledge

32) Social network analysis indicates that information flows more efficiently in a(n)
________ manner.
A) collaborative
B) competitive
C) hierarchical
D) impersonal
Answer: A
Diff: 2
Learning Outcome: Describe the process of selecting and developing an international
business strategy
Skill: Concept
Objective: 3
AACSB: Information technology

33) Firms that compete in the global marketplace typically face the asymmetric forces of
pressures for ________ and ________.
A) global integration; local responsiveness
B) price reductions; cost reductions
C) politically sensitivity; market leadership
D) cost reductions; customer satisfaction
Answer: A
Diff: 1
Learning Outcome: Describe the process of selecting and developing an international
business strategy
Skill: Concept
Objective: 4
AACSB: Application of knowledge
34) No matter the country or culture, money has three fundamental features that motivate
consumers around the world to maximize purchasing power by buying the highest possible
quality product for the lowest possible price. These three features include all of the following
EXCEPT which one?
A) scarcity
B) difficulty of acquisition
C) difficulty of allocation
D) difficulty of saving
Answer: C
Diff: 2
Learning Outcome: Describe the process of selecting and developing an international
business strategy
Skill: Concept
Objective: 4
AACSB: Analytical thinking

35) In the demand-pull versus supply-push views of the market, which of the following is
best classified as a push dynamic that drives supply in the global market?
A) digitization
B) media access
C) standardization
D) market globalization
Answer: C
Diff: 2
Learning Outcome: Describe the process of selecting and developing an international
business strategy
Skill: Concept
Objective: 4
AACSB: Application of knowledge

36) Which of the following would most likely pressure an MNE to globally integrate its value
activities?
A) costs of producing in separate facilities exceeding those of producing in a single facility
B) variability in consumers' tastes and preferences shifting across countries
C) market pressures to add new product features continually
D) demands imposed by host country governments
Answer: A
Diff: 2
Learning Outcome: Describe the process of selecting and developing an international
business strategy
Skill: Concept
Objective: 4
AACSB: Analytical thinking
37) Which of the following is LEAST likely to trigger the standardization of consumer
preferences?
A) intrinsic functions of money
B) global access to common media
C) increasing nationalism
D) improved communications technologies
Answer: C
Diff: 2
Learning Outcome: Describe the process of selecting and developing an international
business strategy
Skill: Concept
Objective: 4
AACSB: Reflective thinking

38) Which of the following is NOT a factor driving local responsiveness among global firms?
A) cross-national differences in distribution channels
B) diverging consumer tastes and preferences
C) cross-national differences in product standards
D) host government support of freer international trade
Answer: D
Diff: 2
Learning Outcome: Describe the process of selecting and developing an international
business strategy
Skill: Concept
Objective: 4
AACSB: Reflective thinking

39) Advantages of following a strategy of local responsiveness include the ________.


A) improved ability to transfer skills to employees in different locations
B) opportunity to adopt a global advertising program
C) ease in standardizing manufacturing methods across countries
D) flexibility to apply location-specific skills to local opportunities
Answer: D
Diff: 2
Learning Outcome: Describe the process of selecting and developing an international
business strategy
Skill: Concept
Objective: 4
AACSB: Analytical thinking
40) Host governments' growing demands for increasing operational transparency will most
likely spur companies to make their value chains more ________.
A) locally responsive
B) globally integrated
C) concise
D) complex
Answer: A
Diff: 2
Learning Outcome: Describe the process of selecting and developing an international
business strategy
Skill: Concept
Objective: 4
AACSB: Analytical thinking

41) According to the integration-responsiveness (IR) grid, consumer electronics and


corporate banking are two types of industries with ________ industry pressure for global
integration and ________ industry pressure for local responsiveness.
A) low; low
B) high; high
C) low; high
D) high; low
Answer: B
Diff: 3
Learning Outcome: Describe the process of selecting and developing an international
business strategy
Skill: Concept
Objective: 4
AACSB: Application of knowledge

42) In the global environment, firms use an international strategy, a multidomestic strategy, a
global strategy, or a ________ strategy.
A) regional
B) standardization
C) transnational
D) locational
Answer: C
Diff: 1
Learning Outcome: Describe the process of selecting and developing an international
business strategy
Skill: Concept
Objective: 5
AACSB: Application of knowledge
43) The integration-responsiveness (IR) grid expresses how a company's ________ is a
function of the relationship between its value chain and the prevailing pressures for global
integration or local responsiveness in its industry.
A) value proposition
B) target market
C) mission
D) strategy
Answer: D
Diff: 1
Learning Outcome: Describe the process of selecting and developing an international
business strategy
Skill: Concept
Objective: 5
AACSB: Analytical thinking

44) When the pressure for global integration is high and the pressure for local responsiveness
is low, a company is most likely to pursue a(n) ________ strategy.
A) global
B) transnational
C) multidomestic
D) international
Answer: D
Diff: 2
Learning Outcome: Describe the process of selecting and developing an international
business strategy
Skill: Concept
Objective: 5
AACSB: Analytical thinking

45) Firms that pursue a(n) ________ strategy create value by transferring core competencies
from the home market to foreign markets in which local rivals lack a competitive alternative.
A) transnational
B) multidomestic
C) global
D) international
Answer: D
Diff: 2
Learning Outcome: Describe the process of selecting and developing an international
business strategy
Skill: Concept
Objective: 5
AACSB: Application of knowledge
46) Which of the following is NOT a disadvantage of the international strategy?
A) Headquarters can misread foreign-market opportunities and threats.
B) The transfer of core competencies to foreign markets is complicated by the need to adapt
to local needs.
C) Local subsidiaries are not given the opportunity to adapt value activities and share what
they have learned with headquarters.
D) The company can be blindsided by an unexpectedly innovative rival in a foreign market.
Answer: B
Diff: 3
Learning Outcome: Describe the process of selecting and developing an international
business strategy
Skill: Concept
Objective: 5
AACSB: Analytical thinking

47) A distinct advantage of an international strategy is ________.


A) transferring core competencies to foreign markets
B) reaping benefits of global learning
C) customizing product offerings to local conditions
D) leveraging local managers' knowledge of their markets
Answer: A
Diff: 2
Learning Outcome: Describe the process of selecting and developing an international
business strategy
Skill: Concept
Objective: 5
AACSB: Analytical thinking

48) A distinct disadvantage of an international strategy is ________.


A) restricting the movement of core competencies to local markets
B) capitalizing on the benefits of global learning
C) customizing product offerings to local conditions
D) relying on home managers' knowledge of foreign markets
Answer: D
Diff: 2
Learning Outcome: Describe the process of selecting and developing an international
business strategy
Skill: Concept
Objective: 5
AACSB: Analytical thinking
49) Darvin Foods is an MNE that is in the process of revising its international business
strategy. Most of the firm's top executives believe that Darvin should implement a
multidomestic strategy. Which of the following conditions would most likely support this
type of strategy?
A) a high need for local responsiveness and a low need to reduce costs via global integration
B) a high need for local responsiveness and a high need to create efficiencies via global
integration
C) a low need for local responsiveness and a low need to reduce costs via global integration
D) a low need for local responsiveness and a high need to create efficiencies via global
integration
Answer: A
Diff: 2
Learning Outcome: Describe the process of selecting and developing an international
business strategy
Skill: Application
Objective: 5
AACSB: Reflective thinking

50) Preston Electronics is an MNE with facilities located in Taiwan, Singapore, and
Germany. Preston gives its local operations the authority to adapt value activities to
prevailing local economic, political, legal, and cultural conditions. Preston is most likely
using a(n) ________ strategy.
A) global
B) multidomestic
C) international
D) continental
Answer: B
Diff: 2
Learning Outcome: Describe the process of selecting and developing an international
business strategy
Skill: Application
Objective: 5
AACSB: Application of knowledge

51) Johnson & Johnson delegates to its subsidiaries a great deal of authority to respond to
local conditions. Many subsidiaries have their own manufacturing, marketing, research, and
human resource functions. This value chain configuration illustrates the ________ strategy.
A) transnational
B) global
C) multidomestic
D) international
Answer: C
Diff: 2
Learning Outcome: Describe the process of selecting and developing an international
business strategy
Skill: Application
Objective: 5
AACSB: Application of knowledge

52) A disadvantage of the ________ strategy is that firms that adopt this strategy can
decentralize too much autonomy to national subsidiaries.
A) multidomestic
B) continental
C) international
D) transnational
Answer: A
Diff: 2
Learning Outcome: Describe the process of selecting and developing an international
business strategy
Skill: Concept
Objective: 5
AACSB: Analytical thinking

53) Google essentially views the entire world as one market and assumes that there are no
fundamental differences among countries with regard to consumers' preferences. Google is
engaging a(n) ________ strategy.
A) international
B) multidomestic
C) transnational
D) global
Answer: D
Diff: 2
Learning Outcome: Describe the process of selecting and developing an international
business strategy
Skill: Application
Objective: 5
AACSB: Reflective thinking

54) A global strategy tends to be more successful when ________.


A) buyers are looking for customized products at bargain prices
B) the industry's product is a commodity
C) entry barriers are low for new firms
D) suppliers have little bargaining power
Answer: B
Diff: 2
Learning Outcome: Describe the process of selecting and developing an international
business strategy
Skill: Concept
Objective: 5
AACSB: Analytical thinking
55) The MNE that applies a global strategy aims to ________.
A) compete essentially the same way wherever the company does business
B) empower local operations to make decisions about value activities
C) adapt activities to the industry standards in critical markets
D) customize its business practices to consumer expectations
Answer: A
Diff: 2
Learning Outcome: Describe the process of selecting and developing an international
business strategy
Skill: Concept
Objective: 5
AACSB: Application of knowledge

56) Which strategy champions worldwide consistency and standardization to support the
firm's goal to become the low-cost leader?
A) international strategy
B) multidomestic strategy
C) global strategy
D) transnational strategy
Answer: C
Diff: 2
Learning Outcome: Describe the process of selecting and developing an international
business strategy
Skill: Concept
Objective: 5
AACSB: Application of knowledge

57) An MNE in which new knowledge and capabilities are developed in both domestic and
foreign locations, both independently and jointly, and then diffused throughout the worldwide
organization, is following a ________ strategy.
A) transnational
B) multidomestic
C) global
D) matrix
Answer: A
Diff: 1
Learning Outcome: Describe the process of selecting and developing an international
business strategy
Skill: Concept
Objective: 5
AACSB: Application of knowledge
58) Which of the following is the primary limitation of the transnational strategy?
A) poor local adaptation
B) cross-unit coordination difficulties
C) duplication of subsidiary activities
D) inability to leverage core competencies
Answer: B
Diff: 2
Learning Outcome: Describe the process of selecting and developing an international
business strategy
Skill: Concept
Objective: 5
AACSB: Application of knowledge

59) Marrin Associates is an MNE with operations in Hong Kong, India, and Canada. The
firm strives to leverage its core competencies worldwide, reduce costs by exploiting location
economics, and adapt when efficient to local conditions. Marrin is most likely following a(n)
________ strategy.
A) global
B) multidomestic
C) transnational
D) international
Answer: C
Diff: 2
Learning Outcome: Describe the process of selecting and developing an international
business strategy
Skill: Application
Objective: 5
AACSB: Analytical thinking

60) The term ________ refers to a global company that thrives on seeking unique ideas and
insights from locations around the world and then leveraging that knowledge in international
markets.
A) multidomestic
B) domestic
C) metanational
D) cross-cultural
Answer: C
Diff: 2
Learning Outcome: Describe the process of selecting and developing an international
business strategy
Skill: Concept
Objective: 5
AACSB: Application of knowledge
61) The framework that managers use to make decisions that maximize their companies'
value creation performance is referred to as a strategy.
Answer: TRUE
Diff: 2
Learning Outcome: Describe the process of selecting and developing an international
business strategy
Skill: Concept
Objective: 1
AACSB: Application of knowledge

62) Markets are not always perfectly competitive and some firms consistently outperform
industry averages. This suggests that firm performance is also influenced by the presence of
bright, motivated managers and their keen sense of innovative products or processes.
Answer: TRUE
Diff: 2
Learning Outcome: Describe the process of selecting and developing an international
business strategy
Skill: Concept
Objective: 1
AACSB: Reflective thinking

63) According to the Industry Organization perspective, firm performance is determined by


the presence of motivated managers and their ability to identify and develop innovative
products for emerging markets.
Answer: FALSE
Diff: 1
Learning Outcome: Describe the process of selecting and developing an international
business strategy
Skill: Concept
Objective: 1
AACSB: Analytical thinking

64) The potential of new entrants in an industry is one of the forces in the Five-Forces Model
of Industry Structure.
Answer: TRUE
Diff: 1
Learning Outcome: Describe the process of selecting and developing an international
business strategy
Skill: Concept
Objective: 1
AACSB: Application of knowledge
65) Change in the long-term industry growth rate has the potential to transform an industry's
structure.
Answer: TRUE
Diff: 2
Learning Outcome: Describe the process of selecting and developing an international
business strategy
Skill: Concept
Objective: 1
AACSB: Analytical thinking

66) Some companies that opt for the cost leadership strategy plan to sell products below the
average industry prices in order to capture market share.
Answer: TRUE
Diff: 2
Learning Outcome: Describe the process of selecting and developing an international
business strategy
Skill: Concept
Objective: 1
AACSB: Application of knowledge

67) When different companies produce the same commodity, their costs are essentially the
same.
Answer: FALSE
Diff: 2
Learning Outcome: Describe the process of selecting and developing an international
business strategy
Skill: Concept
Objective: 1
AACSB: Analytical thinking

68) A differentiation strategy is a unique mixture of the multidomestic and global strategies
whereby the company attempts to capture the advantages of both.
Answer: FALSE
Diff: 2
Learning Outcome: Describe the process of selecting and developing an international
business strategy
Skill: Concept
Objective: 1
AACSB: Reflective thinking
69) A differentiation strategy works well in situations where there are many ways to
differentiate a product or service and many buyers perceive these differences as having value.
Answer: TRUE
Diff: 2
Learning Outcome: Describe the process of selecting and developing an international
business strategy
Skill: Concept
Objective: 1
AACSB: Analytical thinking

70) Managers use a value chain analysis to determine how the company will design, make,
move, and sell products; how it will find efficiencies in doing so; and how it will coordinate
the decisions in one part of the business with those made in other parts.
Answer: TRUE
Diff: 2
Learning Outcome: Describe the process of selecting and developing an international
business strategy
Skill: Concept
Objective: 2
AACSB: Analytical thinking

71) Support activities define the infrastructure of the firm and serve as the basis for the daily
implementation of primary activities in the value chain.
Answer: TRUE
Diff: 2
Learning Outcome: Describe the process of selecting and developing an international
business strategy
Skill: Concept
Objective: 2
AACSB: Application of knowledge

72) Using a concentrated configuration, an MNE performs value activities in different


countries.
Answer: FALSE
Diff: 2
Learning Outcome: Describe the process of selecting and developing an international
business strategy
Skill: Concept
Objective: 3
AACSB: Application of knowledge
73) Dispersed value chains make sense when costs vary across countries.
Answer: TRUE
Diff: 2
Learning Outcome: Describe the process of selecting and developing an international
business strategy
Skill: Concept
Objective: 3
AACSB: Application of knowledge

74) A core competency gives every employee in an MNE a principle that helps them
coordinate transactions between value activities.
Answer: TRUE
Diff: 2
Learning Outcome: Describe the process of selecting and developing an international
business strategy
Skill: Concept
Objective: 3
AACSB: Application of knowledge

75) Digitization has altered location economics and established a new global model for
service providers in the financial and legal industries.
Answer: FALSE
Diff: 2
Learning Outcome: Describe the process of selecting and developing an international
business strategy
Skill: Concept
Objective: 3
AACSB: Information technology

76) Pressures for global integration include economic integration, convergent consumer
preferences, and political demands imposed by host countries.
Answer: FALSE
Diff: 1
Learning Outcome: Describe the process of selecting and developing an international
business strategy
Skill: Concept
Objective: 4
AACSB: Reflective thinking

77) The quest to maximize individual purchasing power compels local responsiveness.
Answer: FALSE
Diff: 2
Learning Outcome: Describe the process of selecting and developing an international
business strategy
Skill: Concept
Objective: 4
AACSB: Analytical thinking

78) Pressures for local responsiveness are especially important in industries where value
creation is a function of a company's capacity to adapt to local market conditions.
Answer: TRUE
Diff: 2
Learning Outcome: Describe the process of selecting and developing an international
business strategy
Skill: Concept
Objective: 4
AACSB: Analytical thinking

79) Pressures for local responsiveness include cross-national differences in terms of


consumer preferences, advances in technology, and government regulations.
Answer: FALSE
Diff: 1
Learning Outcome: Describe the process of selecting and developing an international
business strategy
Skill: Concept
Objective: 4
AACSB: Analytical thinking

80) The fundamental outlook of the multidomestic strategy is standardization.


Answer: FALSE
Diff: 1
Learning Outcome: Describe the process of selecting and developing an international
business strategy
Skill: Concept
Objective: 5
AACSB: Reflective thinking

81) A global strategy drives performance by making standardized products that are marketed
with minimum adaptation to local conditions.
Answer: TRUE
Diff: 1
Learning Outcome: Describe the process of selecting and developing an international
business strategy
Skill: Concept
Objective: 5
AACSB: Analytical thinking
82) The strategy of a firm using an international strategy is likely to entail producing and
marketing mostly standardized products worldwide, with some customization where and
when necessary.
Answer: TRUE
Diff: 2
Learning Outcome: Describe the process of selecting and developing an international
business strategy
Skill: Concept
Objective: 5
AACSB: Application of knowledge

83) Tel-Comm Tek is a company that believes it has core competencies that its competitors in
foreign markets lack and that it faces relatively weak pressures for local responsiveness and
cost reductions. Tel-Comm Tek is likely to adopt an international strategy.
Answer: TRUE
Diff: 2
Learning Outcome: Describe the process of selecting and developing an international
business strategy
Skill: Application
Objective: 5
AACSB: Application of knowledge

84) A multidomestic strategy makes the most sense for companies that see high pressures for
local responsiveness and low pressures for cost reductions.
Answer: TRUE
Diff: 2
Learning Outcome: Describe the process of selecting and developing an international
business strategy
Skill: Concept
Objective: 5
AACSB: Application of knowledge

85) Tel-Comm Tek is a company that sees the world as a single market, assuming that
consumer preferences and industry conditions do not vary much among countries. Tel-Comm
Tek is likely to adopt a multidomestic strategy.
Answer: FALSE
Diff: 2
Learning Outcome: Describe the process of selecting and developing an international
business strategy
Skill: Application
Objective: 5
AACSB: Application of knowledge
86) Mobile Technology is a company that sees the world as a single market, assuming that
consumer preferences and industry conditions do not vary much among countries. Mobile
Technology is likely to adopt a global strategy.
Answer: TRUE
Diff: 2
Learning Outcome: Describe the process of selecting and developing an international
business strategy
Skill: Application
Objective: 5
AACSB: Application of knowledge

87) A company that develops different capabilities and contributions from different countries
and shares them in integrated worldwide operations is using a transnational strategy.
Answer: TRUE
Diff: 1
Learning Outcome: Describe the process of selecting and developing an international
business strategy
Skill: Concept
Objective: 5
AACSB: Application of knowledge

88) Using a transnational strategy pushes a company to centralize some functions in optimal
locations, base some functions in national subsidiaries to ensure local responsiveness, and
develop wide-ranging communications among various units.
Answer: TRUE
Diff: 2
Learning Outcome: Describe the process of selecting and developing an international
business strategy
Skill: Concept
Objective: 5
AACSB: Application of knowledge

89) Organizational challenges are comparatively more difficult for the firm pursuing a
transnational strategy than for the firm pursuing an international strategy.
Answer: TRUE
Diff: 2
Learning Outcome: Describe the process of selecting and developing an international
business strategy
Skill: Concept
Objective: 5
AACSB: Analytical thinking
90) Micro-multinationals are firms that globalize immediately and enter countries with
numerous customers, productive workers, and attractive industries.
Answer: TRUE
Diff: 2
Learning Outcome: Describe the process of selecting and developing an international
business strategy
Skill: Concept
Objective: 5
AACSB: Application of knowledge

91) What is the Industry Organization (IO) paradigm? Discuss the underlying assumptions of
this model.
Answer: According to the Industry Organization (IO) paradigm, an industry's structure
shapes a firm's conduct, namely its strategic and tactical choices regarding research and
innovation, product strategy, plant investment, pricing behavior, and other value activities.
The assumptions underlying this model are that markets demonstrate perfect competition and
that firm performance is a function of its conduct. However, these two assumptions are not
always true. Markets are not always perfectly competitive, and some firms consistently
outperform industry averages. In this light, industry structure is not entirely deterministic of
firm performance. Rather, firm performance is influenced by the presence of bright,
motivated managers and their keen sense of innovative products and processes.
Diff: 3
Learning Outcome: Describe the process of selecting and developing an international
business strategy
Skill: Critical Thinking
Objective: 1
AACSB: Reflective thinking
92) Describe the two basic ways that companies can create value. What role does the value
chain play in these methods?
Answer: Companies create value in two basic ways—cost leadership and differentiation.
Cost leadership emphasizes high production volumes, low costs, and low prices to attract
customers. Firms that choose this strategy strive to be the low-cost producer in an industry for
a given level of quality. This strategy pushes a firm to sell its products either at average
industry prices to earn a profit higher than that of rivals or below average industry prices to
capture market share. A differentiation strategy spurs the company to provide a unique good
or service that rivals find hard, if not impossible, to match or copy. Firms that choose this
strategy aspire to develop products that offer unique attributes that they reason are highly
valued by customers and that customers perceive to be better than or sufficiently different
from products offered by other companies. The practices of successful MNEs advise
managers to interpret the activities the firm performs as elements of a value chain. The value
chain follows from the principle that "every firm is a collection of discrete activities
performed to do business that occur within the scope of the firm." It specifies a clear-cut
framework that lets managers deconstruct the abstraction of "create value" into a step-by-step
system. Modeling its sequence requires MNEs configure functions and coordinate processes
that move products from conception in R&D through sourcing materials, organizing
manufacturing, supervising logistics, applying marketing, and setting-up service options.
Diff: 3
Learning Outcome: Describe the process of selecting and developing an international
business strategy
Skill: Synthesis
Objective: 1, 2
AACSB: Analytical thinking

93) What is the difference between primary and secondary activities in the value chain?
Describe the functions of configuration and coordination in these value activities.
Answer: A value chain has primary and secondary activities. Primary activities are those
involved in the physical movement of raw materials and finished products, in the production
of goods and services, and in the marketing, sales, and subsequent services of the outputs of
the business. Secondary activities make up the managerial infrastructure of the firm that
supports carrying out the primary activities. The support activities include the processes and
systems installed to coordinate decisions and transactions among the various value activities.
Secondary activities apply to each primary activity. An MNEs competitiveness depends on
efficiently distributing value activities and effectively linking them. Distributing value
activities around the world is the matter of configuration. Linking them is the matter of
coordination. Configuration and coordination, flip sides of the value chain coin, are
intrinsically related.
Diff: 3
Learning Outcome: Describe the process of selecting and developing an international
business strategy
Skill: Synthesis
Objective: 2, 3
AACSB: Analytical thinking
94) What is configuration? Briefly list and discuss the factors that influence value chain
configuration.
Answer: Configuration is the way that managers arrange the activities of the value chain.
MNEs greatly improve their competitiveness and performance by configuring value activities
to capture potential location economies—namely, the economies that arise from performing a
value creation activity in the optimal location for that activity, given prevailing economic,
political, and cultural conditions. Therefore, several conditions shape how managers
configure value chains worldwide, most notably, cost factors, cluster effects, logistics,
digitization, economies of scale, and business environments.
a. Differences in cost factors, such as wage rates, worker productivity, inflation rates, and
government regulations, create significant variations in production costs from country to
country.
b. The cluster effect is when a particular industry gradually clusters more and more related
value creation effects in a specific location. Each economic cluster creates unique location
advantages that offer firms in that locale access to specialized resources that can dramatically
improve the potential for innovation.
c. Logistics is how companies obtain, produce, and exchange material and services in the
proper place and in proper quantities for the proper value activity.
d. Degree of digitization, or the degree to which an analog product can be converted into a
string of zeros and ones, influences how a company configures its value chain.
e. An economy of scale refers to the reductions in unit cost achieved by producing a large
volume of a product. Generally, economies of scale occur in industries with high capital costs
in which those costs can be distributed across a large number of units of production, thereby
resulting in lower per-unit costs.
f. The business environment is influenced by government policies, and these policies can
make a country more or less attractive for an MNE.
Diff: 3
Learning Outcome: Describe the process of selecting and developing an international
business strategy
Skill: Critical Thinking
Objective: 3
AACSB: Analytical thinking
95) What is coordination? Describe the factors that influence value chain coordination.
Answer: Coordination is the way that managers connect the discrete activities of the value
chain.
Several factors influence value chain coordination:
a. Operational obstacles: MNEs regularly run into problems when trying to get the various
links of their global value chain to deal with each other. Communication challenges
especially arise when trying to synchronize languages or deal with different time zones. Well-
planned coordination preempts these threats, letting workers worry less about what is
supposed to happen with material transfers and product delivery and worry more about
creating value.
b. National cultures: National cultures can also impose higher hurdles in coordinating a
transaction from one stage of the value chain with another. Units anchored in different
cultures may disagree over how much information they should share or who should take lead
responsibility. Coordination can then suffer from conflict.
c. Learning effects: Learning effects refer to cost savings that come from learning by doing.
Managers, for example, learn by recurrence how to transfer best practices from one country
to another, such as innovative ways to improve internal and external customer service.
Successfully transferred, an MNE can convert higher productivity into lower costs or higher
customer satisfaction into higher prices.
d. Subsidiary networks: The current culmination of globalization trends is a world marked
by real-time connectivity among the subsidiaries of an MNE. Subsidiaries around the world
can exchange information freely through communication networks. Moreover, there are an
astounding number of companies, including their affiliates, which engage in international
business. Skills, ideas, and technologies can be created anywhere within an MNE's global
network of subsidiaries. An increasingly vital task for managers, then, is to coordinate the
company's value chain so that it can leverage the competencies developed within any
subsidiary and apply them wherever they can create value within the firm's global network.
Diff: 3
Learning Outcome: Describe the process of selecting and developing an international
business strategy
Skill: Critical Thinking
Objective: 3
AACSB: Analytical thinking
96) Describe the pressures for local responsiveness that international companies face.
Answer: The two main pressures for local responsiveness that international companies face
are consumer divergence and host-government policies.
a. Consumer divergence: Some maintain that fundamental divergences in consumer tastes
and preferences across countries have and will continue to exert strong pressure for local
responsiveness. Many think that differences in consumer tastes and preferences across
countries emerge and endure due to several factors, including cultural predisposition,
historical legacy, emergent nationalism (i.e., "buy local" campaigns), and economic
prosperity. No matter the cause, proponents of customer divergence say the outcome is the
same: consumers prefer goods that are sensitive to their way of life.
b. Host-government policies: Host-country governments mandate policies that differ widely
from each other, causing variability in political, legal, and economic situations around the
world. The movement toward privatization, economic freedom, and deregulation has reduced
the variability among countries, but differences among countries remain. In light of the recent
economic crisis, these differences are again growing as countries take more steps to intervene
in an unstable marketplace. These differences push firms to determine how to best configure
and coordinate their value chain so that they provide the necessary degree of local
responsiveness without jeopardizing their capability to create value. Host governments also
have a range of aggressive tools to ensure that an MNE is locally responsive. These tools can
be broad policy directives, explicit threats or acts of trade protectionism, local content rules,
or simply national product standards that can be met only by local operations. Each policy
boosts the pressure on companies to make sure that part or all of its value chain can respond
to the local pressures.
Diff: 3
Learning Outcome: Describe the process of selecting and developing an international
business strategy
Skill: Critical Thinking
Objective: 4
AACSB: Reflective thinking
97) Discuss the characteristics of international, multidomestic, global, and transnational
strategies. Include situations and a specific example in which each strategy would be most
appropriate.
Answer:
a. International strategy: Companies adopt the international strategy when they aim to
leverage their core competencies by expanding opportunistically into foreign markets. The
international model relies on local subsidiaries in each country to administer business as
instructed by headquarters. Some subsidiaries may have latitude to adapt products to local
conditions as well as set up some light assembly operations or promotion programs. Still,
ultimate and absolute control resides with managers at headquarters who reason they know
best the basis and potential extension of the company's core competencies.
b. Multidomestic strategy: A strategy in which the company allows each of its foreign
country operations to act fairly independently, such as designing and producing a product or
service in France for the French market and in Japan for the Japanese market. The main
reason for adopting a multidomestic strategy is that in some cases, cultural, legal-political,
and economic conditions may dictate very different optimum operating practices from one
country to another.
c. Global strategy: A strategy in which a company integrates its operations located in
different countries. For example, it might design a product or service with a global market
segment in mind. Or it might depend on its operations in different countries to produce the
components used in the products and services. In this type of company, managers in the
company's home country essentially develop capabilities and make decisions to diffuse them
globally.
d. Transnational strategy: A strategy in which a company develops different capabilities and
contributions from different countries and shares them in integrated worldwide operations. In
essence, this is a hybrid of multidomestic and global strategies in that the company attempts
to gain the advantages of both. This strategy is ideal for companies that gain a great deal from
global integration and need a great deal of adaptation to local markets. Such industries as
pharmaceuticals and automobiles fall into this category.
Diff: 3
Learning Outcome: Describe the process of selecting and developing an international
business strategy
Skill: Critical Thinking
Objective: 5
AACSB: Reflective thinking
98) In a brief essay, discuss the strategy used by Zara, the firm described in the opening case.
Also discuss the features of Zara's value chain.
Answer: Zara realized that offering standardized fashion styles at reasonable prices
neutralized stubborn local preferences. Its global network, supported by state-of-the-art
logistics, gave customers worldwide real-time access to the newest, coolest fashion trends.
Global markets let Zara leverage its global scale investment in design, manufacturing,
distribution, and retail activities. The resulting efficiencies, in turn, supported making high-
quality, low-cost products that, by offering compelling value, re-powered the cycle. Zara does
not adapt products to a particular country's preferences. The convergence of fashion and taste
across national boundaries endorses management's bias toward standardization. However,
some product designs cater to physical, cultural, or climate differences—smaller sizes in
Japan, special women's clothing in Arab countries, and different seasonal weights in South
America. Still, Zara standardized about 85 percent of its designs for the global market. The
firm has a concentrated value chain with its product design, manufacturing, and logistics
activities located in Spain at the firm's headquarters.
Diff: 3
Learning Outcome: Describe the process of selecting and developing an international
business strategy
Skill: Synthesis
Objective: 2, 5
AACSB: Reflective thinking

99) What is the relationship between the five-forces model and a firm's international business
strategy? Provide examples to illustrate your answer.
Answer: The five-forces model maps the relationship among companies within an industry,
highlighting how competitors, new entrants, suppliers, buyers, and substitute products affect
profitability. It holds that firm performance is a function of its strategy, which is determined
by industry factors that shape the corresponding pattern of competition. For example, an
industry with few entry barriers, lots of accessible buyers, and an expanding supply of low-
cost, powerful technologies tends to have many firms competing for profits. Think of, for
example, cell phones, e-commerce, financial services, or entertainment. Conversely, an
industry with high entry barriers, steep capital requirements, and extreme research and
development standards tends to have few firms competing for profits. Think of, for example,
the pharmaceutical, energy, aircraft, or automobile industries. In both types, the prevailing
industry structure shapes an MNE's choices regarding research and innovation, product
strategy, plant investment, pricing behavior, among others. These choices ultimately
influence its performance.
Diff: 3
Learning Outcome: Describe the process of selecting and developing an international
business strategy
Skill: Synthesis
Objective: 1, 5
AACSB: Analytical thinking
100) What are core competencies? How do firms that adopt an international strategy utilize
their core competencies? How do firms that adopt a transnational strategy utilize their core
competencies?
Answer: A core competency is the special outlook, skill, capability, or technology that runs
through the firm's operations, threading disconnected activities into an integrated value chain.
The competitive imperative of necessity of leveraging core competencies throughout the
value chain intensifies the importance of skillfully coordinating activities. Companies adopt
an international strategy when they leverage core competencies internationally in an industry
marked by low pressure for global integration and local responsiveness. The transnational
strategy holds that in today's environment of interconnected consumers, industries, and
markets, an MNE must configure a value chain that exploits location economies as well as
coordinates value activities in order to leverage core competencies while simultaneously
reconciling global and local pressures.
Diff: 3
Learning Outcome: Describe the process of selecting and developing an international
business strategy
Skill: Synthesis
Objective: 3, 5
AACSB: Analytical thinking
International Business: Environments and Operations, 15e (Daniels et al.)
Chapter 13 Country Evaluation and Selection

1) Comparing countries in international business is LEAST useful for determining the


________.
A) best location for sales and production
B) sequence of entering different countries
C) amount of resources to allocate in each country
D) selection of which managers to send to which countries
Answer: D
Diff: 2
Learning Outcome: Describe how global production and logistics decisions are made
Skill: Concept
Objective: 1
AACSB: Application of knowledge

2) International managers most likely need to understand how to evaluate international


geographic alternatives because ________.
A) they usually have a surplus of resources and need to take advantage of all opportunities
B) many regional trading groups prohibit companies from outside of the trading group from
manufacturing in more than one member country
C) the commitment of resources to one locale may require forgoing projects in other locales
D) decreased worldwide transportation costs and increased trade liberalization now allow
companies to serve worldwide markets from a single production location
Answer: C
Diff: 2
Learning Outcome: Describe how global production and logistics decisions are made
Skill: Application
Objective: 1
AACSB: Analytical thinking

3) Executives at Wilson Enterprises need to determine how to leverage and improve the
firm's existing competencies on a global basis. What are the two most basic questions that
they must answer?
A) Which markets should we serve and where should production be located to serve those
markets?
B) What are the short-term competitive advantages of the project and what is the return on
investment?
C) What is the total investment required and what are the managerial resources needed to
supervise the investment?
D) What is the availability of land and what is the cost of labor?
Answer: A
Diff: 3
Learning Outcome: Describe how global production and logistics decisions are made
Skill: Application
Objective: 1
AACSB: Application of knowledge

4) A company's overall geographic strategy should be flexible enough to ________.


A) implement concentration strategies instead of diversification strategies
B) respond to new opportunities and withdraw from less profitable ones
C) import from anywhere in the world to a single production location
D) export anywhere in the world from a single production location
Answer: B
Diff: 2
Learning Outcome: Describe how global production and logistics decisions are made
Skill: Concept
Objective: 1
AACSB: Analytical thinking

5) Elison Enterprises is planning international geographic expansion. A manager at Elison


has been given the task of scanning for locations primarily to ________.
A) reduce the number of options available to a manageable number for further detailed
analysis
B) assure the compatibility between the mode of corporate operation and the country
C) assure that all countries within a region have similar investment climates
D) decide whether to use a concentration or a diversification strategy
Answer: A
Diff: 2
Learning Outcome: Describe how global production and logistics decisions are made
Skill: Application
Objective: 2
AACSB: Application of knowledge

6) Which of the following most accurately compares the techniques of scanning versus
detailed analysis of countries?
A) Scanning is used for planning and detailed analysis is used for control.
B) Detailed analysis is used to consider countries overlooked in the scanning process.
C) Scanning considers a large number of countries so that only the most promising ones
undergo a detailed analysis.
D) Scanning compares one country to another, whereas detailed analysis compares regions
within a single country.
Answer: C
Diff: 3
Learning Outcome: Describe how global production and logistics decisions are made
Skill: Concept
Objective: 2
AACSB: Analytical thinking
7) Opal Computers is considering international production expansion. After scanning to
decide on a few countries to consider more closely, Opal managers will most likely need to
________.
A) identify firms with which to form joint ventures
B) add some more countries for closer consideration
C) travel to the locations to analyze and collect specific data
D) make final decisions by expanding in locations near their rivals
Answer: C
Diff: 2
Learning Outcome: Describe how global production and logistics decisions are made
Skill: Concept
Objective: 2
AACSB: Analytical thinking

8) Escalation of commitment is best described as the ________.


A) strategy of first entering a country on a small scale
B) process of entering a country because "everyone else is going there"
C) expectation of a higher return in more politically risky environments
D) increased likelihood of investing in a country because of having spent considerable time
and money in examining it
Answer: D
Diff: 2
Learning Outcome: Describe how global production and logistics decisions are made
Skill: Concept
Objective: 2
AACSB: Application of knowledge

9) Sales expansion is probably the most important variable in determining international


location decisions. This statement is most likely based on the assumption that ________.
A) consumer demand exceeds supply
B) increased sales will lead to more profits
C) the company will have a first-mover advantage
D) raw materials are available in the country targeted for sales
Answer: B
Diff: 2
Learning Outcome: Describe how global production and logistics decisions are made
Skill: Concept
Objective: 3
AACSB: Analytical thinking
10) Dawson Manufacturing produces and sells DVD players and is planning to expand sales
internationally. Dawson has narrowed down the list of potential countries to India and
Guatemala. A Dawson manager has the task of obtaining data regarding the number of DVD
players sold annually in India and Guatemala. If unable to locate this information, she might
most likely estimate the sales potential of these two countries by ________.
A) determining average wages
B) calculating future inflation rates
C) reviewing the countries' dependence on steel imports
D) examining the sales history of flat-screen televisions
Answer: D
Diff: 2
Learning Outcome: Describe how global production and logistics decisions are made
Skill: Application
Objective: 3
AACSB: Application of knowledge

11) Gucci, a maker of luxury fashion and leather goods, plans to expand its sales market. The
firm needs to compare countries for the market potential of its products. Which of the
following is the best indicator for Gucci to use?
A) per capita income in each country
B) population size of each country
C) the number of millionaires in each country
D) gross domestic product for each country
Answer: C
Diff: 2
Learning Outcome: Describe how global production and logistics decisions are made
Skill: Application
Objective: 3
AACSB: Analytical thinking

12) When examining economic and demographic variables to compare countries' sales
potential for your product, you should also consider all the following EXCEPT which one?
A) If countries depend heavily on the import of raw materials, what is the price of elasticity
for the demand?
B) Consumers in some countries may more conveniently substitute certain products than
consumers in some other countries.
C) Consumers in developing countries may leapfrog technologies by first purchasing the
latest products.
D) Trading blocs may enhance sales potential above what is indicated in individual country
figures.
Answer: A
Diff: 3
Learning Outcome: Describe how global production and logistics decisions are made
Skill: Application
Objective: 3
AACSB: Application of knowledge

13) Which of the following is most likely a true statement about companies' acquisition of
resources/assets abroad?
A) Regardless of industry, cheap labor is the most sought after resource.
B) Resource availability limits a firm's production location choices.
C) Regardless of industry, raw materials are the most sought after resource.
D) Risks are higher for resource-seeking than for sales-seeking foreign operations.
Answer: B
Diff: 3
Learning Outcome: Describe how global production and logistics decisions are made
Skill: Concept
Objective: 3
AACSB: Analytical thinking

14) The ability to compare production costs among countries in an effort to determine where
to locate production is significantly hampered by all of the following EXCEPT ________.
A) the number of ways the same product can be made
B) restrictions on the international flow of data
C) the ways that laws may be enforced
D) future costs from exchange rate changes
Answer: A
Diff: 2
Learning Outcome: Describe how global production and logistics decisions are made
Skill: Concept
Objective: 3
AACSB: Application of knowledge

15) In which of the following situations would tax rate differences among countries be most
important for deciding where to place an investment?
A) Companies find advantages in being located near specialized private and public
institutions.
B) Companies must compare the benefits of labor- versus capital-intensive production.
C) Companies want to serve an entire region within a regional trading bloc.
D) Companies must deal with difficult start-up regulations.
Answer: C
Diff: 3
Learning Outcome: Describe how global production and logistics decisions are made
Skill: Concept
Objective: 3
AACSB: Analytical thinking
16) Labor cost advantages gained by moving into a country with low wages may be short-
lived because ________.
A) transport costs go up to cancel out the cost savings
B) tax increases cancel out all labor cost differentials
C) rivals adopt capital-intensive production methods
D) competitors follow leaders into low-wage areas
Answer: D
Diff: 2
Learning Outcome: Describe how global production and logistics decisions are made
Skill: Concept
Objective: 3
AACSB: Reflective thinking

17) Which of the following statements is NOT true about risk as it affects companies' choice
of locations for foreign operations?
A) Companies and their managers differ in their perception of what is risky.
B) One company's risk may be another company's opportunity.
C) There are means to reduce risk other than avoiding locations.
D) Companies choose the cheapest location regardless of risks.
Answer: D
Diff: 3
Learning Outcome: Describe how differences in political economy influence economic
development
Skill: Concept
Objective: 3
AACSB: Analytical thinking

18) A company's operations are most likely to be taken over by a host government when
________.
A) the operations are relatively small and, thus, unlikely to incur the wrath of the company's
home government
B) the operations are substantial and have a widespread effect on the country because of the
company's size
C) the host country becomes involved in a regional war
D) the firm produces discretionary rather than essential products
Answer: B
Diff: 3
Learning Outcome: Describe how differences in political economy influence economic
development
Skill: Concept
Objective: 3
AACSB: Analytical thinking
19) In terms of political risk, it is most accurate to state that high risk ________.
A) affects all geographic regions of a country equally
B) affects all foreign companies in the same manner
C) if avoided, may lead to higher competitive risk
D) triggers government turnovers
Answer: C
Diff: 2
Learning Outcome: Describe how differences in political economy influence economic
development
Skill: Concept
Objective: 3
AACSB: Analytical thinking

20) Fidelity Manufacturing is considering expanding its operations into the Philippines. A
manager at Fidelity has the task of predicting political risk in the Philippines. Which of the
following approaches should the manager LEAST use to accomplish the task?
A) analyzing the market share of competitors in the country
B) analyzing the country's past political patterns and trends
C) seeking and analyzing opinions of influential people in the country
D) examining social and economic conditions within the country
Answer: A
Diff: 3
Learning Outcome: Describe how differences in political economy influence economic
development
Skill: Application
Objective: 3
AACSB: Analytical thinking

21) The concept of liquidity preference in international operations refers to ________.


A) a company's willingness to accept a lower rate of return on investments in countries where
it can more easily sell them and convert the proceeds at a favorable rate
B) a company's willingness to accept lower rates of return in poor countries that really need
the investments
C) management's need to maintain sufficient funds, preferably in local currency, in each
country of operation to ensure meeting daily cash needs
D) investors' preference for foreign stocks over foreign bonds because of the larger market
for them
Answer: A
Diff: 2
Learning Outcome: Describe how global production and logistics decisions are made
Skill: Concept
Objective: 3
AACSB: Application of knowledge
22) Risks to companies from natural disasters and communicable diseases are ________.
A) evenly distributed around the world
B) more complicated today because of publicity
C) a minor issue to global firms because of insurance
D) most prevalent in the poorest countries of the world
Answer: D
Diff: 2
Learning Outcome: Describe how global production and logistics decisions are made
Skill: Concept
Objective: 3
AACSB: Diverse and multicultural work environments

23) U.S. companies generally put earlier and more sales-seeking emphasis on countries
________.
A) with the largest economies
B) with regional trading blocs and high tariffs
C) where governments give operating incentives
D) where operating conditions seem similar to those at home
Answer: D
Diff: 2
Learning Outcome: Describe how global production and logistics decisions are made
Skill: Concept
Objective: 3
AACSB: Analytical thinking

24) The lower survival rate of foreign companies in comparison to local firms for many years
after they begin operations is known as ________.
A) ethnocentric reaction
B) polycentric reaction
C) liability of foreignness
D) most-favored-nation behavior
Answer: C
Diff: 1
Learning Outcome: Describe how global production and logistics decisions are made
Skill: Concept
Objective: 3
AACSB: Application of knowledge
25) Which of the following best explains why U.S. firms typically place earlier and greater
emphasis on expansion into Canada and the U.K.?
A) most significant sales opportunities
B) similarities in culture and legal systems
C) availability of necessary natural resources
D) government incentives for allied nations
Answer: B
Diff: 2
Learning Outcome: Describe how global production and logistics decisions are made
Skill: Concept
Objective: 3
AACSB: Analytical thinking

26) Which of the following best explains Blockbuster's failed expansion into Germany?
A) laws limiting hours of operation
B) lack of public interest in films
C) inadequate tax incentives
D) communication problems
Answer: A
Diff: 2
Learning Outcome: Describe how global production and logistics decisions are made
Skill: Concept
Objective: 3
AACSB: Analytical thinking

27) The crowding of a foreign market to prevent competitors' advantages is known as


________.
A) oligopolistic reaction
B) concentration strategy
C) liability of foreignness
D) a harvesting strategy
Answer: A
Diff: 1
Learning Outcome: Describe how global production and logistics decisions are made
Skill: Concept
Objective: 3
AACSB: Application of knowledge
28) Companies are more likely to gain advantages by locating near competitors for all the
following reasons EXCEPT to ________.
A) take advantage of competitors' research to pick an ideal location
B) attract multiple suppliers and personnel with specialized skills
C) agree with competitors on production limitations
D) attract buyers who want to compare suppliers
Answer: C
Diff: 2
Learning Outcome: Describe how global production and logistics decisions are made
Skill: Concept
Objective: 3
AACSB: Analytical thinking

29) An example of a first-mover advantage in international operations is ________.


A) gaining economies of scale at a lower output level than competitors
B) increasing sales response functions and customer service
C) using a small country for market tests prior to entering a large country
D) lining up the best suppliers and distributors before competitors enter the market
Answer: D
Diff: 2
Learning Outcome: Describe how global production and logistics decisions are made
Skill: Concept
Objective: 3
AACSB: Application of knowledge

30) Which of the following describes a company's strategy of moving first to those countries
where local competitors are most likely to catch up to the firm's innovative advantage?
A) lead country strategy
B) imitation lag strategy
C) oligopolistic reaction strategy
D) liability of foreignness strategy
Answer: B
Diff: 1
Learning Outcome: Describe how global production and logistics decisions are made
Skill: Concept
Objective: 3
AACSB: Application of knowledge
31) A manager has the task of collecting and analyzing data that will help the firm decide
where to locate its international operations. Which of the following best describes how the
manager should handle this task?
A) conduct extensive research, regardless of the expense, in order to avoid costly mistakes
B) compare the costs of data collection with the probable payoff for the firm in order to
budget and schedule the collection
C) continue data gathering until all data have been collected, regardless of how long this
takes
D) focus all data collection on governmental resources because they have the highest
reliability
Answer: B
Diff: 3
Learning Outcome: Describe how global production and logistics decisions are made
Skill: Application
Objective: 4
AACSB: Analytical thinking

32) Which of the following is the LEAST likely reason for inaccuracies in published
governmental data?
A) translation errors from the host country language
B) limitations of government resources and finances
C) purposeful publication of misleading information
D) false information provided to data collectors
Answer: A
Diff: 2
Learning Outcome: Describe how global production and logistics decisions are made
Skill: Concept
Objective: 4
AACSB: Reflective thinking

33) Which of the following is the LEAST likely reason that inaccuracies appear in published
information about countries?
A) inclusion of both legal and illegal economic activities
B) inclusion of both market and non-market economic activities
C) poor methodology used in data collection
D) use of different translation software
Answer: C
Diff: 2
Learning Outcome: Describe how global production and logistics decisions are made
Skill: Concept
Objective: 4
AACSB: Reflective thinking
34) Which of the following is generally the most costly information source for companies?
A) individualized reports
B) reports from international agencies
C) reports from government agencies
D) published reports by accounting firms
Answer: A
Diff: 2
Learning Outcome: Describe how global production and logistics decisions are made
Skill: Concept
Objective: 4
AACSB: Reflective thinking

35) Top executives at Jordan, a U.S. consulting firm, are debating whether or not to expand
into a country with a great deal of violence by staffing mostly with U.S. personnel. A vice
president argues that Jordan should forego sending its employees there because of the high
risk for them of kidnappings in the region. Which of the following statements best supports
the vice president's position?
A) There is a high correlation between violence and life-threatening natural disasters.
B) Violence is a harbinger of additional risks that affect operations negatively.
C) Local personnel are immune from violence and are capable of filling positions.
D) The ability to evacuate people when necessary is much slower than it was in the past.
Answer: B
Diff: 3
Learning Outcome: Discuss the role of ethics and social responsibility in international
business
Skill: Critical Thinking
Objective: 4
AACSB: Analytical thinking

36) Top executives at Jordan, a U.S. consulting firm, are debating whether or not to expand
operations into a country with a great deal of violence by staffing mostly with U.S. personnel.
A vice president argues that Jordan should send its employees there. Which of the following
statements LEAST supports the vice president's position?
A) Jordan can evacuate personnel more quickly than in earlier eras in case of a real
emergency.
B) It is hard to identify countries without a possibility for violence.
C) Operating costs are lower in violent areas.
D) Jordan's industry does not allow the firm the luxury of avoiding high risk locations.
Answer: C
Diff: 3
Learning Outcome: Discuss the role of ethics and social responsibility in international
business
Skill: Critical Thinking
Objective: 4
AACSB: Analytical thinking
37) Grids are a useful method of comparing countries for international business expansion
because they ________.
A) generally show how countries will perform in the future
B) show risk on one axis and opportunity on another
C) set minimum scores for proceeding further
D) highlight first-mover advantages
Answer: C
Diff: 2
Learning Outcome: Describe how global production and logistics decisions are made
Skill: Concept
Objective: 5
AACSB: Application of knowledge

38) A manager needs to prepare a grid to compare countries for location of the firm's
international operations. It would be most useful for the manager to ________.
A) prepare an opportunity analysis in-house, but out-source the risk analysis
B) have agents within each country supply governmental data
C) prepare the risk analysis in-house, but out-source the opportunity analysis
D) use a team made up of people from different functions within the company
Answer: D
Diff: 2
Learning Outcome: Describe how global production and logistics decisions are made
Skill: Application
Objective: 5
AACSB: Analytical thinking

39) Which of the following best describes the purpose of using of an opportunity-risk matrix
for comparing countries?
A) narrow alternatives so decision makers can make a detailed analysis of the strongest
candidates
B) eliminate countries that have specific unacceptable conditions
C) determine whether to use a concentration versus diversification strategy for international
expansion
D) estimate where competitors are most likely to globalize
Answer: A
Diff: 2
Learning Outcome: Describe how global production and logistics decisions are made
Skill: Concept
Objective: 5
AACSB: Analytical thinking
40) The major use of the matrix as a tool in international location strategy is to ________.
A) pinpoint acceptable and unacceptable characteristics of countries
B) indicate the relative placement of countries in terms of attributes
C) rank countries on the basis of expected investment return
D) show the degree of certainty for projected returns
Answer: B
Diff: 2
Learning Outcome: Describe how global production and logistics decisions are made
Skill: Concept
Objective: 5
AACSB: Application of knowledge

41) In a concentration strategy of foreign expansion, a company would go to ________.


A) many countries very rapidly, and then build up slowly in each
B) a foreign country with one product and not sell other products in that country until a target
market share is reached
C) a reporting system that measures performance on a regional rather than a country-by-
country basis
D) one or a few foreign countries and build a strong involvement there before going to other
countries
Answer: D
Diff: 2
Learning Outcome: Describe how global production and logistics decisions are made
Skill: Concept
Objective: 6
AACSB: Application of knowledge

42) In a diversification strategy for international expansion, a company would move


________.
A) rapidly into many foreign countries, and then gradually increase its presence in those
countries
B) rapidly into a few foreign countries with many of its products and most of its resources
C) into one foreign country and fully expand its product lines in that country before moving
to another country
D) quickly into a regional foreign market but build up its resources in only a few of the
countries in the region
Answer: A
Diff: 2
Learning Outcome: Describe how global production and logistics decisions are made
Skill: Concept
Objective: 6
AACSB: Analytical thinking
43) A company should probably use a concentration strategy for international expansion
when there are ________.
A) high needs for product adaptation and low growth in each market
B) short competitive lead times and low spillover effects
C) high growth rate and long competitive lead times
D) low sales stability and short competitive lead times
Answer: C
Diff: 2
Learning Outcome: Summarize the main entry strategies and modes that businesses use to
enter into foreign markets
Skill: Application
Objective: 5
AACSB: Analytical thinking

44) The decision-making process for a company's reinvestment choices is often different
from those for new investment choices because ________.
A) internal rate of return and other financial measurement criteria are more difficult to
compile and analyze on existing operations, given currency translation distortions
B) failure to support an existing investment may jeopardize the firm's operations and
competitiveness in that country
C) most of the net value of foreign investment comes from new international capital transfers
rather than from reinvestment of earnings abroad
D) corporate management feels that country managers are best able to make divestment
decisions
Answer: B
Diff: 3
Learning Outcome: Describe how global production and logistics decisions are made
Skill: Concept
Objective: 5
AACSB: Reflective thinking

45) Which of the following is NOT true about the harvesting or divestment of foreign
operations?
A) One of the motives is to use resources where the performance prospects are better.
B) Companies can harvest or divest by selling existing facilities.
C) Closing a facility can be difficult because of governmental performance contracts.
D) Companies have tended to divest too soon, rather than working to improve performance.
Answer: D
Diff: 3
Learning Outcome: Describe how global production and logistics decisions are made
Skill: Concept
Objective: 5
AACSB: Application of knowledge
46) Which of the following best explains why foreign subsidiary managers are often reluctant
to propose divestments in the countries where they are working?
A) They are afraid of proposing the elimination of their jobs.
B) They are usually poorly trained in how to sell units or how to close them down.
C) They are too nationalistic to examine political risk objectively.
D) Many are in countries where the cultural attribute of power-distance is very high.
Answer: A
Diff: 3
Learning Outcome: Describe how global production and logistics decisions are made
Skill: Concept
Objective: 5
AACSB: Application of knowledge

47) The origin of investment proposals differs from the origin of divestment proposals in that
the divestment proposals are more likely to come from ________.
A) subsidiary management
B) outside the organization
C) higher up in the organization
D) line personnel as opposed to staff personnel
Answer: C
Diff: 2
Learning Outcome: Describe how global production and logistics decisions are made
Skill: Concept
Objective: 5
AACSB: Reflective thinking

48) A go/no-go decision means ________.


A) an individual project decision is based on whether the project meets threshold criteria
B) projects are ranked and approved from the top of the list down until available resources
are exhausted
C) management reviews existing information and decides whether additional individualized
feasibility studies are warranted
D) projects are approved or disapproved based on the potential ease of divestment
Answer: A
Diff: 2
Learning Outcome: Describe how global production and logistics decisions are made
Skill: Concept
Objective: 7
AACSB: Application of knowledge
49) Instead of comparing different proposals involving foreign operations, companies often
make decisions by looking at proposals one at a time. All of the following are reasons for this
behavior EXCEPT which one?
A) Companies need to respond quickly to opportunities.
B) Defensive decisions typically need to be made rapidly.
C) A lack of comparable data on different countries renders comparison impossible.
D) Conclusion of different proposals or studies does not usually happen simultaneously.
Answer: C
Diff: 2
Learning Outcome: Describe how global production and logistics decisions are made
Skill: Concept
Objective: 7
AACSB: Analytical thinking

50) Assume Company A receives a proposal from Company B to be a joint venture partner
abroad. Company A is most likely to make its decision based on ________.
A) an opportunity-risk matrix
B) a go/no-go basis
C) a global matrix comparison
D) an oligopolistic reaction
Answer: B
Diff: 2
Learning Outcome: Describe how global production and logistics decisions are made
Skill: Application
Objective: 7
AACSB: Reflective thinking

51) Which of the following reasons most compels companies to make location decisions on
one international opportunity at a time rather than comparing among two or more?
A) The lack of comparability in data among countries renders comparison unfeasible.
B) The information on some countries is so unreliable that companies must deal with these
countries separately.
C) Decisions are made by teams, and it is usually not feasible to give so many people time
away from their usual duties to examine multiple proposals.
D) If an important customer develops opportunities in a foreign country, a company may
have little alternative except to follow that customer's lead.
Answer: D
Diff: 3
Learning Outcome: Describe how global production and logistics decisions are made
Skill: Concept
Objective: 7
AACSB: Analytical thinking
52) Demographers project that the share (percentage of population) of what we now consider
the working-age population in developed countries will decrease up to the year 2050. Which
of the following is the most likely result of this trend?
A) an increase in foreign exchange among trading blocs
B) an increase in FDI provided by developed economies
C) a higher percentage in per capita GDP in today's developing economies than in today's
developed economies
D) a higher percentage in per capita GDP in today's developed economies than in today's
developing economies
Answer: C
Diff: 2
Learning Outcome: Describe how global production and logistics decisions are made
Skill: Concept
Objective: 7
AACSB: Reflective thinking

53) Which of the following is true about projected demographic changes up to the year 2050
that could affect future production and sales locations?
A) The share of the working population should rise in developed countries and fall in
developing countries.
B) The growth in per capita GDP should be higher in today's developing economies than in
today's developed economies.
C) The percentage of the world population living in today's developed countries is expected
to increase.
D) The population should fall in sub-Saharan Africa.
Answer: B
Diff: 3
Learning Outcome: Describe how global production and logistics decisions are made
Skill: Concept
Objective: 7
AACSB: Reflective thinking

54) We now have technology to allow people to communicate globally without traveling as
much. Leading researchers on urbanization and planning suggest that the most likely
consequence of this is ________.
A) a decrease in international airline travel
B) a decreased need for immigration restrictions
C) a smaller number of retirees living in urban areas
D) a greater number of self-motivated workers e-mailing and teleconferencing with
colleagues
Answer: D
Diff: 3
Learning Outcome: Describe how global production and logistics decisions are made
Skill: Concept
Objective: 7
AACSB: Analytical thinking
55) All of the following have been predicted to occur in the future as the result of advances in
global communications EXCEPT which one?
A) In spite of being able to work anywhere, people will choose to live primarily where their
employers are headquartered.
B) The brightest minds will work more at home but will still need face-to-face interaction
with their colleagues.
C) People will be drawn to live in the same places that attract people as tourists.
D) People who are both highly motivated and highly creative will continue to be attracted to
interact with people like themselves.
Answer: A
Diff: 2
Learning Outcome: Describe how global production and logistics decisions are made
Skill: Concept
Objective: 7
AACSB: Analytical thinking

56) Carrefour has been more successful than Walmart in Europe, whereas Walmart has been
more successful than Carrefour in the United States. What is the most likely reason for these
results?
A) first-mover advantages
B) nationalistic preferences of consumers
C) lack of knowing how to adapt products
D) increased exporting fees and transportation costs
Answer: A
Diff: 2
Learning Outcome: Describe how global production and logistics decisions are made
Skill: Concept
Objective: 3
AACSB: Application of knowledge

57) Carrefour expanded internationally by first ________.


A) entering adjacent countries
B) licensing its name to other companies
C) buying companies in foreign countries
D) entering many countries simultaneously with small commitments in each
Answer: A
Diff: 2
Learning Outcome: Describe how global production and logistics decisions are made
Skill: Concept
Objective: 6
AACSB: Application of knowledge
58) Which of the following best explains why Burger King has developed such a strong
presence in many of the small countries of Latin America and the Caribbean?
A) These business environments allowed Burger King to take advantage of economies of
scale.
B) These countries are close to a Burger King's headquarters.
C) These countries offered greater mobility of funds than countries in the European Union.
D) Unlike the BRIC countries, these business environments did not require escalation of
commitment.
Answer: B
Diff: 3
Learning Outcome: Describe how global production and logistics decisions are made
Skill: Concept
Objective: 3
AACSB: Application of knowledge

59) Because many regional trading groups prohibit companies from producing in more than
one member country, companies need to understand how to evaluate international geographic
alternatives.
Answer: FALSE
Diff: 2
Learning Outcome: Describe how global production and logistics decisions are made
Skill: Concept
Objective: 1
AACSB: Analytical thinking

60) Committing resources to one country usually means forgoing or delaying projects in
others.
Answer: TRUE
Diff: 1
Learning Outcome: Describe how global production and logistics decisions are made
Skill: Concept
Objective: 1
AACSB: Application of knowledge

61) When planning international geographic expansion, decision makers use scanning to
reduce the number of options available to a manageable number for further detailed analysis.
Answer: TRUE
Diff: 1
Learning Outcome: Describe how global production and logistics decisions are made
Skill: Concept
Objective: 2
62) Good scanning helps managers avoid the need to make a detailed analysis of countries
when deciding where to operate.
Answer: FALSE
Diff: 1
Learning Outcome: Describe how global production and logistics decisions are made
Skill: Concept
Objective: 2
AACSB: Reflective thinking

63) Sales potential is probably the most important variable in determining international
location decisions because consumer demand exceeds supply.
Answer: FALSE
Diff: 1
Learning Outcome: Describe how global production and logistics decisions are made
Skill: Concept
Objective: 2
AACSB: Reflective thinking

64) When comparing economic and demographic variables among countries, one should
consider that consumers in developing countries do not necessarily follow the same historical
patterns as those in more developed countries.
Answer: TRUE
Diff: 2
Learning Outcome: Describe how global production and logistics decisions are made
Skill: Concept
Objective: 2
AACSB: Diverse and multicultural work environments

65) Although capital intensity is growing in most industries, labor compensation remains a
significant cost for most companies.
Answer: TRUE
Diff: 2
Learning Outcome: Describe how global production and logistics decisions are made
Skill: Concept
Objective: 2
AACSB: Application of knowledge

66) Labor cost advantages gained by moving into a country with low wages may be short-
lived because tax increases cancel out the low-wage advantages.
Answer: FALSE
Diff: 2
Learning Outcome: Describe how global production and logistics decisions are made
Skill: Concept
Objective: 2
AACSB: Application of knowledge
67) Governments that conduct takeovers of foreign companies rarely make formal
declarations of their intent to take over in advance of the action.
Answer: FALSE
Diff: 2
Learning Outcome: Describe how global production and logistics decisions are made
Skill: Concept
Objective: 3
AACSB: Application of knowledge

68) In assessing political risk, the observation of past patterns is problematic because
situations may change for better or worse.
Answer: TRUE
Diff: 1
Learning Outcome: Describe how global production and logistics decisions are made
Skill: Concept
Objective: 3
AACSB: Analytical thinking

69) Companies are usually willing to accept a lower rate of return on their investments in
countries where they can more easily sell those investments and convert the proceeds at a
favorable rate.
Answer: TRUE
Diff: 2
Learning Outcome: Describe how global production and logistics decisions are made
Skill: Concept
Objective: 3
AACSB: Reflective thinking

70) Losses to companies from natural disasters are much less risky than losses from operating
in violent areas.
Answer: FALSE
Diff: 2
Learning Outcome: Describe how global production and logistics decisions are made
Skill: Concept
Objective: 3
AACSB: Reflective thinking

71) U.S. companies generally put earlier and more emphasis on countries where they
perceive it's easier to operate.
Answer: TRUE
Diff: 1
Learning Outcome: Describe how global production and logistics decisions are made
Skill: Concept
Objective: 3
AACSB: Analytical thinking
72) Liability of foreignness refers to the situation in which a government has more stringent
legal operating regulations on foreigners and foreign companies than on its own citizens and
companies.
Answer: FALSE
Diff: 1
Learning Outcome: Describe how differences in political economy influence economic
development
Skill: Concept
Objective: 3
AACSB: Analytical thinking

73) An advantage of locating operations where there are many competitors is that the cluster
of competing firms attracts multiple suppliers and specialized personnel.
Answer: TRUE
Diff: 2
Learning Outcome: Describe how global production and logistics decisions are made
Skill: Concept
Objective: 3
AACSB: Analytical thinking

74) A company can best benefit from a first-mover advantage by moving into a small
country, before entering a much larger country.
Answer: FALSE
Diff: 2
Learning Outcome: Describe how global production and logistics decisions are made
Skill: Application
Objective: 3
AACSB: Analytical thinking

75) Published government data is most often inaccurate because of translation errors from
other countries' languages.
Answer: FALSE
Diff: 1
Learning Outcome: Describe how global production and logistics decisions are made
Skill: Concept
Objective: 4
AACSB: Reflective thinking

76) Comparability of economic information among countries is hampered by countries' use of


different definitions for similar terms.
Answer: TRUE
Diff: 1
Learning Outcome: Describe how global production and logistics decisions are made
Skill: Concept
Objective: 4
AACSB: Application of knowledge

77) When choosing international operating locations, companies should outsource the
preparation of grids or matrices to experts rather than preparing them with their own
personnel.
Answer: FALSE
Diff: 2
Learning Outcome: Describe how global production and logistics decisions are made
Skill: Concept
Objective: 5
AACSB: Reflective thinking

78) Unlike grids, matrices do not require managers to determine weights for factors that
indicate risk.
Answer: FALSE
Diff: 2
Learning Outcome: Describe how global production and logistics decisions are made
Skill: Concept
Objective: 5
AACSB: Application of knowledge

79) In a concentration strategy for international expansion, a company goes first to one or a
few countries and builds up fast there before going to other countries.
Answer: TRUE
Diff: 1
Learning Outcome: Describe how global production and logistics decisions are made
Skill: Concept
Objective: 6
AACSB: Application of knowledge

80) The more a company needs to alter its products and ways of doing business to be
successful abroad, the more it should rely on a diversification strategy for entering foreign
markets.
Answer: FALSE
Diff: 2
Learning Outcome: Describe how global production and logistics decisions are made
Skill: Concept
Objective: 6
AACSB: Reflective thinking

81) Headquarters management often feels that people within an established operation are the
best judge of the operation's investment needs.
Answer: TRUE
Diff: 2
Learning Outcome: Describe how global production and logistics decisions are made
Skill: Concept
Objective: 6
AACSB: Reflective thinking
82) Companies have tended to wait too long to divest poorly performing foreign facilities,
trying instead to improve performance through expensive means.
Answer: TRUE
Diff: 2
Learning Outcome: Describe how global production and logistics decisions are made
Skill: Concept
Objective: 6
AACSB: Application of knowledge

83) A go/no-go decision for foreign expansion means that management reviews existing
information and decides whether more information is needed.
Answer: FALSE
Diff: 1
Learning Outcome: Describe how global production and logistics decisions are made
Skill: Concept
Objective: 7
AACSB: Analytical thinking

84) Profit figures for individual country operations may obscure the real impact those
operations have on total global performance.
Answer: TRUE
Diff: 2
Learning Outcome: Describe how global production and logistics decisions are made
Skill: Concept
Objective: 7
AACSB: Reflective thinking

85) In developed countries, the percentage of the working-age population (using today's
standards) is expected to rise by 2050.
Answer: FALSE
Diff: 2
Learning Outcome: Describe how global production and logistics decisions are made
Skill: Concept
Objective: 7
AACSB: Application of knowledge

86) It is generally agreed that because of technical advancements, managers will not need
face-to-face communication in the future.
Answer: FALSE
Diff: 2
Learning Outcome: Describe how global production and logistics decisions are made
Skill: Concept
Objective: 7
AACSB: Information technology
87) When income inequality is high in a specific country, the per capita GDP figures are
more meaningful.
Answer: FALSE
Diff: 2
Learning Outcome: Describe how global production and logistics decisions are made
Skill: Concept
Objective: 3
AACSB: Application of knowledge

88) The number of computer industry firms located in California's Silicon Valley exemplifies
the concept of agglomeration.
Answer: TRUE
Diff: 2
Learning Outcome: Describe how global production and logistics decisions are made
Skill: Concept
Objective: 3
AACSB: Analytical thinking

89) What is the relationship between a company's international market and its production
location decisions? How do firms benefit from the use of scanning techniques when making
location decisions?
Answer: If a company develops a product that consumers find attractive, it must still find
production and transportation cost advantages so that it can price the product favorably
enough to sell it. These cost advantages may come from producing near the market, thus
allowing the company to sustain a long-term competitive advantage. To compare countries,
managers use scanning techniques based on broad variables that indicate opportunities and
risk. That way, decision-makers can perform a detailed analysis of a manageable number of
geographic locations. Scanning is useful in that a company might otherwise consider too few
or too many possibilities. When scanning, managers will look at external conditions in a host
country that could significantly affect the success or failure of a foreign enterprise. Scanning
can help managers determine whether a company will make a detailed study of the area, as
well as the terms under which it will initiate a project.
Diff: 3
Learning Outcome: Describe how global production and logistics decisions are made
Skill: Synthesis
Objective: 1, 2
AACSB: Reflective thinking
90) What is scanning? What opportunities and risks are most relevant to scanning?
Answer: To compare countries, managers use scanning techniques based on broad variables
that indicate opportunities and risk. When scanning, managers will look at external conditions
in a host country that could significantly affect the success or failure of a foreign enterprise.
Revenues less costs determine opportunities. From a broad scanning perspective, there are
variables that indicate the amount of revenue, cost factors, and risk that might be forthcoming
from one country to another. The factors that have the most influence on the placement of
marketing and production emphasis are sales expansion, economic variables, demographic
variables, resource acquisition, infrastructure, and the ease of transportation and
communications. Sales potential is probably the most important variable managers use in
determining where and whether to make an investment.
Companies weigh opportunities against risks when making decisions. Political risk may occur
because of changes in political leaders' opinions and policies, civil disorder, and animosity
between the host and other countries, particularly the firm's home country. Changes in
exchange rates or the ability to move funds out of a country may also affect an MNE. Other
types of risk that are usually considered include the risk of disease or natural disaster and the
competitive risk that develops from competitors' actions.
Diff: 3
Learning Outcome: Describe how differences in political economy influence economic
development
Skill: Synthesis
Objective: 2, 3
AACSB: Reflective thinking
91) In a short essay, discuss why simply examining a country's per capita GDP and its
population doesn't necessarily lead to a good estimate for potential demand.
Answer:
a. Obsolescence and leapfrogging of products: Consumers in emerging economies do not
necessarily follow the same patterns as those in higher-income countries. In many emerging
economies, consumers have leapfrogged the use of traditional telephones by jumping from
having no telephones to using cellular phones exclusively.
b. Prices: If prices of essential products are high, consumers may spend more than what one
would expect based on per capita GDP. The expenditures on food in Japan are higher than
would be predicted by either population or income level because food is expensive and work
habits promote eating out. However, if costs are high for a non-necessity, expenditures will
likely be lower.
c. Income elasticity: A common tool to predict total market potential is to divide the
percentage of change in product demand by the percentage of change in income in a given
country. The more that demand increases, the more elastic is the demand in response to
income change. Income elasticity varies by product and by income level.
d. Substitution: Consumers in a given country may have products or services that substitute
more conveniently in some countries than in others for the products that companies would
like to sell. For example, there are fewer automobiles in Hong Kong than one would expect
based on income and population because the crowded conditions make the efficient mass
transit system a desirable alternative to automobiles.
e. Income inequality: Where income inequality is high, the per capita GDP figures are less
meaningful, as most people have very little to spend and some people have substantial
income to spend.
f. Cultural factors and taste: Countries with similar per capita GDPs may have different
preferences for products and services because of values or tastes.
g. Existence of trading blocs: Although a country may have a small population and GDP, its
presence in a regional trading bloc gives its output access to a much larger market.
Diff: 3
Learning Outcome: Describe how global production and logistics decisions are made
Skill: Critical Thinking
Objective: 3
AACSB: Analytical thinking
92) In a short essay, discuss liquidity preference as it relates to monetary risk.
Answer: If a company's expansion occurs through direct investment abroad, exchange rates
on and access to the invested capital and earnings are key considerations. Liquidity
preference is the theory that investors usually want some of their holdings to be in highly
liquid assets, on which they are willing to take a lower return. Liquidity is needed in part to
make near-term payments, such as paying out dividends; in part to cover unexpected
contingencies, such as stockpiling materials if a strike threatens supply; and in part to be able
to shift funds to even more profitable opportunities, such as purchasing materials at a
discount during a temporary price depression.
Diff: 3
Learning Outcome: Describe how differences in political economy influence economic
development
Skill: Application
Objective: 3
AACSB: Analytical thinking

93) What is meant by liability of foreignness? How might this influence location and
allocation decisions?
Answer: Liability of foreignness describes the situation in which foreign companies have a
lower survival rate than local companies for many years after they begin operations. When a
company operates abroad, it usually has higher uncertainty than at home because the foreign
operations are in environments with which the company is less familiar. As a company gains
experience in operating in a particular country or in similar countries, it improves its
assessments of consumer, competitor, and government actions—thereby reducing its
uncertainty. Companies may reduce the risk of liability of foreignness by moving first to
countries more similar to their home countries. Alternatively, they may contract with
experienced companies to handle operations for them, limit the resources they commit to
foreign operations, and delay entry to many countries until they are operating successfully in
one or a few.
Diff: 3
Learning Outcome: Describe how global production and logistics decisions are made
Skill: Synthesis
Objective: 3, 6
AACSB: Analytical thinking

94) Compare the advantages of locating foreign operations to avoid where competitors have
gone versus locating where competitors are.
Answer: By being the first major competitor in a market, a company can gain the best
partners, locations, and suppliers. However, companies may gain other advantages by
locating where competitors already are. To begin with, the competitors may have performed
the costly task of evaluating locations, so a follower may get a "free ride." Moreover, clusters
of competitors in various locations attract suppliers, buyers, and specialized labor.
Diff: 3
Learning Outcome: Describe how global production and logistics decisions are made
Skill: Synthesis
Objective: 1, 3
AACSB: Analytical thinking
95) What problems are common with the published data available about different countries?
Answer: For the most part, incomplete or inaccurate published data result from the inability
of many governments to collect the needed information. Poor countries may have such
limited resources that other projects necessarily receive priority in the national budget.
Economic factors also hamper record retrieval and analysis. The result may be information
that is years old before it is made public. Cultural factors affect responses. Mistrust of how
the data will be used may lead respondents to answer incorrectly, particularly if questions
probe financial activities. However, not all inaccuracies are due to governmental collection
and dissemination procedures. People's desire and ability to cover up data on themselves—
such as unrecorded criminal activity—may distort published figures. Finally, researchers may
use poor collection and analysis methods.
Diff: 3
Learning Outcome: Describe how global production and logistics decisions are made
Skill: Application
Objective: 4
AACSB: Reflective thinking
96) What are the major types of published data that managers can use to compare countries?
Describe the tools available to managers for making country comparisons.
Answer: Market research and business consulting companies conduct studies for a fee in
most countries.
Some research organizations prepare fairly specific studies that they sell to any interested
company at costs much lower than for individualized studies. Most companies that provide
services to international clients publish reports. These reports usually are geared toward
either the conduct of business in a given area or some specific subject of general interest,
such as tax or trademark legislation. Governments and their agencies are another source of
information. Different countries' statistical reports vary in subject matter, quantity, and
quality. Numerous organizations and agencies are supported by more than one country. These
include the United Nations, the World Trade Organization, the International Monetary Fund,
the Organization for Economic Cooperation and Development, and the European Union. All
of these organizations have large research staffs that compile basic statistics as well as
prepare reports and recommendations concerning common trends and problems. Trade
associations connected to various product lines collect, evaluate, and disseminate a wide
variety of data dealing with technical and competitive factors in their industries. A number of
companies have information-retrieval services that maintain databases from hundreds of
different sources. For a fee, or sometimes for free at public libraries, a company can obtain
access to such computerized data and arrange for an immediate printout of studies of interest.
Two common tools for analysis are grids and matrices. A company may use a grid to
compare countries on whatever factors it deems important. The grid technique is useful even
when a company does not compare countries because it can set a minimum score necessary
for either investing additional resources or committing further funds to a more detailed
feasibility study. An opportunity-risk matrix is important as a reflection of the placement of a
country in comparison to other countries. The companies must determine which factors are
good indicators of its risk and opportunity and weigh them to reflect their importance.
Diff: 3
Learning Outcome: Describe how global production and logistics decisions are made
Skill: Synthesis
Objective: 4, 5
AACSB: Analytical thinking
97) In a short essay, compare the strategies of diversification versus concentration and
provide examples of situations in which each would be used.
Answer: Ultimately, a company may gain a sizable presence and commitment in most
countries; however, there are different paths to that position. Although any move abroad
means some geographic diversification, the term diversification in the context of location is
when the company moves rapidly into many foreign markets, gradually increasing its
commitments within each. At the other extreme, with a concentration strategy, the company
will move to only one or a few foreign countries until it develops a very strong involvement
and competitive position there.

When the growth rate in each market is high, a company usually should concentrate on a few
markets because it will cost a great deal to expand output sufficiently in each market. The
more stable sales and profits are within a single market, the less advantage there is to be
gained from a diversification strategy. Similarly, the more interrelated markets are, the less
smoothing is achieved by selling in each. If a company determines that it has a long
competitive lead time, it may be able to follow a concentration strategy and still beat
competitors into other markets. When marketing programs reach many countries, such as by
cable television or the Internet, a diversification strategy has advantages. Companies may
have to alter products and their marketing to sell in foreign markets, a process that, because
of cost, favors a concentration strategy. The more a company needs to control its operations
in a foreign country, the more it should develop a concentration strategy. If a company is
constrained by the resources it needs to expand internationally compared to the resources it
can muster, it will likely follow a concentration strategy.
Diff: 3
Learning Outcome: Summarize the main entry strategies and modes that businesses use to
enter into foreign markets
Skill: Critical Thinking
Objective: 6
AACSB: Analytical thinking

98) Why do companies often treat foreign reinvestment decisions differently than new
foreign investment decisions?
Answer: Companies treat decisions to replace depreciated assets or add to the existing stock
of capital from retained earnings in a foreign country somewhat differently from original
investment decisions. Once committed to a given locale, a company may find it doesn't have
the option of moving a substantial portion of the earnings elsewhere—to do so would
endanger the continued success of an operation in a given foreign location. Aside from
competitive factors, a company may need several years of almost total reinvestment and
allocation of new funds to one area in order to meet its objectives. Another reason a company
treats reinvestment decisions differently is that once it has experienced personnel within a
given country, it may believe they are the best judges of what is needed for that country, so
headquarters managers may delegate certain investment decisions to them.
Diff: 3
Learning Outcome: Describe how global production and logistics decisions are made
Skill: Critical Thinking
Objective: 6
AACSB: Analytical thinking

99) Why do companies engage in international harvesting or divestment?


Answer: Companies commonly reduce commitments in some countries because those
countries have poorer performance prospects than do others, a process known as harvesting
or divesting. Companies may divest by selling or closing facilities. They usually prefer
selling because they receive some compensation. A company that considers divesting because
of a country's political or economic situation may find few potential buyers except at very
low prices. In such situations, the company may try to delay divestment, hoping the situation
will improve. If it does, the firm that "waits out" the situation generally is in a better position
to regain markets and profits than one that forsakes its operations.
Diff: 3
Learning Outcome: Describe how global production and logistics decisions are made
Skill: Critical Thinking
Objective: 6
AACSB: Analytical thinking

100) Why do most companies examine expansion proposals one at a time rather than
comparing various expansion proposals? Do you think this is effective? Why or why not?
Answer: Three major factors restricting companies from comparing investment opportunities
are cost, time, and the interrelation of operations. Clearly, some companies cannot afford to
conduct many investigations simultaneously. If they are conducted simultaneously, they are
apt to be in various stages of completion at a given time. Further, in many cases they need to
respond quickly to an opportunity they had not anticipated, such as an unsolicited proposal or
limited offer from a government.
Diff: 3
Learning Outcome: Describe how global production and logistics decisions are made
Skill: Critical Thinking
Objective: 7
AACSB: Analytical thinking

You might also like